MBE - FINAL STUDY SET

Lakukan tugas rumah & ujian kamu dengan baik sekarang menggunakan Quizwiz!

29. The owner of a pet shop received an e-mail from a professional bird breeder that included the following offer: "Lovebirds! $120 a pair! Delivery in 4-6 weeks. Terms of sale, cash within 30 days of delivery." The shop owner e-mailed the bird breeder on April 26, agreeing to purchase one pair of lovebirds for $120. After looking over the e-mail the next day, the bird breeder sent a reply e-mail to the shop owner indicating that there was a mistake in the initial e-mail; it should have read "$120 per bird." The bird breeder went on to state that she would ship the pair of birds to the shop owner if she would pay the additional $120. In her e-mailed reply, the shop owner authorized the breeder to ship the pair of lovebirds and agreed to pay the additional $120, but she noted that they must be delivered on or before May 21. The bird breeder immediately e-mailed the shop owner agreeing to deliver the birds to her by May 21. However, the birds did not arrive at her shop until June 1, and the shop owner refused to accept them. In an action by the bird breeder against the shop owner for breach of contract, which of the following awards is most likely? (A) Nothing, because the birds did not arrive at the shop owner's store by May 21. (B) $120, because the promise to pay the additional amount with a new delivery date was only a counteroffer. (C) $120, because the bird breeder had a preexisting legal obligation to ship the birds to the shop owner. (D) $240, because consideration is not required to modify a contract for the sale of goods.

ANSWER: (A) Nothing, because the birds did not arrive at the shop owner's store by May 21. The bird breeder will most likely recover nothing in the suit. The bird breeder is not entitled to recover anything because she has breached the modified contract. As discussed below, the parties validly modified the contract and agreed to a delivery date. The breeder's failure to tender the birds by that date constituted a breach of the contract, which excused the shop owner's duty of counterperformance (that is, to pay the additional amount). Perfect tender is required under the UCC, which governs contracts for the sale of goods. This means that if the goods or their delivery fail to conform to the contract in any way, there is a breach. In that case, the buyer may accept all, reject all, or accept any commercial units and reject the rest. Therefore, the breeder breached the contract, and the shop owner was free to reject the birds. (B) is incorrect because under the UCC, an acceptance is valid even if it contains additional terms. Therefore, the shop owner's promise would be enforceable if not for the breach. Note that even if the common law rule applied and the shop owner's acceptance were considered a counteroffer, it was accepted by the breeder when the breeder replied by e-mail agreeing to the delivery date. (C) is incorrect because the common law preexisting legal duty rule does not apply to modifications under the UCC. Under the UCC, a promise to modify an existing contract is enforceable even without consideration. The only requirement is that the proposal to modify be made in good faith. Here, the proposal was made in good faith, because it was to correct an error in the email. (D) is incorrect because the nonconforming delivery was a breach by the breeder. If the breeder were not in breach, (D) would be correct.

(Property) 17. A landowner owned a large tract of mineral-rich land in a sparsely populated area. He entered into a lease with a prospector who was interested in developing the land for mining. The term of the lease was two years and gave the prospector an option to buy the property at any time after the first year. The prospector did not record the lease. Six months later, the prospector left the land for a period of time to prospect in Mexico, leaving no goods on the land that would identify him. The landowner then conveyed the property in fee simple to a developer, who had inspected the property while the prospector was in Mexico and was unaware of the prior transaction. The developer did not immediately record her deed. After three months in Mexico, the prospector returned and recorded his lease. As he was making preliminary mining plans on the land, he encountered the developer. A statute in the jurisdiction provides, in part: "No conveyance or mortgage of an interest in land, other than a lease for less than one year, is valid against any subsequent purchaser for value without notice thereof whose conveyance is first recorded." (RACE-NOTICE) If the developer brings an action to quiet title, how should the court rule? (A) The developer takes title subject to the prospector's leasehold interest and his option to purchase, because the prospector acquired his interest first. (B) The developer takes title subject to the prospector's leasehold interest but not his option to purchase, because he does not yet have the power to exercise the option. (C) The developer takes title subject to the prospector's leasehold interest and option to purchase regardless of whether she now records, because she will have had notice of the prospector's interest before recording. (D) The developer takes title free of the prospector's leasehold interest and option, because the prospector failed to record before the developer purchased the property without notice of his interest.

ANSWER: (A) The developer takes title subject to the prospector's leasehold interest and his option to purchase, because the prospector acquired his interest first. The developer's ownership of the land is subject to the prospector's interest because the prospector's interest was first in time. The common law rule that priority is given to the grantee who was first in time still applies unless operation of the jurisdiction's recording statute changes the result. The statute in this question is a race-notice statute, under which a subsequent bona fide purchaser is protected only if she records her interest before the prior grantee does. While the developer is a bona fide purchaser, she must still win the race to the recording office to prevail over the prospector's prior interest. Since neither party has recorded in this fact pattern, the developer will take the property subject to the prospector's prior interest. (B) is wrong because the prospector's option is treated as an interest in land just like his leasehold interest, regardless of the fact that it cannot yet be exercised. (C) is wrong because the developer probably would prevail if she were to record now. Under the recording statute, it is irrelevant that the developer became aware of the prospector's interest at a later point; at the time of the conveyance to her, she did not have notice of his interest and therefore qualifies as a bona fide purchaser. Had she recorded when she first encountered the prospector, she would have prevailed. Even if she were to record now, she could still prevail if the prospector does not record first. (D) is wrong because it states the result under a notice statute. In contrast to a notice statute, the race-notice statute in the question would still permit the prospector to prevail even though he failed to record before the developer purchased the property, as long as the developer does not record before he does.

(Torts) 8. A homeowner called a tree trimmer to his house to trim some branches on a large pine tree in the homeowner's backyard. The tree trimmer trimmed off several large branches and pruned several others. The homeowner paid the tree trimmer, who left and did not return. A week after the tree trimmer trimmed the tree, one of the pruned branches split from the trunk of the tree and landed on a neighbor's car. If the neighbor brings suit against the tree trimmer for damage to his car, what is the best defense for the tree trimmer?

ANSWER: (B) He could not reasonably foresee that the branch would split off and fall.

(Torts) 17. The owner of a valuable painting hired professional movers to transport it to an auction house when she decided to sell it. As the movers were carrying it to their van, a window air conditioner that a tenant had been trying to install fell out of his second floor window and crashed through the painting and onto the ground. The owner had been watching from across the street and saw her painting destroyed. She became extremely upset and needed medical treatment for shock. If the owner brings a claim for negligent infliction of emotional distress against the tenant, is she likely to recover? (A) Yes, because she suffered physical symptoms from her distress. (B) Yes, because she was a foreseeable plaintiff. (C) No, because she was not within the zone of danger. (D) No, because she suffered no physical impact.

ANSWER: (C) No, because she was not within the zone of danger. Because the owner was across the street at the time of impact, she was not within the zone of danger, thus precluding her recovery for negligent infliction of emotional distress. A defendant breaches a duty to avoid negligent infliction of emotional distress when he creates a foreseeable risk of physical injury to the plaintiff through causing a threat of physical impact that leads to emotional distress. Damages generally are recoverable only if the defendant's conduct causes some physical injury, rather than purely emotional distress (although a severe shock to the nervous system that causes physical symptoms is sufficient). If plaintiff's distress is caused by threat of physical impact to her, she must have been within the zone of danger. Here, the owner witnessed the air conditioner striking her painting from across the street. This vantage point placed her outside the zone of danger from the falling air conditioner. Thus, the owner cannot recover for negligent infliction of emotional distress. It is true that, as implied by (A), there usually can be no recovery for this tort absent some accompanying physical consequences. However, even though the owner did suffer physical symptoms from her distress, she cannot recover because she was not within the zone of danger. Therefore, (A) is incorrect. (B) is incorrect because the owner's distance from the accident makes her an unforeseeable plaintiff, because it is unforeseeable that someone on the other side of the street would suffer physical impact or the threat thereof from the tenant dropping the air conditioner. Had she been standing near the painting when the air conditioner fell, she would be in the zone of danger and a foreseeable plaintiff. Furthermore, even though it may be foreseeable that she would suffer emotional distress from seeing her painting destroyed, she could not recover for that distress in a negligent infliction of distress claim because such a claim requires a threat of physical injury; threat of property damage does not suffice. (D) is incorrect because physical impact is not required for this tort; the threat of impact is enough. Therefore, even in the absence of impact, the owner could recover if she had been within the zone of danger from the tenant's negligence.

(Property) 20. A photographer borrowed $100,000 from a bank, secured by a mortgage on his home, to build a studio and darkroom in the home. The bank properly recorded the mortgage. After completing this project, the photographer decided to remodel his kitchen and borrowed $25,000 from a lending company, also securing the loan with a mortgage on his home. The lending company did not record its mortgage. After the remodeling was complete, the photographer borrowed $15,000 from an investor, secured by a mortgage on his home, to redo his in-ground pool. Learning of this transaction, the lending company raced to the recording office and recorded its mortgage. The next day, the investor recorded its mortgage. A few months later, the photographer defaulted on all three mortgages, having not made any principal payments. The lending company brought a foreclosure action, joining the investor in the proceeding. The foreclosure sale resulted in $150,000 in proceeds after all expenses and fees were paid. A statute of the jurisdiction in which the photographer's home is located provides: "Any conveyance of an interest in land shall not be valid against any subsequent purchaser for value, without notice thereof, unless the conveyance is recorded." (NOTICE) Which of the following statements is true? (A) The bank is entitled to $100,000 of the foreclosure proceeds, the lending company is entitled to $25,000 of the proceeds, the investor is entitled to $15,000 of the proceeds, and the photographer is entitled to the remaining $10,000 in proceeds. (B) The buyer at the foreclosure sale will take the home subject to the bank's mortgage, the lending company is entitled to $25,000 of the proceeds, the investor is entitled to $15,000 of the proceeds, and the photographer is entitled to the remaining $110,000 in proceeds. (C) The buyer at the foreclosure sale will take the home subject to the bank's and the investor's mortgages, the lending company is entitled to $25,000 of the proceeds, and the photographer is entitled to the remaining $125,000 in proceeds. (D) The buyer at the foreclosure sale will take the home subject to the bank's and the investor's mortgages, the lending company is entitled to $25,000 of the proceeds, and the buyer at the foreclosure sale is entitled to the remaining $125,000 in proceeds.

ANSWER: (C) The buyer at the foreclosure sale will take the home subject to the bank's and the investor's mortgages, the lending company is entitled to $25,000 of the proceeds, and the photographer is entitled to the remaining $125,000 in proceeds. The buyer at the foreclosure sale will take the home subject to the bank's and the investor's mortgages, the lending company is entitled to $25,000 of the proceeds, and the photographer is entitled to the remaining $125,000 in proceeds. When an interest is foreclosed, after the expenses and fees are paid, the proceeds of the sale are first used to pay the principal and accrued interest on the loan that was foreclosed, next to pay off any junior liens, and finally any remaining proceeds are distributed to the mortgagor. (A) is wrong because the bank's interest, an interest senior to the lending company's, is not affected by the foreclosure. As a senior interest, the bank was not a necessary party to the foreclosure action and did not need to be named in the foreclosure action. Thus, the bank is not entitled to a share of the proceeds, and its lien continues on the property in the buyer's hands. (B) is wrong because the investor's mortgage is also senior to the lending company's, and as a result, the buyer also takes subject to the investor's mortgage. Under a notice statute, which this jurisdiction has, a subsequent bona fide purchaser prevails over a prior grantee who fails to record. The important fact under a notice statute is that the subsequent purchaser had no actual or constructive notice at the time of the conveyance (or mortgage), not at the time of recording. Mortgagees for value are treated as "purchasers." Here, when the mortgage on the property was granted to the investor, it had neither actual nor constructive notice of the mortgage given to the lending company. The fact that the lending company recorded its mortgage first is irrelevant. Thus, the investor was a bona fide purchaser and would be entitled to protection under the statute. (A) and (D) are wrong because the mortgagor (the photographer) rather than the buyer at the foreclosure sale is entitled to the surplus proceeds.

(Criminal Procedure) 18. The defendant was at a bus station on a stopover between two cities along a known drug trafficking route. An officer of the Federal Drug Enforcement Administration noticed that the defendant was tightly clutching a small bag and making numerous phone calls from his cell phone, which the officer knew through his training are actions within the drug courier profile. The officer approached the defendant, identified himself, and began asking the defendant questions about his destination. The defendant agreed to follow the officer into an office for an inspection of his bag. In the office, the officer searched the defendant's bag and found nothing suspicious. He then proceeded to pat down the defendant and felt a pouch around the defendant's stomach, which proved to contain cocaine. The defendant was charged with possession of cocaine. If the defendant seeks to have the cocaine excluded from evidence, how should the court rule? (A) The cocaine is admissible, because of the emergency conditions arising from the growing menace of drugs to the public. (B) The cocaine is admissible, because, in consenting to the search of his bag, the defendant impliedly consented to a body search. (C) The cocaine is inadmissible, because the officer had neither probable cause nor reasonable suspicion to search the defendant. (D) The cocaine is inadmissible, because the officer had no right to search the bag.

ANSWER: (C) The cocaine is inadmissible, because the officer had neither probable cause nor reasonable suspicion to search the defendant. The cocaine is inadmissible because it is the product of an unconstitutional search of the defendant. The Fourth Amendment prohibition against unreasonable searches and seizures applies to an investigatory detention and any type of search during the detention. Under Terry v. Ohio (1968), police have the authority to briefly detain a person for investigative purposes, even if they lack probable cause to arrest, as long as they have an articulable and reasonable suspicion of criminal activity. However, an investigatory detention does not create the right to search the person being detained. The officer may conduct a protective frisk (a patdown of the outer clothing) only if he reasonably believes that the person may be armed and presently dangerous. A full search of the person is only permitted if the detention establishes probable cause for a lawful arrest. In this case, the officer may have had sufficient grounds to detain the defendant and to ask him questions. Whether police have a reasonable suspicion—supported by articulable facts—of criminal activity is judged by the totality of the circumstances. [United States v. Sokolow (1989)] Here, the defendant's conduct may have made the officer reasonably suspicious that he was smuggling drugs. However, after finding nothing in his search of the bag, and having no reason to believe that the defendant was armed and presently dangerous, the officer had neither probable cause nor reasonable suspicion to either search or frisk the defendant. Hence, under the exclusionary rule, the cocaine obtained as a result of the unlawful search of the defendant is inadmissible against him. (A) is incorrect because there is no general emergency exception justifying searches without probable cause, and the search at issue here had none of the exigent circumstances that the Supreme Court has relied on in prior cases to permit warrantless searches (such as the hot pursuit of a fleeing felon or the evanescent nature of the evidence). (B) is incorrect because the scope of a search permitted by consent is limited by the scope of the consent. The defendant's consent to the search of his bag created no implication of a consent to a body search. (D) is incorrect because the officer had the defendant's consent to search his bag. Police may conduct a valid and warrantless search if they have a voluntary consent to do so from the individual being searched; the defendant's consent here satisfies these requirements.

(Criminal Procedure) 33. A woman was stopped at a police roadblock to check for drunk drivers. The police were stopping every third vehicle that came through the checkpoint, and the woman's car turned out to be a third vehicle. After failing a field sobriety test, the woman was arrested and charged with driving while intoxicated. Was the stop of the woman's car legal? A. Yes, because the car was stopped at a fixed checkpoint to check for drunk drivers. B. Yes, because temporarily stopping a car does not constitute a seizure of the automobile. C. No, because not every car was being stopped. D. No, because there was no probable cause to stop the vehicle.

ANSWER: A. Yes, because the car was stopped at a fixed checkpoint to check for drunk drivers.

(Evidence) 20. The defendant is on trial for assault with a deadly weapon. The sole prosecution witness is the victim, who testifies as to his version of the events leading up to and including the charged assault. The defense's first witness contradicts the victim's testimony that the defendant engaged in an unprovoked attack. The witness testifies that the victim pulled a knife on the defendant and that the defendant, in defending himself, wrested the knife away and accidentally stabbed the victim. The defense's next and final witness intends to testify that the defendant's reputation in the community for honesty and veracity is very good. Aware of the intended testimony, the prosecutor moves in limine to exclude it. How should the court rule? A For the state, because the defendant may not introduce evidence of his character to prove that he acted in conformity therewith. B For the state, because the testimony as to the defendant's honesty and veracity is irrelevant. C For the defendant, because a criminal defendant may put his character in issue. D For the defendant, because a criminal defendant's reputation for honesty and veracity is always at issue.

(B) For the state, because the testimony as to the defendant's honesty and veracity is irrelevant. The court should rule for the state. A criminal defendant, to show his innocence of the charged crime, may call a qualified witness to provide reputation or opinion testimony regarding the defendant's good character for a trait involved in the case (i.e., to prove that he acted in conformity with that good trait during the events at issue). Therefore, (A) is incorrect. Here, however, the defendant is charged with a crime of violence, so his character for honesty and veracity is not pertinent to the case. Furthermore, although any witness may be impeached with reputation or opinion evidence of his bad character for honesty and veracity, the defendant did not testify. For these reasons, the proffered evidence is irrelevant. Thus, (C) is incorrect. A criminal defendant may offer evidence of character for a certain trait only when the trait is relevant to the charges, and so (D) is a misstatement of law.

6. To fight drug abuse, a state enacted a statute forbidding the selling of model airplane glue to anyone under the age of 18 except in small quantities in prepackaged model kits. Violation of the statute was penalized by fines or, in cases of multiple violations, possible imprisonment. The statute also required that all elementary and secondary schools licensed by the state provide comprehensive drug education programs. Neither the legislature nor the courts of the state have abolished the common law tort defense of assumption of the risk. The owner of a hobby shop in the state sold a large tube of airplane glue to a 15-year-old boy who reasonably appeared to be at least 18 years old. The boy had received drug education in his school, as mandated by the statute, including coverage of the dangers of glue sniffing. The boy understood the anti-drug instruction, but he wanted to experience it for himself. The boy sniffed the glue repeatedly and suffered permanent brain damage. If the boy's parents file suit on the boy's behalf against the store owner, for whom is the court likely to rule?

ANSWER: (A) For the boy, because the store owner violated the statute when she sold the glue to the boy. The boy will prevail because the store owner's violation of the statute resulted in the boy's injuries. The applicable standard of care in a cause of action can be established by proving the applicability to that action of a statute providing for a criminal penalty. If this is done, a clearly stated specific duty imposed by the statute will replace the more general common law duty of care. For the statutory standard to be applicable, the plaintiff must show that (i) he is within the class intended to be protected by the statute, and (ii) the statute was designed to prevent the type of harm that the plaintiff suffered. Here, the statute clearly prohibited the transaction that took place between the boy and the store owner. It establishes the standard of care because the boy is a minor purchasing a large quantity of model airplane glue, and the serious injury he suffered from sniffing the glue was one of the harms that the statute was designed to prevent. The effect of establishing a violation of the statute is that a conclusive presumption of duty and breach of duty is established. The boy should then be able to establish that the store owner's sale of the glue was the actual cause and proximate cause of the boy's injuries, completing the prima facie case of negligence on the store owner's part. (B) is incorrect because minors such as the boy who were experimenting with glue sniffing were a primary target of the statute. (C) is incorrect because the boy's assumption of the risk is not a defense under these circumstances. The facts indicate that the state has retained the common law tort defense of assumption of the risk. Under this defense, a plaintiff will be denied recovery in a negligence action if he either expressly or impliedly knew of the risk of injury and voluntarily proceeded in the face of the risk. However, courts refuse to permit an assumption of risk defense in some situations because of public policy considerations. When a statute applies and is enacted to protect a class, members of that class will not be deemed to have assumed any risk. Here, even though the boy was aware of the danger when he voluntarily sniffed the glue, the statute was enacted to protect minors such as the boy from the dangers of glue sniffing. Thus, the store owner cannot rely on assumption of risk as a defense. (D) is incorrect because the statute does not provide for a reasonable mistake to excuse its violation. If the common law duty of reasonable care were applicable here, the reasonableness of the store owner's mistake would be relevant. However, the statute's specific duty replaces the more general duty of reasonable care, and violation of a statutory standard will only be excused where compliance would cause more danger than violation or would be beyond the defendant's control. Neither situation is indicated here, so the statute applies

(Evidence) 13. A plaintiff brought an action against a defendant for property damages, alleging that the defendant's car nicked the side of the plaintiff's truck while the defendant was changing lanes on an expressway. At trial, the defendant sought to introduce evidence of her good driving record. Is the evidence admissible? (A) No, because it is character evidence. (B) No, because it is self-serving. (C) Yes, because it is character evidence. (D) Yes, because it is habit evidence.

ANSWER: (A) No, because it is character evidence. The driving record is inadmissible because it is being offered as character evidence. In a civil case, evidence of character to prove the conduct of a person in the litigated event is generally not admissible. The slight probative value of character is outweighed by the dangers of prejudice and distracting the jury from the main issues. Therefore, circumstantial use of prior behavior patterns for the purpose of drawing the inference that a person has a particular character trait and that, at the time and place in question, she probably acted in conformity with it is not permitted. Evidence of the defendant's good driving record is being offered to show that she is a careful driver and to raise the inference that, when the accident occurred, she was acting in conformity with that trait. This constitutes impermissible use of character evidence and is inadmissible. (B) is incorrect because evidence is not excludable because it is self-serving. Virtually all evidence is self-serving to the party offering it. (C) is incorrect because it is based on the mistaken assumption that character evidence is admissible. As stated above, character evidence is generally inadmissible in a civil case. It is admissible in a civil case only when proof of a person's character, as a matter of substantive law, is an essential element of a claim or defense (e.g., in a defamation case). The defendant's character is not in issue, so the driving record is inadmissible. (D) is incorrect because this is not habit evidence. Habit describes one's regular response to a specific set of circumstances. Character describes one's disposition with respect to general traits. The defendant's good driving record describes a general behavior pattern of careful driving, rather than a regular response to a specific set of circumstances. Thus, this is character evidence, rather than habit evidence.

2. A child was severely injured at an amusement park when she was ejected from a ride that went slightly off its track. The ride malfunctioned as a result of a manufacturer's defect, but had the child been properly secured in the ride's seatbelt by one of the ride operators, she would not have been injured. The child was unable to identify which ride operator improperly buckled her in. In the child's suit against the amusement park, who will win?

ANSWER: (A) The child will win, because a ride operator failed to use reasonable care in securing the seatbelt. Because the ride operator was negligent in improperly securing the child, the amusement park is vicariously liable under the doctrine of respondeat superior. This doctrine imposes liability on an employer for the tortious conduct of its employee occurring within the scope of the employment relationship. Here, securing the children in the ride was one of the ride operator's tasks. Since this task was performed negligently and this negligence was one of the causes of the child's injuries, the amusement park will be liable. (B) is incorrect because the failure to make a reasonable inspection of the ride is not the negligent conduct suggested by the facts; rather, it was the failure to belt the child in securely that was negligent. Nothing in the facts indicates that a reasonable inspection would have disclosed the defect. (C) is incorrect because the amusement park will be liable even if the child cannot identify which specific employee was negligent. Through the doctrine of res ipsa loquitur, the child can establish breach of duty just by the fact that she was ejected from the ride after she had been strapped in; in other words, an inference of negligence is established because the accident causing her injury is the type that would not normally occur unless someone was negligent. The other two elements required for res ipsa loquitur are that the plaintiff was free of fault, which is easy to show in this case, and that the negligence was attributable to the defendant; i.e., that this type of accident ordinarily happens because of the negligence of someone in the defendant's position. If the plaintiff were suing a ride operator individually, this requirement would prevent her from using res ipsa loquitur in most jurisdictions because she could not establish that that particular operator was negligent. However, the amusement park will be liable under the doctrine of respondeat superior regardless of which of its employees was negligent, because the amusement park, through its employees, did have exclusive control over the ride; therefore, the negligence is attributable to the amusement park. (D) is incorrect because the amusement park will lose even though the injury to the child was caused by a defect present at the time the ride was purchased. The negligence of one of the park's employees was also a cause of the child's injuries.

(Evidence) 9. A defendant is on trial in federal court, charged with having sold cocaine to an undercover agent. The defendant calls a witness to the stand who testifies that she was with the defendant in another state on the date of the alleged drug sale. Following her testimony, the prosecution seeks to introduce the record of judgment of the witness's seven-year-old fraud conviction. The defendant's attorney objects. Is the record of judgment admissible? (A) Yes, because it goes to the witness's credibility. (B) Yes, provided no appeal is pending. (C) No, because the record is inadmissible hearsay. (D) No, because, for impeachment, a specific act of misconduct cannot be shown by extrinsic evidence.

ANSWER: (A) Yes, because it goes to the witness's credibility. The court should find the record admissible. A witness other than the accused may be impeached by (i) any felony conviction (unless the judge determines that its probative value is substantially outweighed by Rule 403 considerations) or (ii) conviction of any other crime requiring proof or admission of an act of dishonesty or false statement. [Fed. R. Evid. 609(a)] Embezzlement is both. The fact that an appeal is pending does not affect the admissibility of the conviction; thus, (B) is incorrect. The official record of judgment is always admissible proof that the judgment was entered; (C) is therefore incorrect. While the proposition in (D) is correct, it does not apply to proof of prior convictions. A prior conviction may be shown not only by direct or cross-examination of the witness but also by introducing a record of judgment.

(Torts) 15. The superintendent of a waste management company that operated a landfill noted that some children who lived in a nearby residential development had taken to sledding down the snow-covered mounds of dirt that were piled on the site. A construction company needing dirt fill for a highway project had offered to remove the mounds of dirt at minimal cost but the superintendent had not yet arranged for their removal. In the meantime he posted numerous signs around the landfill site that stated in bold letters, "NO TRESPASSING—NO SLEDDING." Despite the signs, which he saw and read, a 10-year-old neighborhood child sledded down one of the mounds of dirt and was propelled onto a busy highway adjacent to the landfill, where he was struck by a car and seriously injured. Is the waste management company liable for the child's injuries? (A) Yes, because the company could have had the piles of dirt removed at minimal cost. (B) Yes, because the company created a public nuisance. (C) No, because the child was a trespasser. (D) No, because the child read and understood the warning signs and appreciated the danger

ANSWER: (A) Yes, because the company could have had the piles of dirt removed at minimal cost. The waste management company breached its duty of ordinary care to the child because it could have had the piles of dirt removed at minimal cost. Most courts impose upon a landowner the duty to exercise ordinary care to avoid reasonably foreseeable risk of harm to children caused by artificial conditions on his property. Under the general rule, to assess this special duty upon the owner or occupier of land in regard to children on his property, the plaintiff must show the following: (i) there is a dangerous condition present on the land of which the owner is or should be aware; (ii) the owner knows or should know that young persons frequent the vicinity of this dangerous condition; (iii) the condition is likely to cause injury (i.e., is dangerous) because of the child's inability to appreciate the risk; and (iv) the expense of remedying the situation is slight compared with the magnitude of the risk. If all of these elements are present, the child has a cause of action under the "attractive nuisance" doctrine. Under these facts, the mounds of dirt sloping down toward the highway constituted a dangerous condition and the superintendent was aware that children were sledding down the mounds of dirt next to a busy highway. Because the company could have had the dirt removed at minimal cost, the expense of remedying the situation would have been slight compared to the magnitude of the risk, making the company liable under the attractive nuisance doctrine. (B) is incorrect because the mounds of dirt did not constitute a public nuisance, which is unrelated to the attractive nuisance doctrine. A public nuisance is an act that unreasonably interferes with the health, safety, or property rights of the community. The dirt did not unreasonably interfere with the safety rights of the community. The dirt was piled on a site next to a highway and did not create an obstruction for pedestrians or drivers. (C) is incorrect because even though the child was a trespasser, infant trespassers come within an exception to the standard duty of care owed by an owner or occupier of land to a trespasser for artificial conditions. Thus, the child's status as a trespasser does not prevent his recovery. (D) is incorrect because even though the child read the sign, the sign did not alert him to the danger that the piles of dirt created, and there is no other evidence that he was aware of and appreciated the danger of sliding onto the highway. For assumption of the risk to be a defense, the plaintiff must be aware of the risk and then voluntarily assume that risk.

7. The owner of a shopping center entered into a two-year lease with a tenant who was opening a greeting card shop. When the lease still had 14 months left to run, the tenant decided to sell his business and inventory to a successor. When the tenant told the mall owner that he planned to assign his lease to the successor, the mall owner objected, citing a clause in the lease requiring consent to any assignment and providing that any attempted assignment without consent is grounds for terminating the lease. The tenant assigned the lease to the successor anyway. The successor operated the card store in the same manner as the tenant, the only difference being a change of the store's name. Does the mall owner have any recourse against the tenant or the successor?

ANSWER: (A) Yes, he may evict the successor and collect damages from the tenant because nonassignment clauses are valid and enforceable. The nonassignment clause is valid and the mall owner may enforce it by terminating the lease and regaining possession of the store. Nonassignment clauses in leases are valid and enforceable even though strictly construed, and allow the landlord in most states to refuse to consent to an assignment even if the refusal is unreasonable. If a tenant makes an assignment in violation of a nonassignment clause in the lease, the transfer is not void. However, the landlord may terminate the lease if specifically provided by the nonassignment clause. Alternatively, he may sue for damages if he can prove any. Only (A) upholds the validity of the nonassignment clause. (B) is incorrect. While most courts require a landlord to mitigate damages where a tenant unjustifiably abandons the property, this duty does not affect the landlord's power to terminate the lease because of a breach of the lease agreement. The tenant will still be liable to the mall owner for any damages from the unauthorized assignment, including the cost of evicting the successor. (C) is incorrect because most states do not require that a landlord's refusal to accept a new tenant be reasonable. Furthermore, the fact that the successor is operating the same type of business does not mean that the mall owner has suffered no harm, since she might be a much poorer credit risk than the tenant. (D) is incorrect. As an exception to the general rule that any restriction on the transferability of a legal interest in property is void, a provision in a lease prohibiting the lessee's assignment or subletting of her leasehold interest without the consent of the landlord is given effect in all jurisdictions.

1. A retail store ordered 100 women's swimsuits at $10 each, as advertised in the catalog of a swimwear manufacturer. The manufacturer shipped 40 swimsuits to the store, along with a letter stating in relevant part: "We have shipped you 40 swimsuits at $10 each in response to your recent order. Please remit $400. Be informed that limited inventory will prevent us from being able to ship any additional suits at this time or at any time during this year's beach season." The store took the 40 suits and began to sell them. The store immediately sought an alternate supplier of swimsuits. The best price it could obtain was $11 per suit from a different company. The store ordered, received, accepted, and paid for 60 suits at $11 each from the other company. The store has refused to pay the manufacturer the $400, and the manufacturer has sued for payment. What is the manufacturer entitled to recover?

ANSWER: (B) $340, the contract price for the 40 suits, less the store's cover damages. The manufacturer is entitled to $340. As a general rule, advertisements, catalogs, and other price quotations are construed as invitations for offers rather than offers. If the language of the catalog could be construed as a definite promise to specific offerees, a court might construe it as an offer, but there is no evidence in this question for such an interpretation. The store's order is therefore an offer to purchase 100 swimsuits. Because goods are involved, UCC Article 2 applies. Under UCC section 2-206(1)(b), an offer to buy goods for prompt shipment invites acceptance either by a promise to ship or by prompt shipment of conforming or nonconforming goods. In this case, the manufacturer shipped goods that did not conform to the quantity term of the store's offer. The Code provides that the shipment of nonconforming goods amounts to both an acceptance of the offer and a breach of the newly formed contract unless the shipper precedes or accompanies the shipment with notice that it is not accepting the offer and is offering the shipment as an accommodation. Proper accommodation notice makes the shipment a counteroffer, rather than an acceptance. The letter sent by the manufacturer stating that it will be able to ship only 40 suits is not proper accommodation notice. Nothing indicates that the manufacturer is making a counteroffer rather than an acceptance. In fact, the demand for payment suggests that the manufacturer believes there is a contract. Without sufficient accommodation notice, a nonconforming shipment is both an acceptance of the offer and a breach of contract, entitling the nonbreaching party to damages. Thus, in this case, the store is entitled to deduct the $60 in damages from its payment. [See UCC §2-711] (A) is incorrect because, as stated above, the letter was not proper accommodation notice. If the manufacturer had stated that the shipment was intended as an accommodation and the store then accepted the suits, the manufacturer would have been entitled to the full $400. In that case, the accommodation shipment would be a counteroffer, and the contract would be for 40 swimsuits. (C) is incorrect because the store accepted the 40 swimsuits. Once a buyer accepts goods, payment is due at the contract rate, not fair market value. The acceptance does not, however, bar a claim for damages. (D) too is incorrect because the store accepted the suits. When a buyer receives nonconforming goods, it may accept all, reject all, or reject some and keep some. If, as here, the buyer chooses to accept the goods, payment is due at the contract rate—minus damages.

(Evidence) 5. The defendant is being tried for murder in the bludgeoning death of his brother. The defendant denies any involvement in the crime. He calls a witness to the stand, who testifies that, in his opinion, the defendant is a nonviolent, peaceable man. Which of the following, if offered by the prosecution, would most likely be admissible? (A) A neighbor's testimony that the defendant has beaten his wife on several occasions. (B) A police officer's testimony that the defendant has a general reputation in the community as a violent person. (C) A neighbor's testimony that the defendant has a reputation for being untruthful. (D) Evidence that the defendant has a conviction for aggravated battery.

ANSWER: (B) A police officer's testimony that the defendant has a general reputation in the community as a violent person. Testimony of the defendant's reputation as a violent person is admissible to rebut the defendant's character evidence. The general rule is that the prosecution cannot initiate evidence of the bad character of the defendant merely to show that he is more likely to have committed the crime of which he is accused. However, if the defendant puts his character in issue by having a character witness testify as to his opinion of the defendant, the prosecution may rebut with evidence of the defendant's bad character. One means of rebutting a defendant's character evidence is by calling qualified witnesses to testify to the defendant's bad reputation for the particular trait involved in the case. Here, the defendant put his character in issue by having his witness testify to the defendant's nonviolent nature, which is relevant to whether he committed the crime charged. The prosecution, assuming that it can show that the police officer has knowledge of the defendant's reputation in the community, can have the officer testify that the defendant had a reputation as a violent person. (A) is incorrect because the prosecution cannot prove the defendant's bad character for the pertinent character trait with evidence of specific bad acts like beating his wife. The prosecution may only introduce character evidence in the form of reputation or opinion testimony after the defendant has first introduced evidence of his good character. The prosecution may also cross-examine the defense witness with questions about specific acts of misconduct by the defendant, but the prosecution may not prove those acts with extrinsic evidence. (C) is incorrect. While the defendant has "opened the door" to evidence of his bad character by presenting testimony of his good character, the evidence must pertain to the particular trait involved in the case. Here, the defendant's capacity for violence has been placed in issue by the defendant, but his reputation for truthfulness is not relevant to whether he has committed the crime for which he is charged. (And because the defendant has not placed his credibility in issue by taking the stand as a witness, his reputation for truthfulness cannot be offered for impeachment purposes.) (D) is incorrect because the basic rule is that when a person is charged with one crime, extrinsic evidence of his other crimes or misconduct is inadmissible if offered solely to establish a criminal disposition, regardless of whether the defendant has placed his character in issue. [Fed. R. Evid. 404(b)] While evidence of other crimes is admissible if it is independently relevant to some other issue (e.g., motive, intent, or identity), the defendant's battery conviction in this case appears to have no relevance other than as evidence of his violent disposition. It is therefore inadmissible.

(Torts) 20. A state statute required that anyone performing construction work for another in exchange for value must either have a contractor's license or work for someone who is licensed. A homeowner wishing to remodel his kitchen and living room knew that a friend had previously worked for a licensed contractor for many years and was extremely knowledgeable about her trade, so he offered her money to assist in the project. When the friend explained that she would not be able to get a contractor's license in time to do the work, the homeowner responded that he did not care about her lack of a license. The friend began working in the homeowner's house, and parked her truck containing all her tools and materials in his driveway. Late in the day, a meteoroid fell from the sky and struck a can of solvent lying in the bed of her pickup. The solvent exploded from the force of the impact, setting the truck afire, which itself exploded when the gas tank caught fire, in turn igniting the homeowner's house. Is the friend liable for the damage to the house? (A) No, because the homeowner assumed the risk of any property damage when he knowingly solicited her to work without a contractor's license. (B) No, because no carelessness on her part was a legal cause of the property damage to the homeowner. (C) Yes, because her lack of a contractor's license is negligence per se. (D) Yes, because there would have been no damage to the homeowner's house but for the friend's decision to park her truck in his driveway.

ANSWER: (B) No, because no carelessness on her part was a legal cause of the property damage to the homeowner. The friend is not liable because proximate cause cannot be established. Although the statute involved appears to be designed to protect persons who contract with construction workers (so the homeowner is within the protected class) and to prevent damage occurring during the performance of construction work (which is the risk protected against), there must be both an actual and a proximate causal connection between the activity of the defendant and the injury to the plaintiff. Here, the friend's decision to park in the homeowner's driveway resulted in injury to the homeowner through intervention of a completely unforeseeable event. Thus, the friend's activities were not the proximate cause of the injury to the homeowner. Therefore, (B) is correct. (C) is incorrect because negligence per se establishes only a presumption of duty and breach of duty. The plaintiff still must establish causation and damages to complete a prima facie case for negligence, and causation cannot be established in this case. (A) is not the best answer because a member of a protected class may not assume the risk that the statute was intended to prevent. (D) is wrong because the "but for" test establishes actual cause only. Proximate cause is not established because the chain of events was unforeseeable.

(Evidence) 1. A plaintiff sued a chimney sweeping company for personal injury and property damages resulting from an explosion in her chimney the evening after the company had cleaned it. The explosion, which occurred when the plaintiff lit a fire in the fireplace, caused minor damage to the chimney, roof, and to the plaintiff, who was hit by falling bricks. As evidence that she assumed the risk of injury, the company offers to have its foreman testify that he had told the plaintiff not to use the fireplace for 24 hours to allow certain chemicals to evaporate. Is the foreman's proposed testimony hearsay? (A) No, because the declarant is testifying as a witness at the hearing. (B) No, because the statement is not offered for its truth. (C) Yes, but it should be admitted as part of the res gestae. (D) Yes, but it should be admitted under the present state of mind exception to the hearsay rule.

ANSWER: (B) No, because the statement is not offered for its truth. The evidence is not hearsay because the statement is not offered for its truth; the statement is offered to show its effect on the plaintiff. Hearsay is a statement, other than one made by the declarant while testifying at the trial or hearing, offered in evidence to prove the truth of the matter asserted. [Fed. R. Evid. 801(c)] If a statement is hearsay, and no exception to the hearsay rule is applicable, the evidence must be excluded upon appropriate objection to its admission. [Fed. R. Evid. 802] A statement that would be inadmissible hearsay to prove the truth of the statement may be admitted to show the statement's effect on the hearer or reader. Thus, in a negligence case, where knowledge of a danger is at issue, a statement of warning is admissible for the limited purpose of showing knowledge or notice on the part of a listener. Here, the defense of assumption of the risk has been raised. Whether the plaintiff knew of the danger involved in lighting a fire within 24 hours of the chimney cleaning is an issue. Consequently, the statement of the foreman is admissible to show that the plaintiff had knowledge of the possible danger. The statement is not hearsay because it is not offered to prove that it was in fact dangerous for the plaintiff to light a fire. (A) incorrectly states that the reason the statement is not hearsay is that the declarant is testifying as a witness. The fact that the declarant is now testifying does not alter the hearsay nature of a statement. Any out-of-court statement offered for its truth is hearsay in most jurisdictions (the Federal Rules have a few specific statements characterized as nonhearsay) regardless of whether the declarant is testifying. The reason hearsay is excluded is that there is no opportunity for cross-examination at the time the statement was made. The key in this case is not that the declarant is testifying, but that the statement is not being offered for its truth. (C) characterizes the testimony as hearsay, which is incorrect because it is not being offered for its truth. Even if this testimony were hearsay, it is incorrect to state that it is "part of the res gestae." Formerly, a wide class of declarations was loosely categorized under the label of res gestae exceptions to the hearsay rule. This group included the following exceptions: (i) present state of mind, (ii) excited utterances, (iii) present sense impressions, and (iv) declarations of physical condition. The testimony of the foreman would not come within any of these exceptions. (D) incorrectly characterizes the testimony as hearsay. In addition, this statement, even if hearsay, would not come within the present state of mind exception. A statement of a declarant's then-existing state of mind is admissible when the declarant's state of mind is directly in issue and material to the controversy, or as a basis for a circumstantial inference that a particular declaration of intent was carried out. The declarant here is the foreman. There is no indication that his state of mind is at all relevant to this litigation, nor is the statement offered indicative of any particular intent on the part of the foreman. Thus, the present state of mind exception is inapplicable.

(Criminal Procedure) 35. After being arrested on suspicion of murder, a suspect was taken to the police station and informed of his constitutional rights as required by the Miranda decision. He immediately requested that a lawyer be provided because he had no money to hire one. The arresting officer said that he would get the suspect's lawyer after he was booked, and the officer proceeded to book him. During the booking search, the suspect said to the arresting officer, "I only killed the bastard because he made a pass at me." If the suspect attempts to prevent introduction of the statement made by him to the officer during booking, will he most likely succeed?

ANSWER: (B) No, because the statement was not the result of a custodial interrogation.

(Torts) 10. The class president invited his class to his home to celebrate homecoming. When the sun began to set, a student built a bonfire in the backyard. The student continued to feed the flames until the bonfire was quite large. Suddenly, a gust of wind blew the flames to a neighboring property, igniting the neighbor's shed. If the neighbor sues the student for the damage to his shed on a theory of negligence, under which of the following arguments, if sustained by the facts, would the student most likely be able to avoid liability? (A) The lighting of bonfires on homecoming is an accepted custom in the community. (B) The bonfire was positioned by the student in the center of the backyard to avoid harming neighboring property. (C) The fire that started would have burned itself out but for the fact that the neighbor's shed was built out of substandard, highly flammable material. (D) The student was a guest on the class president's property and entitled to the same restricted scope of liability as the property owner.

ANSWER: (B) The bonfire was positioned by the student in the center of the backyard to avoid harming neighboring property. The student probably will be able to avoid liability if he can establish that he built the bonfire to avoid harming neighboring property. The element of breach of duty in a negligence case requires a showing that the defendant acted unreasonably, which is a question for the trier of fact. Assuming that the student owed a duty to avoid harm to the neighbor's shed, the student will be able to argue that he did not breach that duty because he positioned the bonfire to avoid damaging the neighbor's shed.

4. The plaintiff was exiting from a parking garage owned and operated by the city when he discovered that the exit ramp was blocked by construction barricades and a pile of broken-up concrete. No workers or detour signs were around and the plaintiff was in a hurry, so he backed up and drove down an entrance ramp that was clearly marked as such. As he came around a corner, his car was broadsided by a pickup truck. The plaintiff was seriously injured in the collision. A statute in the jurisdiction requires drivers to obey all traffic directional markings in both public and private parking lots and garages. The jurisdiction retains governmental immunity for municipalities. If the plaintiff brings a lawsuit against the city to recover for his injuries, which of the following facts will be LEAST helpful in the city's defense?

ANSWER: (B) The construction workers responsible for blocking off the exit ramp were employees of an independent contractor rather than the city. The fact least helpful to the city's defense of the plaintiff's lawsuit is the identity of the workers who blocked the exit ramp. Under vicarious liability rules, a principal will be liable for the tortious acts of an independent contractor if the duty is nondelegable on public policy grounds; included is the duty of a possessor of land to keep its premises safe for its invitees. If the workers were negligent in leaving the ramp blocked without providing another means of exiting, the fact that they were not city employees would not absolve the city of liability; hence, their identity would be of no help to the city's defense. (A) is incorrect because if the plaintiff was aware of an alternate route, he may have been contributorily negligent in exiting down the entrance ramp. A plaintiff's contributory negligence may be established by violation of an applicable statute. However, as with a statutory duty imposed on a defendant, the plaintiff's violation of the statute may be excused if compliance was beyond the plaintiff's control. If no other means of exiting the garage were known to the plaintiff, he may be excused for violating the traffic statute; however, if he knew of an alternative exit, the city will probably be able to establish contributory negligence on his part by his violation of the statute, reducing his potential recovery. (C) is incorrect because whether the city collects fees and makes a profit in operation of the garage will be considered by the court in determining whether the jurisdiction's governmental immunity applies. Where municipal immunity still exists, courts have limited its scope by differentiating between "governmental" and "proprietary" functions of the municipality. If the municipality is performing a function that might as well have been provided by a private corporation, the function may be construed as a proprietary one and no immunity will attach. The inference that a function is proprietary will be strengthened where the city collects revenues by virtue of providing the service. Hence, the fact that the city is not collecting revenues or making a profit in operating the garage will make it less likely that the function will be deemed to be proprietary and more likely that it will be deemed to be governmental and thus immune; in other words, it will be more helpful rather than less helpful in the city's defense. (D) is incorrect because the pickup truck driver's conduct under these circumstances would be deemed a superseding force that breaks the causal connection between any negligence on the part of the city and the plaintiff's injury. Assuming that the city workers were negligent, the fact that an independent intervening force caused the injury generally would not cut off the city's liability, because its negligence created a foreseeable risk of that harm occurring. However, where this foreseeable harm is caused by an unforeseeable crime or intentional tort of a third party, most courts would not hold the city liable, treating the crime or tort as a superseding force. Here, while blocking the exit ramp created a foreseeable risk that someone might collide with the plaintiff, it was not foreseeable that his enemy would take that opportunity to commit an intentional tort against him. Because the pickup truck driver's conduct was unforeseeable under the circumstances in choice (D), the city would be relieved of liability for any negligence in blocking the ramp.

(Evidence) 3. A pedestrian sued a local bar for injuries he suffered when he was struck by a car driven by a bar patron that had run a red light. He claimed that the patron was permitted to drink too much liquor at the bar before leaving. At trial, the pedestrian called a witness to the stand. The witness testified that she and a friend had visited the bar on the night in question. The witness seeks to testify that she remarked about the patron to her friend, "Look at that guy. He's so drunk he can't even stand up." Is the witness's testimony concerning her remark to her friend admissible? (A) Yes, as a prior consistent statement. (B) Yes, as a present sense impression. (C) Yes, as an excited utterance. (D) No, because it is hearsay not within any exception.

ANSWER: (B) Yes, as a present sense impression. The witness's remark is admissible under the present sense impression exception to the hearsay rule. [Fed. R. Evid. 803(1)] Under this exception, a declarant's statement describing or explaining an event or condition is admissible if it was made while the declarant was perceiving the event or condition or immediately thereafter. Here, the witness made the observation of the intoxicated patron while she was observing him. (A) is wrong. First, there is no indication that the witness's prior statement at the bar is consistent with anything the witness has said on the stand (i.e., she has not testified that the patron was drunk; she has only testified about her prior statement at the bar). Furthermore, even if her statement at the tavern is consistent with her current testimony, it is not admissible on this basis. A witness's prior consistent statement is admissible when it is offered to (i) rebut a charge that the witness is lying or exaggerating because of some motive (e.g., bias), provided the prior consistent statement was made before the onset of the alleged motive; or (ii) rehabilitate a witness whose credibility has been impeached on some other non-character ground (e.g., a sensory deficiency). Here the witness has not been impeached, so her prior consistent statement cannot be introduced. (C) is incorrect because the declarant's statement would be unlikely to qualify as an excited utterance. For her statement to be admissible as an excited utterance, it would have to have been made under the stress of excitement produced by a startling event. The facts do not indicate that the declarant's observation of the patron was startling or stressful; just that she noticed that he was drunk. (D) is wrong because the statement comes within the present sense impression exception to the hearsay rule.

(Evidence) 8. The plaintiff, an electrical contractor, sued the defendant homeowner for refusal to pay for extensive wiring repairs performed on his home by the plaintiff's employee. The plaintiff called the employee to the stand. The employee, under oath, testified that he did not perform any work at the defendant's home. The employee also denied writing a letter to a friend telling the friend that the employee was going to do electrical work on the home. Without releasing the employee as a witness, the plaintiff offers into evidence the letter written by the employee to his friend. If the employee's letter to his friend is properly authenticated, should the court admit the letter? (A) Yes, for impeachment purposes only. (B) Yes, as both substantive and impeachment evidence. (C) No, because a party may not impeach his own witness. (D) No, because it is inadmissible hearsay.

ANSWER: (B) Yes, as both substantive and impeachment evidence. The letter is admissible as substantive evidence as well as for impeachment purposes. For the purpose of impeaching the credibility of a witness, a party may show that the witness has, on another occasion, made statements that are inconsistent with some material part of his present testimony. This may be done by first questioning the witness as to the prior inconsistent statement that he has made. If the witness denies having made the statement or fails to remember it, the making of the statement may be proved by extrinsic evidence. A proper foundation must be laid by giving the witness an opportunity to explain or deny the statement, and it must be relevant to some issue in the case. Here, the plaintiff's employee has denied that he wrote the letter to his friend. The plaintiff can then impeach the employee by offering the letter into evidence. Because the employee has not been released as a witness, he will have an opportunity to explain or deny the statement, and it is relevant to whether any work was done at the home. Because prior inconsistent statements are generally hearsay, they often are admissible only for purposes of impeachment. The Federal Rules do categorize a testifying witness's prior inconsistent statement as nonhearsay if it was made under penalty of perjury at a prior trial, hearing, or proceeding, or in a deposition. Here, of course, the employee's letter to the friend was not made under oath, so it is hearsay. However, it is still admissible as substantive evidence because it falls within an exception to the hearsay rule. Under Rule 803(3), a statement of a declarant's then-existing state of mind is admissible as a basis for a circumstantial inference that the declarant acted in accordance with his state of mind. [See also Mutual Life Insurance Co. v. Hillmon (1892)] The employee's statement that he was going to do electrical work on the home is admissible as circumstantial evidence tending to show that he followed through with his plans and did the electrical work, which is what the statement is being offered to establish. In this case, therefore, the letter should be admissible as both substantive and impeachment evidence, making (B) correct and (A) incorrect. (C) is incorrect because the Federal Rules provide that the credibility of a witness may be attacked by any party, including the party calling him. [Fed. R. Evid. 607] (D) is incorrect. The letter is hearsay because it is being offered to prove the truth of the matter asserted—that the employee was going to do electrical work on the home—as a basis for inferring that the employee did do the work. Additionally, it is not categorized as nonhearsay under the Federal Rules because it was not made under oath. However, as discussed above, it falls within the "present state of mind" exception to the hearsay rule.

(Evidence) 6. The complainant was robbed by a man wielding an unusual knife with a pearl-studded handle. The defendant was arrested and charged with armed robbery of the complainant. At trial the prosecution calls a witness to testify that, three days after the robbery of the complainant, she was robbed by the defendant with a knife that had a pearl-studded handle. Should the court rule that the witness's testimony is admissible? (A) Yes, as showing habit. (B) Yes, as establishing an identifying circumstance. (C) No, because it is improper character evidence. (D) No, because its probative value is substantially outweighed by the danger of unfair prejudice.

ANSWER: (B) Yes, as establishing an identifying circumstance. (MIMIC) The court should admit the evidence for purposes of establishing identity. Other crimes and wrongs are generally not admissible to prove that a person acted in conformity with his bad character. However, they are sometimes admissible for other purposes, such as to establish the identity of the accused. Other crimes are admissible on identity when they are committed in a unique way that shows what amounts to a "signature" of the perpetrator. Given the highly unusual weapon in this case, the court should hold that the evidence is admissible to show that the defendant was the perpetrator. Because the evidence is relevant for a purpose other than conformity, (C) is wrong. Theoretically, even signature crimes can be excluded if the judge determines that the probative value is substantially outweighed by the danger of unfair prejudice. However, a crime qualifying as a signature crime is highly probative and would rarely be excluded under that theory. Therefore, (D) is wrong. (A) is wrong. One other crime could not establish habit.

(Evidence) 2. In a wrongful death action, the plaintiff claimed that the defendant intentionally caused the death of the plaintiff's husband, who was the defendant's co-worker. At trial, the plaintiff's attorney called another co-worker to the stand as a witness. The defendant's attorney did not object to the witness's testimony that there had been "bad blood" at work between the deceased and the defendant. However, the attorney objected to one line of questioning, but he was overruled by the judge. The line of questioning was as follows: Attorney: "Now, you've told us how the deceased came back to the shop after he had obviously been hit hard on the jaw. What did he say at that time?" Witness: "He said that the defendant did it, and just then I noticed that the defendant was in the shop too." Attorney: "What did the defendant do?" Witness: "He just smirked and started laughing." Was the judge correct in overruling the objection to the admission of this portion of the witness's testimony? (A) Yes, because it is a statement against interest by the defendant. (B) Yes, because it is an adoptive statement by a party-opponent. (C) No, because the introduction of the deceased's out-of-court statement would violate the defendant's right to confront witnesses. (D) No, because the deceased's statement that the defendant did it is hearsay, and cannot qualify as a dying declaration.

ANSWER: (B) Yes, because it is an adoptive statement by a party-opponent. The judge was correct in overruling the objection because it was an adoptive statement by the defendant. A statement by an opposing party is not hearsay. A party may expressly or impliedly adopt someone else's statement as his own. One type of adoptive statement occurs when a party hears an accusation and remains silent when a reasonable person would have protested if the accusation were untrue. Under those circumstances, both the accusation and the lack of protest can be introduced by the party-opponent if relevant. Here, the deceased accused the defendant of hitting him in the jaw and the defendant failed to deny the accusation. Therefore, (B) is correct. Regarding (D), it is true that the deceased's statement would probably not qualify as a dying declaration. A declaration made by the now-unavailable declarant, while believing his death was imminent, that concerns the cause or circumstances of what he believed to be his impending death is admissible. The question does not indicate that the deceased believed that his death was imminent at the time he made his statement; therefore, the statement would not qualify as a dying declaration. However, (D) is incorrect because the testimony would be admissible as an adoptive statement by an opposing party. (A) is wrong because the "statement against interest" hearsay exception can only be used when the declarant is unavailable to testify. Here, the defendant is available. Furthermore, laughing and smirking would not qualify as a statement against interest; the defendant didn't make any affirmative statement. The only reason his reaction is admissible is because it qualifies as an adoptive statement. (C) is wrong because the Confrontation Clause does not apply to civil cases.

(Evidence) 22. A plaintiff sued his neighbor over a 10-foot-high stockade fence that the neighbor was building adjacent to the plaintiff's backyard. The local zoning ordinance permitted a fence of this height unless it was a "spite fence," defined as a fence erected solely for the purpose of interfering with neighboring landowners' use and enjoyment of their property. The plaintiff alleged that the neighbor was building the fence to block sunlight to the garden that the plaintiff had planted. The neighbor denied that she was building the fence for that purpose. The plaintiff wishes to introduce evidence that the neighbor had sprayed herbicide towards the garden previously. Should the judge permit the plaintiff's testimony? A Yes, because the neighbor's character is at issue in the case. B Yes, because it pertains to the neighbor's motivation in building the fence. C No, because the plaintiff's testimony is evidence of specific conduct, which is not admissible in this case because the neighbor's character is not in issue. D No, because character evidence generally is not admissible in a civil case.

ANSWER: (B) Yes, because it pertains to the neighbor's motivation in building the fence. The judge should permit the plaintiff's testimony because evidence of specific acts of misconduct is admissible to show motive. Under Federal Rule 404(b), evidence of other acts may be admissible in a criminal or civil case if they are relevant to some issue other than character, such as motive. Here, whether the neighbor was motivated by an improper purpose in building the fence is the key issue in the lawsuit by the plaintiff. The neighbor's prior misconduct in spraying herbicide toward the plaintiff's garden is circumstantial evidence that her hostility toward the garden motivated her to build the fence. (A) is wrong because even though the neighbor's motivation and intent are at issue in the case, her character is not. In the absence of character being directly in issue in the case, evidence of character to prove the conduct of a person in the litigated event is not admissible. (C) and (D) are wrong even though they correctly state general rules: evidence of specific acts of misconduct is generally inadmissible, and character evidence is generally inadmissible in a civil case. However, when the specific acts are being offered for a purpose other than to show bad character or conduct in conformity to character, they are admissible in both criminal and civil cases.

(Torts) 13. A 12-year-old snowboarder at a ski resort lost control while he was going down a slope and collided with a middle-aged skier who was ahead of him. The collision caused the skier to fall hard and sustain multiple fractures of her leg. The injuries required surgery and several months of rehabilitation before the skier could walk again. If the skier prevails in a personal injury action against the snowboarder, what is the most likely reason? (A) The skier had not signed a ski resort waiver assuming the risk of any injuries suffered while skiing. (B) The severity of the skier's injuries dictated that an adult standard of care be applied to the snowboarder's activity. (C) A 12-year-old of like education, intelligence, and experience would have avoided the accident in the exercise of reasonable care. (D) The snowboarder had used a fake pass to gain entrance to the ski resort.

ANSWER: (C) A 12-year-old of like education, intelligence, and experience would have avoided the accident in the exercise of reasonable care. The skier will prevail if a 12-year-old snowboarder acting reasonably would have been able to avoid the accident. A child is required to conform to the standard of care of a child of like age, education, intelligence, and experience in the usual case. This permits a subjective evaluation of whether that particular defendant acted reasonably under the circumstances. If the snowboarder acted unreasonably under this standard, the skier will recover. (A) is incorrect because even if a waiver had been signed by the skier, it would allow only the ski resort to raise the defense of express assumption of risk if sued; the snowboarder could not rely on a waiver to which he was not a party. Thus, the skier's failure to sign a waiver is irrelevant. (B) is wrong because the severity of the skier's injuries do not dictate whether an adult standard of care applies. Courts will require children to conform to an adult standard of care when they are engaged in potentially dangerous activities that normally only adults engage in, such as driving an automobile. Snowboarding would not be considered that type of activity, so an adult standard of care would not apply regardless of the severity of the skier's injuries. (D) is incorrect because the fact that the snowboarder had used a fake pass to gain entrance to the resort does not indicate whether he was complying with the applicable standard of care when he struck the skier, which is the critical question here.

(Torts) 14. After enjoying a wonderful meal in a restaurant, a diner went into the kitchen through a door marked "employees only" to personally compliment the chef. However, before he could get the attention of the chef, he slipped on a puddle of bright yellow grease that had congealed on the floor by the stove. He fell, hitting his head and sustaining a severe head injury. If the diner sues the restaurant for damages in a jurisdiction following the traditional rules for landowners and possessors of land, is he likely to recover? (A) Yes, because the restaurant is a place of public accommodation and breached its duty of care owed to its patrons. (B) Yes, because a restaurant employee could have discovered the dangerous condition of the floor by making reasonable inspections. (C) No, because patrons were not allowed in the kitchen. (D) No, because the puddle of grease was visible on the floor.

ANSWER: (C) No, because patrons were not allowed in the kitchen. The diner is not likely to recover because the restaurant did not permit patrons in the kitchen. In jurisdictions following the traditional rules for landowners and possessors of land, the duty owed by an owner or occupier of land to those entering the land depends on whether the entrant is characterized as a trespasser, licensee, or invitee. Here, the diner was an invitee of the restaurant while he was dining at the restaurant. However, a person loses his status as an invitee if he exceeds the scope of the invitation—if he goes into a portion of the premises where his invitation cannot reasonably be said to extend. Here, the diner lost the status of an invitee when he entered the kitchen; he became, at best, a licensee, perhaps even a trespasser, because patrons were not permitted in the kitchen. While a landowner owes no duty to an undiscovered trespasser, he owes a discovered trespasser the duty to warn of or make safe artificial conditions known to the landowner that involve a risk of death or serious bodily harm and that the trespasser is unlikely to discover. For a licensee, the duty extends to all dangerous conditions that create an unreasonable risk of any harm to the licensee. Had the diner been an invitee, he could have argued that under the duty owed to invitees to make reasonable inspections, a puddle of grease that had time to congeal should have been discovered and cleaned up, or at least been the subject of a warning. Here, however, there are no facts to suggest that anyone in the kitchen knew of the diner's presence or the puddle of grease, even though it had congealed; hence, the facts do not establish a breach of the duty to warn discovered trespassers or licensees of dangerous conditions. (A) is incorrect. While places of public accommodation have an affirmative duty to use reasonable care to aid or assist their patrons, that duty rule does not alter the duty rules pertaining to the condition of the land, which are based on the status of the person on the premises (as discussed above). (B) is incorrect because the failure to exercise reasonable care to discover a dangerous condition breaches a duty owed only to invitees, as discussed above. However, the diner was no longer an invitee when he entered the kitchen area. He was at most a licensee, for whom the duty to make reasonable inspections does not apply. (D) is incorrect. While a duty to warn does not exist where the dangerous condition is so obvious that the invitee should reasonably have been aware of it, just the fact that the grease was visible on the floor does not establish this. "Obviousness" is determined by all of the surrounding circumstances; e.g., one whose attention may have been directed elsewhere may recover even though the condition was visible. In any case, even if the puddle were not visible, the diner would not be able to recover because he did not have invitee status when he entered the kitchen.

9. The defendant was being sued by the plaintiff for driving his car negligently and injuring the plaintiff. The defendant called as a witness his brother, who was in the passenger seat of the defendant's car when the accident occurred. His brother testified that the defendant was driving safely and well below the posted 55-mile-perhour speed limit. Shortly before the trial began, the brother's secretary telephoned the plaintiff and told him that the brother has been understating his income to the government for years. The Internal Revenue Service has never charged the brother with tax evasion, but the secretary's information was accurate. On cross-examination, the plaintiff's attorney asked the brother, "Have you ever cheated on your tax returns?" The defendant's attorney objects. Should the objection be sustained?

ANSWER: (C) No, because the brother's tax evasion shows a tendency to lie. The objection should be overruled because the question is a proper means of impeaching the brother's character for truthfulness through specific instances of misconduct. Under Federal Rule 608(b), subject to the discretion of the trial judge, a witness may be interrogated on cross-examination with respect to any specific act that may impeach his character and show him to be unworthy of belief, as long as the act is probative of truthfulness (i.e., an act of deceit or lying). A conviction of a crime is not necessary under this rule. Cheating on one's taxes is lying, so this would be a specific act of misconduct reflecting on the brother's character for truthfulness. (A) is incorrect. Any matter that tends to prove or disprove the credibility of a witness is relevant because it affects the weight that the trier of fact should give to his testimony. (B) is incorrect because the brother is not being impeached by collateral extrinsic evidence (which is not permitted by Rule 608(b)); he is only being interrogated on cross-examination. (D) is incorrect because it states a requirement for impeachment by a prior conviction under Federal Rule 609, rather than by prior bad acts under Rule 608(b), which does not require that the conduct constitute a felony. Rule 609 is inapplicable because it requires a criminal conviction, and the brother has never even been charged with tax evasion.

8. A pâté manufacturer entered into a written agreement with a gourmet food store. The manufacturer agreed to sell "all" its "output of liver pâté " to the store and the store agreed to sell the manufacturer's pâté "exclusively." The agreement between the store and the manufacturer also contained the statement, "either party may cancel this contract after two months on giving reasonable notice to the other party." After the manufacturer filled the store's orders for six months, the manufacturer determined that it was becoming too costly to operate and maintain the special oven used to roast the pâté loaves, and that it would be difficult and expensive to find a suitable substitute. The manufacturer, therefore, notified the store that it was getting out of the pâté business, and explained why, and that it would stop shipping pâté to the store after 60 days. The store sued the manufacturer, demanding that the manufacturer continue to ship pâté to the store or pay monetary damages. Will the store prevail?

ANSWER: (C) No, because the expense of fixing the oven provides a good faith reason for stopping production. The extreme difficulty and expense of repairing or replacing the oven constitutes a good faith reason for the manufacturer to cancel the contract, as it is entitled to do under the contract on reasonable notice. Thus, the store will not prevail. Reservation of an unqualified right to cancel or withdraw from a contract at any time may amount to an illusory promise. However, the promise is not illusory, and there is a valid consideration, if the right to cancel is restricted in any way. Here, the right of either party to cancel is restricted because it must be preceded by reasonable notice to the other party, which was provided. Therefore, the promises are not illusory, and both parties are bound. The manufacturer finds itself confronted with circumstances that present extreme and unreasonable difficulty and expense in complying with its contractual duties of supplying pâté. While this additional test might not be sufficient to discharge the manufacturer's duties on grounds of impracticability, it is clearly grounds for supplying the manufacturer with a good faith reason for canceling the contract. (A) is incorrect because the manufacturer is entitled to cancel the contract for a good faith reason on giving reasonable notice. It is not necessary to show impossibility. (B) is incorrect because the fact that the parties agreed to a cancellation provision indicates that the manufacturer did not assume all risks of increased expenses in making pâté. The unreasonable amount of expense and difficulty involved with the maintenance and/or replacement of the oven supports the manufacturer's good faith decision to cancel the contract. (D) is incorrect because, while some courts have found promises to be illusory if the contract provided for an unqualified right to cancel the contract at any time, here the right to cancel is restricted by requirements of good faith reasons for cancellation as well as giving reasonable notice.

3. An electrician submitted a bid for electrical work in response to a newspaper ad placed by a general contractor, who was bidding on the renovation of an office building. The electrician's bid was the lowest and the general contractor used it to form his bid submitted to the owner of the building. The general contractor was awarded the contract, but later decided to hire another party, who had initially submitted a higher bid, to perform the electrical work. If the electrician sues for damages, will he prevail?

ANSWER: (C) No, because the general contractor never communicated an acceptance of the electrician's offer. The general contractor did not accept the electrician's bid even though he used it to prepare his bid. The advertisement constituted an invitation for subcontractors to make offers. The electrician's bid constituted an offer. The general rule is that acceptance of an offer must be communicated to the offeror, and here the general contractor did not communicate any acceptance to the electrician. Although statutes in some cases may create an exception to the general rule by making acceptance of a subcontractor bid automatic on the general contractor's being awarded the contract, no such exception is indicated by these facts. (A) is incorrect because the facts do not reveal any basis for an inference that the general contractor would use the lowest bid. Typically, a contractor will consider other factors as well, such as reputation or past performance, in deciding which bid to accept. (B) is incorrect even though the electrician's bid could be treated as an offer for an option contract based on promissory estoppel (detrimental reliance) principles. Here, the acceptance of the option contract to keep the bid open occurred when the general contractor relied on the bid to prepare his own bid. [See Restatement (Second) of Contracts §87] The award of the general contract had no effect on the option contract and did not create an acceptance of the offer for the electrical contract (as discussed above). (D) is incorrect, although it is factually a true statement. The general contractor's advertisement was an invitation for offers, not an offer in itself. The offer on these facts was made by the electrician, and the reason no enforceable contract exists is that the general contractor never accepted the electrician's offer.

(Torts) 16. A motorist driving home one night on a desolate two-lane road stopped when he saw a person lying on the road next to a bicycle. The cyclist had slipped and fallen off his bicycle, and was knocked unconscious when he hit his head on the pavement. Not wishing to get involved and seeing that no one else was around, the motorist got back into his car and drove away without making any effort to help the cyclist, even though he had a cell phone with which he could have summoned aid. The cyclist remained lying in the same place and was later struck by another car. If the cyclist brings suit against the motorist for injuries suffered when he was struck by the other car, will the cyclist prevail? (A) Yes, because a reasonable person under the same circumstances would have come to the cyclist's aid. (B) Yes, because by stopping and examining the cyclist, the motorist, as a matter of law, assumed a duty to aid him. (C) No, because the motorist was not responsible for causing the cyclist to be lying by the side of the road. (D) No, because the cyclist's injuries were caused by the negligence of another driver

ANSWER: (C) No, because the motorist was not responsible for causing the cyclist to be lying by the side of the road. The cyclist will not prevail because the motorist was not responsible for putting the cyclist in a position of danger. While one whose conduct places another in a position of peril has a duty to assist the person, there is no general affirmative duty to rescue, except by a professional rescuer. Also, once any person decides to assist in a rescue, he must act as a reasonable person in an emergency situation. Here, the motorist was under no duty to assist the cyclist, and his actions when he stopped and got out of his car did nothing to make the cyclist's situation worse (such as causing others not to stop because they believed the cyclist was receiving assistance). Hence, the motorist will not be liable. (A) is wrong because even though it appears that the motorist acted unreasonably, he is not liable because he had no duty to rescue. (B) is similarly wrong; his stopping did not by itself create a duty to rescue. (D) is wrong because the fact that another driver negligently injured the cyclist would not preclude the motorist from also being liable if he had been responsible for putting the cyclist in a position of danger.

(Evidence) 4. The plaintiff sued the defendant for bodily injuries suffered by the plaintiff as a result of a collision between the cars they were driving. The accident occurred on a rainy day, when the defendant's car skidded across the center line and struck the plaintiff's car. A principal issue is whether the defendant was driving too fast for the wet conditions. At trial, the plaintiff calls a witness who is prepared to testify that he has lived next door to the defendant for 15 years, and that the defendant is notorious in the neighborhood for driving his car at excessive rates of speed. The defendant's attorney objects. May the witness's testimony be allowed? (A) Yes, because the defendant's character as a careless driver is in issue. (B) Yes, because character may be proven by reputation evidence. (C) No, because the testimony is improper character evidence. (D) No, because the testimony is hearsay, not within any exception

ANSWER: (C) No, because the testimony is improper character evidence. The testimony of the witness is inadmissible character evidence; i.e., it describes the defendant's general behavior patterns. Evidence of character to prove the conduct of a person in the litigated event is generally not admissible in a civil case. Here, the plaintiff is trying to employ the circumstantial use of prior behavior patterns to draw the inference that the defendant drove at an excessive rate of speed at the time of the incident here at issue. Such a use of character evidence is not permitted. (A) is incorrect because character is not in issue. An exception to the general prohibition of character evidence is that, when proof of a person's character, as a matter of substantive law, is an essential element of a claim or defense in the case, character evidence is admissible. The defendant's character as a driver is not in issue; rather, his actions at a specific time and place are in issue. Thus, this exception does not apply to these facts. Regarding (B), it is true that character may be established by reputation evidence under the Federal Rules. However, as explained above, this is not a proper case for the use of character evidence. Therefore, (B) is incorrect. (D) is also incorrect. While it is true that testimony as to a person's reputation in the community may in some sense be considered hearsay (i.e., such testimony reflects what people are saying about a person), reputation testimony is a permissible (and in fact, the most common) means of showing character. Thus, (D) does not present a basis for refusing to allow the testimony.

(Torts) 12. A housecleaning agency was given a key to a customer's house so that the agency could have its employees clean while the homeowner was away. After a maid sent by the agency had finished and left the homeowner's house, she went back because she had forgotten her cigarettes. She neglected to lock the door when she left the second time because she was already late for the next job. When the homeowner returned after a few days away, she discovered that her house had been ransacked and several items of jewelry stolen. The front door was open, and there were no signs of forced entry. If the homeowner brings an action against the agency that employed the maid, what is the likely result? A. She will not prevail, because she is limited to claims for breach of contract based on her agreement with the agency. B. She will not prevail, because the act of the burglar was an independent superseding cause of the homeowner's loss. C. She will prevail, because the maid's failure to lock the door created the risk that someone might enter and take the homeowner's valuables. D. She will prevail, because when the maid returned after having completed her work, she was technically a trespasser, making the agency vicariously liable for any damage she caused to the premises.

ANSWER: (C) She will prevail, because the maid's failure to lock the door created the risk that someone might enter and take the homeowner's valuables. The homeowner will prevail because the maid's negligence increased the risk of criminal conduct by a third party. Criminal acts and intentional torts of third persons are foreseeable independent intervening forces if the defendant's negligence created a foreseeable risk that they would occur. Here, the maid's failure to lock the door was negligent because it created a risk of burglary; hence, the burglary does not cut off the agency's liability for the maid's negligence. As the maid's employer, the agency is vicariously liable under respondeat superior. (A) is wrong because there is nothing in the facts to indicate that the homeowner waived her right to bring tort claims against the agency; having a contractual relationship with a party does not automatically preclude bringing a tort action against the party. (B) is wrong because the burglary was not a superseding cause of the loss; it was within the increased risk caused by the maid's negligence. (D) is wrong because she reentered to retrieve a personal item that she had brought with her when she went to the job; her return just to get the item was within the scope of her employment and would not make her a trespasser.

(Torts) 18. A state child safety statute required children under eight years of age to be in a government-approved car seat when riding in a motor vehicle. A father was driving to a ballgame with his seven-year-old child, who was buckled in the back seat with a regular seat belt. The father did not notice when the child unbuckled himself and started climbing into the front seat. The child grabbed the steering wheel "to help daddy steer," causing the car to swerve into the other lane and collide with another motorist's car. The motorist was seriously injured from the collision; the father and his child were unhurt. The motorist sued the father to recover damages for her injuries. At trial, the motorist presented evidence of the statute, her injuries, and the facts stated above. At the conclusion of the proofs, both parties moved for a directed verdict. How should the trial judge proceed? (A) Grant the motorist's motion, because the father's violation of the statute constituted negligence per se. (B) Grant the father's motion, because the motorist offered no evidence that the statute was intended to prevent the harm that occurred. (C) Deny both motions and submit the case to the jury, because the jury could find that the father is liable for his child's negligent conduct. (D) Deny both motions and submit the case to the jury, because the jury could find that the father breached his duty of care owed to the motorist.

ANSWER: (D) Deny both motions and submit the case to the jury, because the jury could find that the father breached his duty of care owed to the motorist. The court should deny both motions and submit the case to the jury, because the jury could find that the father was negligent in not preventing his child from grabbing the steering wheel. Under ordinary negligence principles, the father owed a duty to other motorists to maintain control of his vehicle. The jury could find that the father breached this duty of care by not noticing when his child unbuckled the seat belt and started climbing into the front seat, and not preventing the child from grabbing the steering wheel. Hence, the case should go to the jury for a determination of whether the father was negligent. (A) is incorrect because the facts do not establish that the statutory standard of care is applicable to these facts. The precise standard of care in a common law negligence case may be established by proving that a statute providing for a criminal penalty applies to the particular case. If that is done, the statute's more specific duty will replace the more general common law duty of care. Violation of the statute establishes negligence per se—a conclusive presumption of duty and breach of duty; the plaintiff must then establish causation and damages to complete the prima facie case of negligence. To prove that the statutory standard applies, the plaintiff must show that (i) she is in the class intended to be protected by the statute, and (ii) the statute was designed to prevent the type of harm that occurred. Here, the statute likely was intended primarily to protect children from injuries caused by not being properly restrained in a vehicle involved in a collision. The motorist has presented no evidence that the statute was intended to protect her from the harm that she suffered. (B) is incorrect because even though the motorist did not establish that the statute applies to her claim, she has presented facts sufficient to allow the jury to find that the father breached the common law duty of care that he owed to other motorists. (C) is incorrect because the father is not vicariously liable for his child's negligent conduct at common law. Any liability of the father in this case would arise from his own potential negligence in failing to control his child while driving.

(Torts) 11. A young boy was hospitalized after he accidentally swallowed a coin while playing. The boy recovered, but his mother began suffering feelings of anxiety, worrying that her child might be injured again. The mother went to a movie to relax and ordered some popcorn. While handling her change, the concession stand attendant carelessly dropped a quarter into the popcorn, which the mother did not notice until she bit the quarter. She did not swallow the quarter, nor did she bite on it hard enough to injure her teeth or gums, but the feel of the coin brought back all her anxieties about the boy's injury. The mother sued the movie theater for her emotional distress. If the jury finds that a reasonable person would not have suffered similar distress under the circumstances, is the mother likely to prevail? (A) Yes, because the concession stand attendant was negligent. (B) Yes, because she suffered emotional distress. (C) No, because a reasonable person would not have suffered similar distress under the circumstances. (D) No, because she suffered no actionable harm.

ANSWER: (D) No, because she suffered no actionable harm. The mother will lose because she will not be able to establish a prima facie case for negligent infliction of emotional distress. An action for negligent infliction of emotional distress requires the plaintiff to show that the defendant's conduct created a foreseeable risk of physical injury to the plaintiff, such as by threat of physical impact, and, in the usual case, that the emotional distress caused by this conduct also manifests itself in physical symptoms. Here, the first element is established because the negligence of the theater employee created a foreseeable risk that the mother would be injured by biting down on the coin, but the second element is not established because the mother's distress does not amount to an actionable physical injury, and she suffered no other injury by biting the coin. While many courts do not require physical symptoms if the defendant's negligence creates a great likelihood of substantial emotional distress, these facts do not rise to that level. Note that if she had suffered physical injury by biting the coin, she would have a prima facie case against the theater for negligence. She then would have been able to recover for her emotional distress as an additional element of her noneconomic damages.

(Evidence) 7. A defendant was on trial for burglary, and he took the stand in his own defense. On direct examination, the defendant vigorously denied having committed the burglary. Also on direct examination, the defendant stated that his last regular employment was as a bookkeeper for a corporation. On cross-examination, the prosecutor asked the defendant if he had embezzled funds from the corporation. The defendant denied that he had embezzled from the corporation or from anyone else. The prosecutor then wanted to call a police officer to the stand to testify that when she arrested the defendant for embezzlement, the defendant admitted to the officer that he had embezzled money from the corporation. Assuming that the defendant has not yet been tried on the embezzlement charges, may the prosecutor call the officer to the stand? (A) Yes, but only for purposes of impeachment. (B) Yes, both for impeachment of the defendant and as substantive evidence. (C) No, because the defendant has not yet been convicted of embezzlement. (D) No, because the evidence would be extrinsic.

ANSWER: (D) No, because the evidence would be extrinsic. The officer may not testify about the embezzlement because it constitutes impeachment by extrinsic evidence of a specific instance of misconduct. A specific act of misconduct offered to attack the witness's character for truthfulness can be elicited only on cross-examination. If the witness denies the act, the cross-examiner cannot refute the answer by calling other witnesses or producing other evidence. Because the alleged embezzlement is admissible, if at all, only as impeachment evidence, when the defendant denied it the prosecutor could not call the officer to testify. (A) is wrong because extrinsic evidence, such as the officer's testimony, of an instance of misconduct is not admissible. (B) is wrong because when a person is charged with one crime, extrinsic evidence of other crimes or misconduct is inadmissible to establish criminal disposition. Because nothing in the facts indicates that such evidence is being offered to prove something other than disposition (e.g., motive, identity, mistake, intent, & common plan or scheme), the officer's testimony is not admissible as substantive evidence. As discussed above, for impeachment, the prosecutor is limited to inquiry on cross-examination regarding the embezzlement. (C) is wrong because it implies that a witness may not be impeached with a prior act of misconduct unless it resulted in a conviction, and this is not the case. A witness may be impeached on cross-examination with a prior act of misconduct that is probative of truthfulness even if the misconduct did not result in a conviction. Here, the problem is that the prosecutor is attempting to prove the bad act with extrinsic evidence, which is not allowed. Also, even if the defendant had been convicted of the embezzlement, the officer's testimony would not be admitted as proof of the conviction. The fact that a witness has been convicted of a crime usually is proved by eliciting an admission on direct or cross-examination or by the record of conviction. A judge is less likely to allow proof of conviction by testimony because it may be too time-consuming and more prejudicial than other means of proof. And here, the officer's testimony that the defendant admitted to embezzling the money is not sufficient to prove that the defendant was convicted.

(Torts) 21. A patient visited a physician because of a painful foot. The physician told the patient that he could choose several weeks of physical therapy or undergo elective surgery to correct the problem. The physician did not tell the patient that there was a risk of nerve damage if he underwent the surgery, although most physicians proposing this surgery would have done so. The patient chose the surgery and the physician performed the surgery successfully. However, the patient was upset when he learned afterward about the risk of nerve damage, because, had he known of the risks of the surgery, he would have chosen the therapy instead. In an action against the physician for failing to inform the patient of the risks of the surgery, is the patient likely to prevail? (A) Yes, because he would have refused the operation had he known of the risks. (B) Yes, because most physicians proposing this surgery would have informed the patient of the risks. (C) No, unless the jury concludes that a reasonable person would have withheld consent to the operation if informed of the risks. (D) No, because the surgery was successful.

ANSWER: (D) No, because the surgery was successful. The patient most likely will not prevail because the surgery was successful. Physicians owe a duty to patients to provide them with enough information about the risks of a proposed course of treatment or surgical procedure to enable them to give "informed consent" to the treatment. If an undisclosed risk was serious enough that a reasonable person in the patient's position would have withheld consent to the treatment, the physician has breached this duty. However, breach of duty is only one element of a cause of action for negligence. The plaintiff must also establish actual and proximate cause and some damage to the plaintiff's person or property. Damage means actual harm or injury. Unlike for some intentional torts, damage will not be presumed in a negligence action, and nominal damages are not available. While a complete absence of consent to a medical or surgical procedure may in some cases constitute battery, which does not require damage as an element, a nondisclosure of the risks of the procedure is characterized instead as a breach of the duty of care. Here, the patient's surgery was successful and no other injury is apparent from the facts. The patient's distress at not being informed of the risk is not, standing alone, a compensable injury. [Restatement (Second) of Torts §436A] Hence, the patient is not likely to prevail. (A) is incorrect. While a showing that he would have refused consent had he known of the risks would establish causation for any injury, the patient still needs to establish damages. (B) is wrong because, even though the fact that other physicians would have informed the patient of the risks indicates that the physician may have breached his duty, the damage element must be shown as part of the prima facie case. (C) is incorrect even though that fact would establish breach of duty by the physician. As discussed above, the patient will not prevail absent evidence of damages.

(Torts) 19. In a state that imposed a maximum speed limit of 65 m.p.h. by statute, a truck driver was going 75 m.p.h. on a stretch of highway when a car pulled in front of her. The truck driver was unable to stop in time and hit the car, pushing it into the next lane where it was struck by a bus traveling the speed limit. The collision with the bus caused serious injury to the driver of the car. In an action against the truck driver for negligence, uncontroverted evidence was introduced that, even if the truck driver had been going 65 m.p.h., she would have struck the car and pushed it into the path of the bus. Is the driver of the car likely to prevail? (A) Yes, because the truck driver's breach of the speed limit statute was negligence per se. (B) Yes, because the truck driver's conduct was criminal as well as tortious. (C) Yes, but his recovery will be reduced by the percentage of fault attributed to him by the trier of fact for pulling in front of the truck driver. (D) No, because the violation of the speed law was not a cause in fact of the harm.

ANSWER: (D) No, because the violation of the speed law was not a cause in fact of the harm. The driver of the car will likely not prevail on causation grounds. Even though there is an apparent unexcused violation of the statute, as stated by choice (A), the most that this showing establishes is duty and breach of duty. The plaintiff still must show that the breach of the duty in fact caused the injury to him. The facts indicate that even if the truck driver had been going the legal speed limit, she would have struck the car and pushed it into the path of the bus. Thus, there is no showing that her breach of this duty in fact caused the harm to the driver of the car, and (D) is the best answer. (C) is incorrect. Had the driver of the car established a prima facie case, his recovery would have been reduced because of his contributory fault in pulling in front of the truck driver, but under the facts here he cannot establish the prima facie case. (B) is irrelevant; violation of a criminal statute does not establish tort liability in the absence of causation.

5. A state statute provides that any teacher, guardian, therapist, school administrator, or any other person standing in loco parentis is subject to the same rights and duties as actual parents. No other statute is applicable to these facts. A residential institute for minors with mental or emotional disabilities had no fence surrounding its premises, but regular bedchecks were conducted each night to determine that all of the children were present. One night, a bedcheck determined that a 12-year-old resident of the institute was absent. The security guard who conducted the bedcheck immediately reported to the director of the institute that the boy was missing. The director initiated steps to find the boy but did not report the boy's absence to the police. Thirty-six hours after the boy ran away from the institute, he savagely beat and severely injured a six-year-old girl who lived a few blocks away from the institute. Her parents filed suit against the director on their daughter's behalf. Which party is likely to prevail in the lawsuit?

ANSWER: (D) The director, unless he had reason to know that the boy had a propensity to commit violent acts. If the director had reason to know of the boy's violent propensity, he could be liable for his own negligence in not alerting the authorities. Under principles of vicarious liability, the tortious conduct of one person may be imputed to another person when a special relationship exists between the tortfeasor and the other person. However, a parent is not vicariously liable at common law for the tortious conduct of her child. While many states have statutes making parents liable up to a certain dollar amount for a child's intentional tort, there is no statute of this type in the question. (Recall that you are to assume no applicable statute in Torts questions unless otherwise specified.) The statute given in the question does not change this result; it merely establishes that the director would be liable for the boy's conduct to the same extent as if he were the boy's parent. Given the absence of vicarious liability in this situation and the absence of any other applicable statute, the only liability that the director might face is for his own negligence in not preventing the beating. If the director had reason to know that the boy had a propensity to commit violent acts, the director owed a duty to persons who might be injured by the boy, and this duty would have been breached by the director's failure to report that the boy was missing. Whether the director's negligence can be shown to be the actual cause of the girl's injury is not conclusively established by the facts; but given the short interval between when the boy ran away and when he was discovered missing, and the long interval between the discovery and the beating a few blocks away from the institute, the girl should be able to prove actual cause. Proximate cause can also be established because the boy's intentional tort was foreseeable; it was the very conduct that the director had a duty to take precautions against. Hence, under the factual assumption in choice (D), the director probably will be liable for the girl's injuries. (A) is incorrect because whether the boy would be liable for administering the beating has no effect on the director's liability for his own negligence. (B) is incorrect because the statute merely makes those standing in loco parentis liable to the same extent as parents. Because parents would not be liable in the absence of reason to know of their child's violent propensity, the director also would be liable only under those circumstances. (C) is incorrect even though its statement of law is accurate. While the director cannot be found vicariously liable for the boy's acts, he may be liable for his own negligence under the circumstances stated in (D).

(Evidence) 15. While cross-examining a defendant on trial for robbery and assault with a deadly weapon, the prosecutor asks him whether he was convicted of fraud within the previous year. Is this question proper?

ANSWER: (D) Yes, because it tends to show that the defendant would lie. The defendant has taken the stand in his own defense, and therefore the prosecutor can attack his credibility as a witness. Under Federal Rule 609, evidence of conviction of a crime requiring proof of an act of dishonesty or false statement can always be used to attack a witness's character for truthfulness.

(Evidence) 16. At the defendant's trial for rape, he calls a witness who testifies that she was on her patio barbecuing some hamburgers at the time of the charged rape and saw the assailant run from the victim's apartment. She further testifies that the person who ran from the victim's apartment was not the defendant. On cross-examination by the prosecutor, to which of the following questions would a defense objection most likely be sustained? A "Weren't you convicted of perjury 11 years ago?" B "Weren't you under the influence of heroin at the time you were barbecuing those hamburgers?" C "Haven't you and the defendant known each other since grammar school?" D "Didn't you embezzle funds from your most recent employer?"

ANSWER: A "Weren't you convicted of perjury 11 years ago?" The objection to the perjury question is most likely to be sustained. Federal Rule 609 permits the prosecution to inquire into prior convictions of crimes requiring proof or admission of dishonesty or false statement unless over 10 years have passed since the date of conviction or date of release from confinement (whichever is later). While the facts do not indicate the latter date (or even whether a confinement occurred), (A) remains the best of the four choices. The conviction in (A) is more than 10 years old, so it probably would be subject to objection as being too remote. (B) relates to the witness's ability to perceive and would be a legitimate question on cross-examination. (C) shows a possible bias on the part of the witness, which is an acceptable method of impeachment. (D) relates to a prior bad act that shows dishonesty. Such acts may be asked about on cross-examination of the witness.

(Evidence) 17. In an accountant's trial for filing fraudulent tax reports, the prosecution calls a former colleague of the accountant, and she testifies that the accountant's reputation in the community is for frequently participating in very questionable reporting, often resulting in unnecessary risk for his clients. She testifies further that she thinks the accountant is dishonest. Should the trial court admit this evidence over the accountant's objection? A No, because the prosecution cannot initiate evidence of the defendant's character. B No, because use of the colleague's opinion is improper. C Yes, because it is evidence of the defendant's character for dishonesty. D Yes, because it is evidence of habit.

ANSWER: A No, because the prosecution cannot initiate evidence of the defendant's character. The court should not admit this evidence because the prosecution cannot initiate evidence of the defendant's bad character. The prosecution may offer such evidence only after the accused has put his character in issue by either taking the stand (thus placing his credibility in issue) or offering evidence of his good character. Thus, (C) is incorrect. (B) is incorrect because, under the Federal Rules, character may be proven by opinion evidence. (D) is incorrect because this does not constitute a regular response to a specific set of circumstances; it is merely reputation and opinion evidence.

(Evidence) 19. A mechanic was tried for the aggravated assault of a nurse. The mechanic called a witness to the witness stand. The witness was to testify that the night before the alleged crime, the mechanic stated to the witness that he was going to visit his mother in a distant city some 1,000 miles away. Is the testimony admissible? A)Yes, because it is a declaration of intent to do a future act. B)Yes, because of the verbal acts exception. C)No, because it is not relevant. D)No, because it is hearsay not within any exception.

ANSWER: A)Yes, because it is a declaration of intent to do a future act. The testimony would be hearsay, but it would be admissible under the exception of a declaration of an intent to do a future act, sometimes referred to as a declaration of present state of mind. Under this exception, a declaration of intent to do something in the future is admitted as circumstantial evidence tending to show that the intent was carried out. [Fed. R. Evid. 803(3)] The trip provides an alibi defense and is therefore relevant; so (C) is incorrect. (B) is wrong because this is not a verbal act; verbal acts are out-of-court statements that when spoken have legal significance, e.g., words of contract or defamation. (D) is incorrect because while the statement constitutes hearsay, it does fall under an exception: the declaration of intent to do a future act.

5. A seller and a buyer entered into negotiations over the telephone for the buyer to buy widgets from the seller. Following their conversation, the seller sent the buyer a contract, already signed by the seller, agreeing to sell 1,000 widgets to the buyer for a total contract price of $10,000. Upon receipt of the contract in the mail, the buyer signed the contract, placed it in an envelope, and deposited the envelope in the mailbox located in front of the buyer's office building. Before the seller received the contract, the buyer had a change of heart. He telephoned the seller and said, "Look, I just can't make a profit on those widgets. I'm not interested in that contract we talked about." The seller replied, "That's all right, I understand. Maybe we can do business some other time." The next day, the signed contract was delivered to the seller's office. The seller, also having had a change of heart, decided that he wanted to enforce the contract.Is the contract enforceable against the buyer? A. Yes, because the acceptance occurred prior to rejection. B. Yes, because of the parol evidence rule. C. No, because the offer to rescind was accepted and that discharged the original contract. D. No, because the rejection by telephone voided the acceptance by mail.

ANSWER: A. Yes, because the acceptance occurred prior to rejection. The contract is enforceable because the "mailbox rule" applies here. Acceptance by mail creates a contract at the moment of posting, properly addressed and stamped, unless the offer stipulates that acceptance is not effective until received, or an option contract is involved. If the offeree sends an acceptance and then rejects the offer, the mailbox rule applies; i.e., a contract is created upon dispatch of the acceptance. Because no option contract is involved here, and the seller's offer did not state that the buyer's acceptance would be effective only when received, the buyer's acceptance was effective the moment he placed the envelope containing the contract in the mailbox. The buyer's attempt to reject occurred after acceptance took place. Thus, a valid contract was formed and the seller may enforce it. (B) is incorrect because the parol evidence rule is inapplicable to the issue of enforceability. The parol evidence rule applies to prevent the introduction of evidence of certain statements made before or concurrently with the execution of a written contract that was intended by the parties to be the complete and final expression of their agreement. These statements are inadmissible to vary the terms of the written agreement. Here, the issue is not the terms of the agreement, but rather whether there is an agreement at all. Note that the parol evidence rule does not prevent the introduction of evidence that affects whether a contract was formed or whether it is valid. Thus, the parol evidence rule would not prevent the introduction of evidence of an oral rescission of the contract. Here, the attempted rescission is ineffective because there was no meeting of the minds. (C) is incorrect because there is no "meeting of the minds" concerning the rescission. A contract may be discharged by an express agreement between the parties to rescind; the agreement to rescind is itself a binding contract. Because the seller did not know that the buyer had accepted the offer, the seller did not know there was a contract to rescind. Thus, seller's statement that "that's all right" cannot be construed as acceptance of the buyer's offer to rescind. Therefore, a contract to rescind was not formed. (D) is incorrect because the telephone rejection did not void the acceptance by mail. As discussed above, if the offeree sends an acceptance first, followed by a rejection, the mailbox rule applies; i.e., a contract is created upon dispatch of the acceptance. Because the buyer's telephone rejection took place after his acceptance by mail, the rejection was ineffective. While an offeree will be estopped from enforcing the contract if the offeror receives the rejection first and changes his position in reliance on it, the offeror (the seller) is the one wanting to enforce the contract here.

(Evidence) 14. In a medical malpractice action, the plaintiff sought to have her neighbor testify that, the day after receiving treatment on her back from the defendant, the plaintiff told the neighbor that her back was getting worse. Upon proper objection, how should the court rule on this testimony? A)Admit it, because it is a statement of personal history. B)Admit it, because it is a statement of a then-existing physical condition. C)Exclude it, because it is hearsay not within any exception D)Exclude it, because it was not made for the purpose of medical treatment.

ANSWER: B) Admit it, because it is a statement of a then-existing physical condition. The testimony will most likely be admissible as a statement of a then-existing physical condition. Declarations of existing physical condition are admissible as an exception to the hearsay rule. Under the Federal Rules, these declarations are admissible even though not made in the context of medical diagnosis or treatment. (C) is incorrect. The statement would be hearsay but it would qualify under the exception for statements describing current physical condition. (D) is incorrect. There is a hearsay exception for statements made for medical diagnosis or treatment. Under that exception, a statement of medical history, past or present symptoms, or their inception or cause is admissible if it was made for, and is reasonably pertinent to, diagnosis or treatment. It is true that the plaintiff's statements would not qualify under that exception because she was not seeking diagnosis or treatment during her conversation with the neighbor. However, the separate exception for a declarant's then-existing physical condition applies regardless of whether the declarant was seeking medical diagnosis or treatment when she made the statement. Thus, the neighbor's testimony about the plaintiff's statement is likely admissible. (A) is incorrect because statements of physical condition do not qualify as statements of personal history. The hearsay exception for statements of personal or family history pertains to statements concerning birth, marriage, adoption, etc. Furthermore, this exception requires the declarant to be unavailable, which is not the case here.

(Evidence) 12. The defendant was charged with first degree felony arson under the state's arson statute. The indictment alleged that the defendant burned down his own building to collect the insurance. At the trial, the defense called a witness who testified on direct examination that he was with the defendant at the place of business when the fire started and that some cleaning solvent caught fire and spread out of control. The witness testified that the ignition of the fire was purely accidental. The defense also introduced evidence that the witness is of good character. On cross-examination, the prosecutor asked the witness if he is being prosecuted for first degree felony arson in a separate trial for the burning of the same building. The defense lawyer objects. Should the court allow the prosecutor's question? A)Yes, because the defense lawyer has introduced evidence tending to establish that the witness is a person of good character. B)Yes, because the question is appropriate to show bias or interest on the part of the witness. C)No, because the witness has not been convicted of the crime. D)No, because the question violates the witness's Fifth Amendment right to be protected from self-incrimination.

ANSWER: B)Yes, because the question is appropriate to show bias or interest on the part of the witness. The court should allow the prosecutor's question because it is appropriate to show bias or interest on the part of the witness. Evidence that a witness is biased or has an interest in the outcome of the case tends to show that the witness has a motive to lie. Bias or adverse interest can be proved by cross-examination or extrinsic evidence, and in some cases, both. Here, the fact that the witness is being prosecuted for the same crime tends to show that he has a motive to lie in saying that the fire started accidentally. Thus, it is proper impeachment for the prosecutor to cross-examine the witness about his own prosecution. (A) is wrong because all witnesses are subject to impeachment, and evidence (including character evidence) that bears on truthfulness is always admissible (although the means of proof may be restricted). This choice confuses the basis for the prosecution's introduction of substantive character evidence against a defendant in a criminal trial with valid impeachment of a witness for bias or interest. Nor would it matter if the defense lawyer's evidence of the witness's good character was admitted erroneously (because generally a witness's testimony cannot be bolstered until the witness has been impeached). Regardless of the defense's introduction of that evidence, a witness may always be impeached with evidence of bias. (C) is wrong because conviction of a crime is not a requisite for introduction of evidence showing bias or interest. A felony conviction or a conviction for a crime involving dishonesty is an entirely separate method of impeachment. (D) is wrong for two reasons: (i) the answer to the question could in no way incriminate the witness; and (ii) even if the answer could tie the witness to the commission of the crime, he could invoke the privilege to refuse to answer—the question itself would not be objectionable.

(Evidence) 18. The defendant was charged with reckless driving after his car hit a car driven by a priest. At trial, the passenger in the defendant's car at the time of the accident testified that the priest slammed on the brakes with no warning before the accident. The defendant was acquitted of the reckless driving charge. Subsequently, the priest sued the defendant for personal injuries and property damages. At the civil trial, counsel for the defendant asked the passenger, "Did the priest stop suddenly before the defendant hit him?" The passenger, who was angry with the defendant after an argument, responded, "I don't remember it that way. The priest may have slowed down a bit." The defense attorney seeks to introduce the transcript of the passenger's statement from the criminal trial. The priest's attorney objects. Is the transcript admissible? A)Yes, for impeachment purposes only. B)Yes, for impeachment purposes and as substantive evidence. C)No, because it is hearsay. D)No, because the passenger is not unavailable.

ANSWER: B)Yes, for impeachment purposes and as substantive evidence. The transcript is admissible both for impeachment purposes and as substantive evidence. For the purpose of impeaching the credibility of a witness, a party may show that the witness has, on another occasion, made statements that are inconsistent with some material part of his present testimony. In most cases, prior inconsistent statements are hearsay, admissible only to impeach the witness. However, under the Federal Rules, if the prior inconsistent statement was made under penalty of perjury at a prior trial, hearing, or deposition, it is not hearsay. Here, the passenger's prior statement was made at the defendant's criminal trial. Therefore, the statement is admissible to impeach the passenger and as substantive proof that the priest stopped suddenly. Thus, (A) and (C) are incorrect. (D) is incorrect because it imposes the unavailability requirement of the former testimony exception to the hearsay rule. Since this evidence is not hearsay under the Federal Rules, the former testimony exception and its requirements do not apply.

(Evidence) 11. The plaintiff slipped on a patch of ice at the defendant's home and is suing the defendant for negligence. The defendant denies that the plaintiff was injured in the fall. The defendant calls his neighbor to testify that right after the plaintiff fell, she said that she had a recurring hip injury and was experiencing a flare-up. Should the court admit the neighbor's testimony? A)No, because it constitutes inadmissible hearsay. B)No, because it is irrelevant. C)Yes, because it qualifies as a statement against interest. D)Yes, because it is a statement of a party-opponent.

ANSWER: D) Yes, because it is a statement of a party-opponent. The court should admit the testimony as a statement of a party-opponent. Any statement attributable to a party-opponent is considered nonhearsay under the Federal Rules. Thus, the plaintiff's statement that her hip injury was a recurrence of an old injury may be offered against her by the defendant. (A) is incorrect because, as discussed above, the plaintiff's statement is nonhearsay. (B) is incorrect because the statement is relevant. It relates to whether the plaintiff's injury resulted from her fall on the defendant's property, which is a fact of consequence in this case. As such, it is relevant. (C) is incorrect because there is no indication that the plaintiff's statement was against her interest when made, even though it is now harmful to her position in her current lawsuit against the defendant. Furthermore, this exception requires the declarant to be unavailable; here the plaintiff is present and there is no indication that she is otherwise unavailable to testify.

(Evidence) 21. A witness was in a store when she heard a screeching of brakes and immediately rushed out of the store. She saw a car speeding off into the distance and found a victim, badly injured and lying in the street. The victim gasped to the witness, "I'm going to die. The car that hit me had license number DD666!" The victim then lapsed into unconsciousness. The witness gave her information to the police, including a description of the car and the victim's comment on the license plate. The police traced the registration to the defendant. The victim recovered from his injuries and filed suit against the defendant for his injuries. At the trial, the victim wants to have the witness testify as to the victim's statement regarding the defendant's license number. The court should rule that such testimony by the witness is: A)Inadmissible, because it is more prejudicial than probative. B)Inadmissible, because it is hearsay not within any recognized exception to the hearsay rule. C)Admissible, as a declaration made in belief of impending death. D)Admissible, as an excited utterance.

ANSWER: D)Admissible, as an excited utterance. The testimony should be admissible. The witness's proposed testimony about the victim's statement ("I'm going to die. The car that hit me had license number DD666!") is hearsay evidence. Hearsay is an out-of-court statement offered in evidence to prove the truth of the matter asserted in the statement. [Fed. R. Evid. 801(c)] Because it appears that the victim's statement is being offered to establish that the car that hit him had license number DD666, it is hearsay. Nevertheless, it is admissible under the "excited utterance" exception to the hearsay rule. Hence, (D) is correct. At first glance it might seem that the "dying declaration" exception to the hearsay rule applies as well. [See Fed. R. Evid. 804(b)(2)] There is no indication, however, that the declarant (the victim) is unavailable to testify, a prerequisite for admitting evidence under the dying declaration exception. [Fed. R. Evid. 804(a)] The victim is a party and thus would seem to be available to testify. If the victim were to testify that he had no memory as to the license number, he would be considered unavailable [Fed. R. Evid. 804(a)(3)], but there is no indication that the victim has so testified. Therefore, the dying declaration exception does not apply, and (C) is incorrect. (B) is incorrect because the witness's proposed testimony is admissible under the hearsay exception for "excited utterances." [Fed. R. Evid. 803(2)] This hearsay exception, like most, does not require the declarant to be unavailable as a condition for the admission of hearsay evidence. It merely requires that the hearsay statement relate to a startling event or condition, made while the declarant was under the stress of excitement caused by the event or condition. The victim's statement relates to a startling event (the car crash) that prompted the statement. Because the statement was made so soon after the crash, while the victim was badly injured, it probably constitutes an excited utterance. The witness's testimony is highly probative in establishing that the defendant was driving the car that struck the victim. (A) is incorrect in asserting that the witness's testimony is inadmissible because it is more prejudicial than probative. The witness's testimony is in no sense unfairly prejudicial to the defendant.

(Evidence) 10. A woman was injured when the car she was driving was struck by a moving truck. The woman brings an action for personal injuries against the moving company. The complaint alleges that the driver was drunk at the time of the accident and that the moving company was negligent in hiring him and permitting him to drive knowing that he had a drinking problem and convictions for drunk driving. The driver is called as a witness by the moving company and is expected to testify that he was not drunk at the time of the accident. Instead, the driver states on direct examination that he had had several beers as he drove his truck that evening and was under the influence of drugs when his truck struck the woman's car. The counsel for the moving company wants to confront the driver with his deposition testimony that he was completely sober at the time of the accident. Will this evidence be permitted? A. No, the statement is hearsay not within any recognized exception. B. No, the moving company cannot impeach its own witness. C. Yes, but it may be used only to impeach the driver. D. Yes, it can be used to impeach the driver and as substantive evidence that the driver was sober.

ANSWER: D. Yes, it can be used to impeach the driver and as substantive evidence that the driver was sober. The evidence will be permitted to impeach and as substantive evidence. For the purpose of impeaching the credibility of a witness, a party may show that the witness has, on another occasion, made statements that are inconsistent with some material part of his present testimony. Furthermore, a testifying witness's prior inconsistent statement made while under oath and subject to penalty of perjury in a deposition or prior hearing is not hearsay under the Federal Rules, and is therefore admissible as substantive evidence of the facts asserted. (A) is therefore incorrect. (B) is incorrect because the Federal Rules permit a party to impeach its own witness, even if not "surprised." (C) is incorrect because it is too narrow; the evidence is admissible as substantive evidence as well as to impeach the driver.

(Criminal Procedure) 36. A defendant was arrested for carrying a concealed weapon in violation of his parole. The police had reason to believe that the defendant was part of a group that sold stolen automobile parts. The police interrogated the defendant continually for 10 hours. Finally, the police threatened to have the defendant's parole revoked if he did not cooperate. Fatigued from the lengthy interrogation and afraid of going back to jail, the defendant confessed to being a member of the group involved in the theft of automobile parts. The defendant gave the police the location of a friend's garage where the stolen parts were located. Based on this confession, the police obtained a search warrant to search the garage, where they discovered a large quantity of stolen automobile parts. The defendant was prosecuted for conspiracy to sell stolen automobile parts. Before trial, he moved to prevent use of the stolen parts as evidence. The court will most likely rule that the evidence should be: (A) Admitted, because it was obtained pursuant to a warrant. ...

*(NOT MY ANSWER OR BARBRI): Suppressed, because fruits of involuntary statement (even though no physical intimidation, statement was made involuntary). WRONG ANSWER: (A) Admitted, because it was obtained pursuant to a warrant.

26. The owner of a house put the property up for sale. A surgeon entered into negotiations with the owner to purchase the house, and the parties agreed upon a sale price of $200,000 and a closing date. The owner told the surgeon that she would drop a contract in the mail and have her attorney draw up a deed. The owner signed a land sale contract, which included the property's address but did not contain a metes and bounds legal description. She mailed the contract to the surgeon that afternoon, although it was mailed too late for the last mail pickup of the day. The owner's attorney promptly drew up a deed and dropped it in the mail to his client. The surgeon received the contract the next day. After she mailed the contract, the owner received an offer of $250,000 for her property from her next-door neighbor, who wanted to expand beyond his own property line. The owner called her attorney and told him to inform the surgeon that the deal was off. The attorney e-mailed the surgeon, stating that his client had found another purchaser for the property, and that all matters regarding the surgeon's offer for the property were rescinded. The surgeon signed the contract anyway and returned it to the homeowner by registered mail. If the surgeon brings an action to compel the owner to convey the property to him for $200,000, is he likely to prevail? (A) Yes, because the owner signed the land sale contract. (B) Yes, because the contract was effective when the owner placed the document in the mail. (C) No, because the surgeon did not mail the signed contract until after he received the revocation by e-mail. (D) No, because the land sale contract does not contain the complete legal description of the property

ANSWER: (A) Yes, because the owner signed the land sale contract. The surgeon is entitled to specific performance because the owner signed the land sale contract. A contract was formed here when the parties orally agreed to the sale of the property. However, the contract was unenforceable at that time because, under the Statute of Frauds, a contract for the sale of land is unenforceable unless a memorandum containing the contract's essential terms is signed by the party to be charged. Here, the party to be charged is the owner, and she signed the land sale contract, a writing sufficient to satisfy the Statute of Frauds (a memorandum for the sale of land is sufficient if it contains the price, a description of the property—which need not be a "legal" description—and a designation of the parties). Thus, the contract was enforceable. Specific performance is allowed when the legal remedy (damages) would be inadequate, such as with contracts to purchase land, which is unique. Therefore, the surgeon is entitled to specific performance (assuming the property has not already been sold to a bona fide purchaser); under the facts the neighbor had made an offer but nothing indicates that the owner accepted the offer yet. (B) is incorrect because the contract was effective when the parties orally agreed to it. The contract became enforceable when the owner signed it, not when she placed it in the mail. (C) is incorrect because this is not a mailbox rule issue. The contract was already formed before the attorney attempted to revoke the offer. Thus, his attempt at revocation was ineffective. It did not matter when the surgeon mailed the contract. (D) is incorrect because, to satisfy the Statute of Frauds, a description need not be a complete legal description, but need merely be sufficient to reasonably identify the subject of the contract. It is sufficient that the property was identified by its address.

(Criminal Procedure) 10. While walking on patrol in a commercial district in the early evening, a police officer noticed that a light was on in the back window of a machine shop. Curious about what was going on inside, the officer went to the back of the building and tried to look through the window of the shop, but it had been painted on the inside so that only a strip about three inches at the top, eight feet above ground level, was still transparent. The officer quietly brought two trash cans from a neighboring business over to the window, stood on them and saw, through the strip of unpainted window, that the shop owner's son was inside with a friend, sucking white powder into his nose through a rolled-up tube of paper from off a small mirror. Recognizing that the shop owner's son was snorting cocaine, the officer knocked at the front door to the shop, and the son let him in. The officer immediately arrested the owner's son and his friend. In the back room of the shop through whose window he had peered, the officer found and seized several grams of cocaine, a razor blade, and a mirror. In the subsequent prosecution of the shop owner's son for possession of cocaine, the owner's son seeks to bar introduction of the cocaine, mirror, and razor blade into evidence. Will his motion be granted? A. Yes, because the officer could not have known that the owner's son was snorting cocaine absent a chemical test of the substance being snorted. B. Yes, because the officer violated the owner's son's reasonable expectation of privacy. C. No, because the search was incident to a valid arrest. D. No, because the owner's son consented to the officer's entry into the shop.

ANSWER: B. Yes, because the officer violated the owner's son's reasonable expectation of privacy. The shop owner's son had a reasonable expectation of privacy, as evidenced by the obscuring of the window so that neighbors could not see into the shop. Hence, the officer's search would have to be based on a valid warrant or qualify under one of the exceptions to the warrant requirement. Putting trash cans next to a back window to climb upon and peering into the window through a narrow opening eight feet above the ground would be considered a violation of the owner's son's Fourth Amendment rights and not a "plain view" of criminal activity. Because the seizure of the cocaine, mirror, and razor was based on the illegal search, the evidence could not be used by the state. (A) is wrong; absolute certainty of illegal activity is not required for a valid search. A reasonable belief is required. (C) is wrong. The arrest itself is probably invalid, and in any event a search of the next room would not be an area within the immediate control of the defendant. (D) is wrong. Consent to enter the shop is not a consent to search the back room.

(Criminal Procedure) 9. A drug smuggler had just returned home after smuggling in a large quantity of cocaine in the false bottom of his suitcase. As he was about to leave his house again to deliver the cocaine to his contact in the city, a police officer arrived with a trained drug sniffing dog and asked him if he could come in and ask him some questions. The smuggler declined but the officer stepped into the doorway and the dog immediately caught the scent of the cocaine and pulled the officer toward the suitcase in the hallway. Based on the dog's clear indication that the suitcase contained narcotics, the police officer opened the suitcase and found the cocaine. The smuggler was then arrested and the cocaine and suitcase seized. At a pretrial hearing, should the judge grant the smuggler's motion to suppress evidence of the cocaine in the suitcase? (A) Yes, because a warrantless search and seizure of items within the defendant's home is not permissible absent exigent circumstances. (B) Yes, because the search and seizure required a warrant. (C) No, because the cocaine was seized as a search incident to a lawful arrest. (D) No, because there is no legitimate expectation of privacy in the smell of one's suitcase.

MY ANSWER (NOT BARBRI'S): (B) Yes, because the search and seizure required a warrant.

11. On January 1, a club owner entered into a written contract with a singer providing that the singer was to sing nightly at the club for the next six months at a set salary, commencing February 1. The club owner received no further communication from the singer until February 1, at which time he received a telegram from the singer stating: "Due to circumstances beyond my control, I will not be able to start my singing engagement at your club until February 10. I'm sorry for any inconvenience this causes you." On February 10 the singer appeared at the nightclub, ready to sing. May the club owner cancel the contract? (A) Yes, because the singer failed to start singing when he contracted to do so. (B) Yes, because the singer's actions constitute a material breach. (C) No, unless the club owner was materially prejudiced by the singer's failure to start singing on February 1. (D) No, because the singer notified the club owner of his delay in performance in a timely fashion.

MY ANSWER (NOT BARBRI'S): (C) No, unless the club owner was materially prejudiced by the singer's failure to start singing on February 1.

(Criminal Procedure) 3. A police officer saw a car containing three teenagers driving slowly down the street at 1 a.m. She waited for it to go by her and, after it was far enough ahead, started to follow it. Several blocks later, the car rolled through a stop sign. The officer immediately pulled the car over and requested the driver's license. A license check showed that the driver had five outstanding parking tickets. A statute in the jurisdiction permits an arrest to be made if a driver has four or more outstanding parking or traffic violations. The officer decided to take the driver in on the tickets. She informed the driver that he was under arrest and asked him to step out of the car. When the driver got out, the officer patted him down and found a gun in his waistband. Calling for backup, she decided to haul all three teenagers to jail. Subsequent testing showed that the gun had been used in a recent homicide during a store robbery by three young men. One of the passengers made a motion to prevent the introduction of the gun at his trial for murder and robbery. How should the judge rule? (A) Deny the motion, because the officer was legitimately concerned for her own personal safety. (B) Deny the motion, because the gun was found after the driver has been arrested. (C) Grant the motion, because the officer had no valid reason to be following the automobile. (D) Grant the motion, because the officer had not arrested the driver for suspicion of robbing the store or committing the homicide. (Same question, different answer choice) E. Deny the motion, because the officer lawfully stopped the car.

*ANSWER: E. Deny the motion, because the officer lawfully stopped the car. (But in my question it would be either A or B). Evidence will be suppressed if it was obtained in violation of the defendant's constitutional rights. Each passenger in a car has standing to challenge a stop of the car. If a stop is invalid, under the fruit of the poisonous tree doctrine, evidence obtained as a result of the invalid stop will be suppressed. Here, the officer had a valid reason to stop the car. An officer may stop a car for violating a traffic law, and here the driver of the car failed to stop at a stop sign. A person may seek suppression of evidence that has been seized only if the seizure is in violation of the person's own constitutional rights. While it is true that the gun was properly seized from the driver because the arrest appears to have been valid, the gun would be admissible against the passenger even if it had been unlawfully seized from the driver. The driver, of course, would have standing to complain of the unlawful seizure, but not the passenger. Barbri outline: Police may order occupants out. Provided that a police officer has lawfully stopped a vehicle, in the interest of officer safety, the officer may order the occupants (i.e., the vehicle's driver and passengers) to get out. Moreover, if the officer reasonably believes that the detainee is armed and dangerous, she may conduct a frisk of the detainee. She may also search the passenger compartment of the vehicle to look for weapons, even after the driver and other occupants have been ordered out of the vehicle. Pretextual stops. If an officer has probable cause to believe that a traffic law has been violated, the officer may stop the suspect's automobile, even if the officer's ulterior motive is to investigate a crime for which the officer lacks sufficient probable cause to make a stop.

(Criminal Procedure) 12. After a bomb explosion in an airport locker, a detective was told by a reliable informant that, three months before, the informant had been in a terrorist's apartment and saw what appeared to be some sticks of dynamite. Reasonably believing that the informant's information established probable cause, the detective prepared an affidavit for a search warrant. After the warrant was issued, the detective and a group of police raided the terrorist's apartment. No evidence connecting the terrorist with the bombing was discovered, but the police did discover several grams of cocaine during the search of the apartment. At his trial for possession of narcotics, the terrorist's motion to suppress the evidence would probably be: (A) Denied, because the informant was a reliable informant and the detective reasonably believed that the informant's information was accurate and that the warrant was properly issued. (B) Granted, because in fact the police did not discover any evidence linking the terrorist to the bombings and, therefore, the seizure of the cocaine was fruit of the poisonous tree. (C) Granted, if the court determines that the information supplied by the informant to the detective concerned information too remote in time to justify a claim of probable cause at the time the detective requested the search warrant. (D) Granted, because the search warrant was not issued for the purpose of searching the terrorist's apartment for illegal drugs.

*MY ANSWER (NOT BARBRI'S): (A) Denied, because the informant was a reliable informant and the detective reasonably believed that the informant's information was accurate and that the warrant was properly issued.

(Criminal Procedure) 1. An underworld informer advised a police investigator that his neighbor was running an illegal bookmaking operation in his apartment, and that the informer had placed bets with the neighbor at this location. The officer obtained a search warrant, based on his affidavit reciting the foregoing facts, and further stating that the underworld informer was a person who had given him accurate information in previous cases, but whose identity could not be revealed because it might jeopardize other criminal investigations being carried on by the police. Armed with the search warrant, police officers went to the neighbor's apartment. They entered when the neighbor opened the door and searched the apartment. They seized various wagering slips and bookmaking apparatus (described in the search warrant) and placed the neighbor under arrest for illegal gambling. Prior to trial, the neighbor challenges the validity of the search warrant. Was the search warrant valid? A No, because it was based on hearsay information. B No, because the officer failed to disclose the identity of the informer, so that the accuracy of his information could not be verified. C Yes, because the identity of the informer is never required. D Yes, because the affidavit accompanying it is sufficiently detailed to allow a determination of probable cause.

ANSWER: D. Yes, because the affidavit accompanying it is sufficiently detailed to allow a determination of probable cause. The search warrant is valid because the affidavit accompanying it is sufficiently detailed to allow a determination of probable cause. A warrant must be based on a showing of probable cause. Along with a request for a warrant, a police officer must submit to a magistrate an affidavit setting forth sufficient underlying circumstances to enable the magistrate to make a determination of probable cause independent of the officer's conclusions. The affidavit may be based on an informer's statements. The sufficiency of the affidavit is evaluated according to the "totality of the circumstances." There must be sufficient information for the magistrate to be able to make a common sense evaluation of probable cause. Among the factors determinative of probable cause are the informer's reliability, credibility, and basis of knowledge. Here, the officer's affidavit indicates that the informer has previously proved to be reliable by providing accurate information in other cases. This, in turn, enhances the credibility of the informer. Also, the informer's knowledge is based on his having personally placed bets with the neighbor at his apartment. Thus, the officer's affidavit is supported by sufficient underlying circumstances to allow a magistrate's finding that there was a showing of probable cause. (A) is incorrect because probable cause for issuance of a search warrant may be based on hearsay, if the information comes from a reliable informer. (B) is incorrect because the failure to disclose the identity of the informer does not necessarily invalidate the search warrant. The identity of an informer does not have to be revealed to allow the magistrate to make a determination of probable cause. The magistrate may make this determination based on the police officer's information about the informer showing reliability, credibility, and knowledge. The magistrate need not personally question the informer. (C) is incorrect because it is overbroad. An informer's identity need not be revealed only if there is sufficient other evidence to make a probable cause determination.

30. (Same as #10 but items and people different) On April 1, a music store owner offered to sell a rare piano to his best customer, a concert pianist, for $100,000. The following day, the pianist, who performs around the world with two of the several pianos he has purchased from the store, wrote to the store owner: I have decided to purchase the piano. A check for $100,000 is enclosed. I am leaving in one week for Canada. I will be gone for one month and will pick up the piano when I return. I will pay you to store the piano in your air-conditioned warehouse. One week later, the pianist left for Canada without hearing from the music store owner. What does the letter that the pianist wrote to the store owner constitute? A: A conditional acceptance. B: A rejection of the offer. C: An acceptance, and the store owner is not bound to store the piano. D: An acceptance, and the store owner must store the piano but is entitled to the reasonable value of that service.

ANSWER: D: An acceptance, and the store owner must store the piano but is entitled to the reasonable value of that service. The pianist's letter to the store owner is an acceptance, and the store owner must store the piano. This is a contract for a sale of goods and thus is governed by the UCC. Under the UCC, an acceptance with additional terms does not constitute a rejection and counteroffer, but rather is an effective acceptance unless made expressly conditional on the assent to the additional terms. Here, the pianist expressly accepted the store owner's offer and included an additional term adding one month of storage. The acceptance was not expressly conditional on the store owner's assent to the storage term. Thus, the acceptance was sufficient to create a contract. Whether additional terms become part of the agreement depends on whether both parties are merchants. If both parties are merchants, the additional terms become part of the contract unless they materially alter the terms of the offer, the offer expressly limited the acceptance to its terms, or they are objected to within a reasonable time. A merchant as one who regularly deals in goods of the kind sold or who otherwise by his occupation holds himself out as having knowledge or skill peculiar to the practices or goods involved. For purposes of the UCC battle of the forms provisions, a merchant is almost anyone in business because anyone in business has knowledge of business practices. Here, the store owner is clearly a merchant. The pianist, by virtue of his occupation, has knowledge or skill peculiar to the goods (piano) involved. He plays the piano professionally and has purchased several pianos for this purpose. The additional term included in the pianist's acceptance did not materially alter the terms of the offer (i.e., it did not change a party s risk or remedies), the offer was not limited to its terms, and the facts indicate that the store owner did not object. Thus, the additional term regarding storage becomes part of the agreement. (A) is incorrect because the pianist did not make the acceptance conditional on the owner's assent to the additional term. (B) is incorrect because, unlike at common law, under the UCC, an acceptance that contains additional terms (i.e., one that is not a mirror image of the offer) is not a rejection and counter-offer. (C) would be the correct answer if the pianist were not a merchant. If one of the parties to a contract is not a merchant, any additional terms in the acceptance will be ignored unless specifically accepted.

10. On July 1, a rancher offered to sell his prize bull to a breeder for $15,000. On July 10, the breeder wrote the rancher as follows: I have decided to take the bull. A check for $15,000 is enclosed. I am leaving for Argentina for six months and will pick up the bull on January 1. I will pay you for its board and care. The breeder's letter is: (a) A counteroffer, because it changes the terms of the offer. (b) A counteroffer, because it was not a definite expression of acceptance. (c) An acceptance, and the rancher must board the bull but is entitled to the reasonable value of that service. (d) An acceptance, and the rancher may refuse to board the bull.

MY ANSWER (NOT BARBRI'S): c) An acceptance, and the rancher must board the bull but is entitled to the reasonable value of that service.

(Criminal Procedure) 6. A police officer spent several hours using binoculars to observe an older man loitering on a college campus. The man, who was shabbily dressed and carrying a backpack, would approach certain students as they walked by him, and after a brief conversation with them, discreetly pass the students a small envelope in exchange for cash. The officer stopped the man under suspicion that he was dealing drugs. The man was not dealing drugs, but instead had been soliciting donations for a radical political group. The man grew irate when the officer opened one of the envelopes in question and discovered that they only contained literature about the group. The officer then frisked the man and discovered an illegal weapon taped to his leg. The officer immediately arrested the man. Which of the following best describes the situation? (A) The officer's actions were unlawful because the officer initially failed to get an arrest warrant before approaching the man, even though he had ample time to do so because the surveillance had been going on for several hours, (B) The man's arrest was unlawful because the officer was mistaken about the man selling drugs and thus the weapon would be inadmissible as fruit of the poisonous tree. (C) The officer's actions were lawful in stopping the man because the officer had reasonable grounds to believe that the man was dealing drugs, but the subsequent search was unlawful once the officer realized his mistake about the drug dealing. (D) The stop, search, and subsequent arrest were lawful.

*ANSWER: (C) The officer's actions were lawful in stopping the man because the officer had reasonable grounds to believe that the man was dealing drugs, but the subsequent search was unlawful once the officer realized his mistake about the drug dealing.

15. A music promoter planned to open a discotheque and bar on the outskirts of town. He hired a builder to build it, with a completion date of September 15. Although the promoter was very optimistic that the disco would be a big success, its profitability could not be determined with certainty. First year profits were estimated to be about $1,000 per day. To encourage the builder to work in a timely manner, the contract included a liquidated damages clause, providing that the builder would pay the promoter $10,000 per day for each day the contract ran over its completion date. The builder's work progressed smoothly and would have been finished on time, except that the builder failed to place in a timely manner his order for the disco's specially manufactured dance floor lighting. Consequently, the work was not completed until October 15. The promoter sued the builder for breach of contract. The builder called a witness who testified that the disco would have received $30,000 in income during that 30-day period and would have expended $20,000, leaving a profit of $10,000. How much should the builder be required to pay to the promoter in damages? (A) $10,000, representing the promoter's lost profits. (B) $30,000, representing the promoter's lost income. (C) $300,000, representing damages provided in the contract. (D) $310,000, representing damages in the contract plus lost profits.

ANSWER: (A) $10,000, representing the promoter's lost profits.

16. (Same question as 15. but slight differences) A hotelier planning to build a new hotel estimated that first year profits would be about $10,000 per day. To encourage its contractor to work in a timely manner, the hotelier included in their contract a liquidated damages clause, providing that the contractor would be liable to pay the hotelier $100,000 per day for each day that the contractor is late in completion of the project. During the project, the contractor failed to place an order for the hotel's elevators in a timely manner. As a result of that oversight, the elevators were installed late and the hotel consequently opened 30 days later than scheduled. The hotelier sued the contractor for damages. At trial, an expert witness testified that the hotel would have received $300,000 in income during that 30-day period and would have expended $200,000, leaving a total profit of $100,000. How much should the contractor be required to pay to the hotelier in damages? (A) $100,000, representing the promoter's lost profits. (B) $300,000, representing the promoter's lost income. (C) $3 million, representing damages provided in the contract. (D) $3.1 million, representing damages in the contract plus lost profits.

ANSWER: (A) $100,000, representing the promoter's lost profits. The hotelier will be able to recover $100,000, its lost profits. The purpose of contract damages is to put the nonbreaching party into as good a position as it would have been in had the breaching party fully performed. This would be represented here by the hotelier's lost profits, which is its income ($300,000) minus its expenses ($200,000), or $100,000. The liquidated damages provision will not apply because it is unreasonable (see below). (B) is incorrect because giving the hotelier its $300,000 income would put it in a better position than it would have been in if the contractor had performed, because such an award does not take into account the expenses that the hotelier avoided by not being in operation. (C) is incorrect because a court would not uphold the liquidated damages clause here. Liquidated damages clauses are enforceable only if damages were difficult to estimate at the time the contract was formed, and the amount agreed upon is a reasonable forecast of the damages that would result from a breach. Here, the damages may have been difficult to predict when the contract was formed because it is not known just how well a new hotel will do; however, the amount agreed upon is too high. The facts indicate that a reasonable estimate at the time of the contract was about $10,000 per day, but since the liquidated damages amount ($100,000 per day) is 10 times that amount, this amount was unreasonable. (D) is incorrect because it seeks to combine the actual damages with the liquidated damages. Regardless of whether the liquidated damages clause will be enforced here, this measure is improper because a party may recover either liquidated damages, or, if not available, the actual damages, but not both.

(Torts) 4. A patient with a degenerative eye disease visited a well-known eye surgeon, who told him that there were two ways to treat his eye: a more traditional surgical method or a recently developed injection method. Although the injection method had the advantage of a higher success rate, it also carried a 25% risk that vision would be lost completely, whereas such a risk was only 10% by the surgical method. The surgeon described both methods to the patient but did not tell the patient of the risk factors involved. The surgeon asked the patient which method he would like to use, but the patient told the surgeon to choose whichever method he preferred because he was the expert. The surgeon selected the injection method and carefully injected the patient's eye with the amount of medicine recommended by the medical literature and known to the surgeon from past experience. Unfortunately for the patient, the attempt to restore his vision to normal failed, and, in fact, he suffered a complete loss of vision as a result of the injection. Afterward, the patient learned of the different risk factors of the two methods. The patient sued the surgeon for his loss of vision. At trial, the above facts were established. The patient testified that he would have chosen the surgery had he known of the varying risk factors. At the close of the patient's case, the surgeon moved for a directed verdict. How should the court rule on the surgeon's motion? (a) Deny the motion, because the jury could find that a reasonable person would not have consented to the injection procedure if informed of the risks.(b) Deny the motion, because the patient would not have undergone the injection procedure had he known of the risks. (c) Grant the motion, because the facts establish that the patient consented to the surgeon's selection of the procedure to use. (d) Grant the motion, because there are no facts to indicate that the surgeon performed the procedure negligently.

ANSWER: (A) Deny the motion, because the jury could find that a reasonable person would not have consented to the injection procedure if informed of the risks.

(Criminal Procedure) 39. A police officer outside a mall heard gunshots coming from the indoor dining area. He rushed into the dining area and saw a woman standing there with an assault rifle lying on the ground beside her. Several patrons were scattered prostrate about the dining area, some obviously suffering from gunshot wounds. The officer said, "What the hell happened here?" The woman replied in a dazed manner, "I just wiped out the lunch crowd." The woman was subsequently charged with several murders and attempted murders. At trial, testimony of the surviving customer was vague and conflicting. The prosecution offers the testimony of the police officer, who will relate what he observed, including the woman's statement. Counsel for the woman objects to admission of any statements made by her to the police officer. How should the court rule?

ANSWER: (A) For the state, because the woman was not in custody when she made the statement to the officer.

(Criminal Procedure) 20. At 3 a.m. on the state turnpike, a driver was stopped for driving 30 m.p.h. over the posted speed limit. Because of the speed at which she was traveling and because the driver possessed an out-of-state driver's license, the officer decided to place the driver under arrest and take her to the station house. Under state law, such an arrest is valid. After placing the driver in his patrol car, the officer searched the passenger area of the automobile. Under the front seat he found a small package containing what he immediately determined to be marijuana. If the driver is charged with possession of marijuana, how should the court rule on her motion to suppress the marijuana? (A) Grant it, because the officer had already placed the driver in the patrol car. (B) Grant it, because the search was limited to the passenger compartment of the car. (C) Deny it, because the search was incident to a valid arrest. (D) Deny it, if the court determines the officer had a reasonable fear of the driver.

ANSWER: (A) Grant it, because the officer had already placed the driver in the patrol car. The motion to suppress will be granted because the officer had already placed the driver in the patrol car when he searched the car. A search incident to arrest may not include the passenger compartment of a vehicle from which the person was arrested unless (i) the person has not been secured or (ii) the arresting officer has reason to believe that the vehicle contains evidence of the crime for which the arrest was made. [Arizona v. Gant (2009)] Here, the officer was able to secure the driver and the arrest was for speeding, so neither justification applies. (B) is incorrect because the motion should be granted because the search did not fit into either prong of a search incident to arrest. The fact that the search was limited to the passenger compartment does not, in itself, make the search invalid. (C) is incorrect because it is too broad. The passenger compartment of a vehicle cannot be searched incident to arrest unless one of the two criteria stated above are met. (D) is incorrect because "reasonable fear" is not a ground for searching a vehicle. A vehicle may be searched incident to arrest under the circumstances detailed above. It may also be searched under the automobile exception if the police have probable cause to believe that the vehicle contains seizeable evidence. Finally, a vehicle can be searched without a warrant pursuant to an investigative stop (i.e., a Terry stop) if the police have reason to believe that the person stopped is armed and dangerous. But simple "fear" is not enough.

(Torts) 1. The owner of a self-propelled riding mower had just started the engine to mow his front lawn when the clutch of the mower suddenly engaged, causing it to lurch forward rapidly and throw him off. By the time the owner caught up with the mower, it had started into the street. A motorist swerved to avoid the mower and struck a tree on the opposite side of the street. An investigation revealed that the sudden shift of the clutch was caused by a defective gear in the transmission. If the motorist brings a negligence action for personal injuries and property damage against the owner, will she prevail? (A) No, because the owner was so startled by the mower's sudden movement that he was unable to react swiftly enough to prevent the harm. (B) No, because the sudden movement was caused by a defective gear in the mower. (C) Yes, because her damages were caused by the owner's operation of a dangerously defective piece of machinery. (D) Yes, because a landowner owes a duty to passersby to exercise reasonable care in activities on his land.

ANSWER: (A) No, because the owner was so startled by the mower's sudden movement that he was unable to react swiftly enough to prevent the harm. The motorist will not prevail under these facts because the owner does not appear to have breached his duty of care to her. A prima facie case for negligence consists of: (i) a duty on the part of the defendant to conform to a specific standard of conduct for the protection of the plaintiff against an unreasonable risk of injury; (ii) breach of that duty by the defendant; (iii) such breach being the actual and proximate cause of the plaintiff's injury; and (iv) damage to the plaintiff's person or property. While the owner owed a duty of care to the motorist because she was passing by his property while he was operating his riding mower, the facts do not suggest a breach of duty. He was thrown off by the sudden forward movement of the mower and could not catch up to it before it entered the street. Absent the owner's breach of duty, the motorist cannot recover. (B) is incorrect because the owner may be found liable for negligence in his operation of the mower regardless of what caused the sudden movement. Here, however, nothing suggests that he was negligent in starting the mower or failing to discover or guard against the defect. (C) is incorrect because the motorist must prove that the owner knew or should have known of the defect and was therefore negligent in operating the mower while it was in such a defective condition. Merely showing the defective condition of the mower, without more, will not suffice to impose liability on the owner, and nothing in the facts indicates that the owner knew of the defect. (D) is incorrect even though a landowner does owe a duty of reasonable care to passersby, such as the motorist. As discussed above, nothing in the facts indicates that the owner breached that duty.

(Property) 11. A businessman entered into a contract to sell his office complex to a purchaser for $1 million. The purchaser paid the businessman $100,000 in earnest money. The day before the date set for closing, the purchaser died intestate, leaving her niece as her only heir. The niece showed up at the closing with a certified check for $900,000. Which of the following is correct? (A) The niece may specifically enforce the agreement. (B) The businessman may return the $100,000 down payment and cancel the contract. (C) Death terminates the agreement. (D) Any title acquired would be unmarketable by reason of the purchaser's death.

ANSWER: (A) The niece may specifically enforce the agreement. Under the doctrine of equitable conversion, if the buyer dies, the takers of her real property can demand a conveyance of the land at the closing of the contract. (B) is wrong because the death of either the seller or the buyer does not render the agreement cancellable at the will of either party. (C) is wrong because a real estate contract survives the death of either party unless the agreement itself provides otherwise. (D) is wrong because marketable title is title reasonably free from doubt. Generally, this involves either defects in the chain of title or encumbrances that might present an unreasonable risk of litigation. Such problems are not present in these facts.

(Criminal Procedure) 4. In response to a telephone call, a report went out over the police radio requesting an ambulance because of a shooting. Several officers who heard the broadcast arrived at the subject residence and knocked on the front door, identifying themselves as police officers. Believing that someone inside the house had been shot, one of the officers entered the building through an unlocked window and opened the front door for the other officers to enter. While searching for victims, the officers saw, in plain view, several containers of cocaine. An occupant of the house was arrested and charged with possession of narcotics. At his trial, the occupant moved to suppress the evidence of the narcotics on the ground that the police officers' warrantless entry into his residence was improper. Did the police officers have a right to enter the building without a warrant? (A) Yes, because exigent circumstances existed as a result of the shooting. (B) Yes, because the police officers were acting in their official capacity. (C) No, because there were no factors to indicate that there was not enough time to obtain a warrant. (D) No, because there was no probable cause to enter without a warrant.

ANSWER: (A) Yes, because exigent circumstances existed as a result of the shooting. Exigent circumstances: Police in hot pursuit of a fleeing felon may make a warrantless search and seizure and may even pursue the suspect into a private dwelling. Police may also seize without a warrant when evidence is likely to disappear before a warrant can be obtained (evanescent evidence). Police may also enter premises without a warrant to address emergencies that could affect health or safety.

27. A manufacturing company was in the business of making copper tubing. A retail seller telephoned the manufacturing company's sales department and placed an order, which the manufacturing company agreed to fulfill. The order was for 10,000 linear feet of copper tubing at a sale price of $2 per foot. The tubing was to be used in the production of a custom order for one of the retail seller's customers. The manufacturing company installed special equipment for the manufacture of the tubing to the retail seller's specifications and had completed a portion of the order when the retail seller again telephoned the sales department. This time, however, the retail seller canceled its order, saying it no longer had need of the tubing because its customer went bankrupt and refused to pay for the order. If the manufacturing company sues for breach, will it win? (A) Yes, because the contract is fully enforceable. (B) Yes, because the contract is enforceable to the extent of the portion of the order completed. (C) No, because a contract for the sale of goods over $500 must be in writing. (D) No, because the parol evidence rule would preclude testimony about the initial telephone call.

ANSWER: (A) Yes, because the contract is fully enforceable. The manufacturing company will win because the contract is fully enforceable under the UCC. Tubing is a good, so Article 2 of the UCC applies. The contract is for the sale of goods over $500 (10,000 linear feet at $2/foot), so ordinarily section 2-201 would require a writing. However, section 2-201(3) provides that a writing is not required where the contract is for "specially manufactured" goods not suitable for resale in the ordinary course of the seller's business and the seller has made a substantial beginning of their manufacture or commitments for their procurement. Because the tubing is a custom order of unique specifications and the manufacturing company has begun manufacturing it, this exception to the UCC Statute of Frauds applies. (B) is incorrect because the contract is fully enforceable. While the manufacturing company is required to mitigate damages, it is entitled to the full range of contract remedies to put it in the position it would have been in had the retail seller not breached (i.e., benefit of the bargain damages on the entire contract). (C) is incorrect because while it is true that a contract for the sale of goods over $500 must ordinarily be evidenced by a signed writing, the "specially manufactured goods" exception (see above) applies here. (D) is incorrect because the parol evidence rule bars admissibility of evidence that varies an integrated writing; here, there is no writing at all.

(Criminal Procedure) 40. An officer went to an apartment to execute a properly obtained search warrant during an investigation of an operation making counterfeit watches. When he arrived he shouted, "Police, open up," but he did not wait before entering the apartment through the unlocked front door. The officer found the defendant half-asleep in a back room, with a workbench for assembling counterfeit watches nearby. Along with some completed counterfeit watches, the officer found a toolset used for making watches and receipts for various watch components in a drawer of the workbench, all of which the officer seized as evidence. The defendant was charged with illegal counterfeiting. Prior to trial, his attorney moves to suppress the evidence obtained during the search. Should the court grant his motion?

ANSWER: (B) No, because the exclusionary rule does not apply to the officer's Fourth Amendment violation.

(Criminal Procedure) 19. The police suspected that a young man who had been convicted of burglary and was out of prison on parole had stolen a rare diamond that was on display at a local museum. They went to the young man's home, where he lived with his mother, the owner of the house. The young man was not at home, but the police asked his mother if they might enter and search the house for the diamond. The mother allowed the police to enter, and she also consented to show them the room where her son slept and kept his personal belongings. There was a locked trunk in the room, and the police asked the mother to open it for them. She told the police that her son had the only key to the trunk, which he always kept locked. She also told them that, as far as she was concerned, they could go ahead and open the trunk if they were able to do so without a key. The police pried the trunk open and found the missing diamond inside. Did this constitute a valid search? (A) No, because the police did not inform the mother that she could refuse permission to allow the search. (B) No, because the mother did not have authority to consent to the search of the trunk. (C) Yes, under the doctrine of parens patriae. (D) Yes, because the mother owned the house and thus could consent to the search of the entire premises.

ANSWER: (B) No, because the mother did not have authority to consent to the search of the trunk. Although the mother, as the owner of the house, had the authority to consent to a search of the house, she had no right to consent to the search of the trunk. A warrantless search by the police is valid if they have a voluntary consent to the search. Any person with equal right to the use or occupation of the property may consent to a search, and any evidence found may be used against the other owners or occupants. The search is valid as long as the police reasonably believed that the consenting party had a right to use or occupy the premises, even if she in fact lacked such right. A parent generally has authority to consent to a search of a child's room (even an adult child) as long as the parent has access to the room. However, depending on the child's age, the parent may not have authority to consent to a search of locked containers in the room. Here, the mother owns the house. As the owner, she clearly had the right to the use and occupation of the house and could thus consent to the warrantless police search. However, the fact that her son kept the trunk locked, with the only key in his possession, indicates that he had the exclusive right of use and access to the trunk. Because the mother told the police that the trunk was always locked and that her son had the only key, they could not have reasonably believed that she had a right of use or access to it. Therefore, consent to search the trunk was not given by a person with a right to give such consent. Absent proper consent, the warrantless search of the trunk was invalid. (D) is incorrect because the mother's ownership of the house does not confer on her a right to use or gain access to the locked trunk. Without such a right, she could not validly consent to a search of the trunk. (A) is incorrect because knowledge of the right to withhold consent is not a prerequisite to establishing a voluntary consent (although it is a factor to be considered). Thus, the failure of the police to inform the mother that she could refuse permission for the search does not automatically invalidate her consent (and the subsequent search). (C) is incorrect. Parens patriae is a doctrine that allows the government to take the role of a parent or guardian of someone who is under a disability (such as minority). As the mother was actually the suspect's mother, the doctrine has no application here.

(Criminal Procedure) 34. While on routine patrol late one night, a police officer noticed that a car was weaving recklessly across several lanes of traffic. He stopped the driver, believing that he was driving while intoxicated. By state law, the officer was empowered to arrest the driver and take him to the nearest police station for booking. As he approached the vehicle, the officer saw the driver put what appeared to be a bottle in the glove compartment. The officer arrested the driver and then searched his vehicle. In the glove compartment, the officer discovered a vial containing a small amount of cocaine. The driver was charged with possession of cocaine. At a preliminary hearing, the driver's attorney moves to prevent introduction of the cocaine into evidence on the grounds that the search violated his client's federal constitutional rights. Will this motion most likely be granted?

ANSWER: (B) No, because the search was incident to a constitutionally valid custodial arrest.

(Property) 19. A chef purchased a restaurant for $100,000. As part of his financing, he obtained a purchase money mortgage from a bank for $60,000. Due to a clerical error by the bank, the mortgage was not recorded in the county recorder's office. A statute in the jurisdiction provides: "No conveyance of an interest in land, other than a lease for less than one year, shall be valid against any subsequent purchaser for value, without notice thereof, whose conveyance is first recorded." (RACE-NOTICE) After the chef's restaurant had been in operation for five years, business dropped dramatically. To stay in business, the chef obtained a mortgage from a financing company for $30,000. The financing company was not informed by the chef of the mortgage held by the bank. The next day, the chef contacted the bank about renegotiating its mortgage. Checking its records, the bank discovered that the original mortgage was not recorded and immediately recorded it. Later that day, the financing company recorded its mortgage. A few days later, the chef and the bank agreed to a modification of their mortgage agreement to allow the chef to make lower monthly payments in exchange for a higher interest rate and a longer period of repayment. Despite this agreement, the chef was unable to make payments on the financing company mortgage. The financing company instituted a foreclosure action six months later, but failed to include the bank as a party to the foreclosure action. If the financing company takes title to the restaurant at the foreclosure sale, which of the following statements most correctly describes the bank's interest? (A) The bank's mortgage on the restaurant survives under its original terms. (B) The bank's mortgage on the restaurant survives under its modified terms. (C) The bank's mortgage is extinguished because when it was modified it became junior to the financing company's mortgage. (D) The bank's mortgage is extinguished regardless of the modification because it had not recorded before the financing company obtained its mortgage interest.

ANSWER: (B) The bank's mortgage on the restaurant survives under its modified terms. The bank's mortgage survives under its modified terms because the financing company did not include the bank in the foreclosure action. The general rule is that when a mortgage is foreclosed, the buyer at the sale will take title as it existed when the mortgage was placed on the property. Thus, foreclosure generally will destroy all interests junior to the mortgage being foreclosed, but will not discharge senior interests. However, those with interests subordinate to those of the foreclosing party are necessary parties to the foreclosure action. Failure to include a necessary party results in the preservation of that party's interest despite foreclosure and sale. Here, the bank's original mortgage was senior to the financing company's mortgage. However, where the landowner enters into a modification agreement with the senior mortgagee, raising its interest rate or otherwise making it more burdensome, the junior mortgagee will be given priority over the modification. Thus, the bank's modification would not have priority over the financing company's mortgage. Nevertheless, because the financing company failed to include the bank in its foreclosure action, the bank's mortgage interest survives under its modified terms, even though the modification did not have priority. (B) is therefore correct and (A) is incorrect. (C) is incorrect because the modification of a senior mortgage does not nullify its original senior status; it only means that the junior mortgage will be given priority over the modification. Because the buyer at the foreclosure sale ordinarily will take title as it existed when the mortgage was placed on the property, the senior mortgage ordinarily survives in its original form. (As noted above, here the mortgage survives in its modified form despite its junior status because of the failure to include the bank in the foreclosure action.) (D) is incorrect because the recording statute applicable here is a race-notice statute rather than a notice statute. Under a race-notice statute, a subsequent bona fide purchaser (including a mortgagee) is protected only if it records before the prior grantee or mortgagee. Here, the bank recorded its mortgage before the financing company recorded its mortgage. The fact that the financing company had no notice of the bank's interest at the time it granted the mortgage on the restaurant does not help the financing company because it did not record first.

(Criminal Procedure) 38. Late one night, a young couple were killed instantly when their car was struck by a speeding truck as the couple's car crossed an intersection with the light green in their favor. Several weeks later, a burglar awaiting trial on burglary charges asked a jail officer to let him speak with a highway patrol officer. When the highway patrol officer came to the cell, the burglar told him that he was the driver of the truck that had struck the car and had been speeding away from a burglary when the accident occurred. The burglar was charged with felony murder, on the theory that he had not yet reached a place of temporary safety when the accident occurred. At trial, the prosecution seeks to introduce the burglar's statements to the highway patrol officer regarding the events of the night of the accident. The burglar's attorney objects. Which of the following is the strongest argument for permitting the statements into evidence? (A) The burglar had not been charged in connection with the auto accident at the time the statements were made to the highway patrol officer. (B) The burglar made the statements spontaneously, without inducement or interrogation by the police. (C) The highway patrol officer had no connection with the burglary investigation for which the burglar had been incarcerated. (D) The burglar's statements were not the product of coercion by the police officers.

ANSWER: (B) The burglar made the statements spontaneously, without inducement or interrogation by the police. The best argument is that the burglar made the statements spontaneously. Prior to a suspect's being charged with a crime, the Fifth Amendment privilege against compelled self-incrimination is the usual basis for ruling on the admissibility of a confession. [Miranda v. Arizona (1966)] Under Miranda, statements made during custodial interrogations are inadmissible unless the defendant is first warned of his right to remain silent and his right to an attorney. Thus, Miranda applies only when the defendant is in custody and only when the defendant's statements are the result of interrogation. Although almost any words or actions on the part of the police that they should know are reasonably likely to elicit an incriminating response qualify as interrogation, Miranda does not apply to spontaneous statements not made in response to interrogation. Here, the police did nothing to solicit the statement from the burglar; it was spontaneous. Thus, (B) is correct. (A) is incorrect because the defendant need not yet be charged for Miranda rights to apply as long as he is in custody (i.e., not free to leave). Being in jail on another charge (as the burglar was) satisfies the custody requirement. (C) is incorrect because the fact that the officer who took the burglar's admission had nothing to do with the investigation of the burglary does not alter the rules of Miranda—questioning that is totally unrelated to the matter for which the accused is in custody may still violate the accused's Miranda rights. (D) is incorrect. Due process requires that a confession be voluntary (i.e., not the product of police coercion). The Miranda rule, however, goes beyond voluntariness. It makes inadmissible all statements obtained without Miranda warnings or without a valid waiver of Miranda rights, not just statements actually coerced by the police.

(Property) 15. A woman and her friend lived together in the woman's home for 20 years. Subsequently, the woman became disabled because of a heart ailment, and the friend took care of her. The woman told the friend that she wanted to be sure that the friend got her house after she died, so she gave the friend a quitclaim deed. The friend did not record the deed but put it in his safe deposit box. Four months later, the woman's son found out about this and told his mother that if she would sell the house to him, she could live there for the rest of her life. The woman, who wanted the money, agreed and carried out the transaction. She told the friend that she had changed her mind and decided to leave the home to her children. The friend promised to destroy the deed, and the next day, he did. Several days later, however, as the friend and the woman were driving to the store, their car was hit by a train and they both died. The woman's and the friend's heirs claim title to the house. The friend's heirs bring an appropriate action to resolve the dispute. Which party will the court likely find owned the house? (A) The friend, because the woman did not tell him the truth about why she was revoking her agreement. (B) The friend, because he did not retransfer title to the woman. (C) The woman, because the friend agreed to return the title and did, in fact, destroy the deed. (D) The woman, because the friend, as a donee, would not be able to prevail against the son, who was a bona fide purchaser.

ANSWER: (B) The friend, because he did not retransfer title to the woman. The court will most likely find that the friend was still the owner of the house because he did not retransfer title to the woman. The deed, once delivered, merely evidences title to the property, and its destruction has no effect on the title. Thus, the friend was still the owner, and (C) is incorrect. (A) is incorrect because the woman cannot affect the ownership of the property after she delivered the deed to the friend, regardless of the truth or falsity of her subsequent reasons for the attempted revocation. (D) is incorrect because the son was not a bona fide purchaser: He was aware of the friend's deed. Also, it is only the subsequent bona fide purchaser, a purchaser for value and without notice, who gets the protection of the recording act. The friend was the first grantee and, thus, his status as a donee would be irrelevant to the applicability of the recording act.

(Property) 4. To satisfy a debt owing to her friend, a granddaughter executed and delivered to the friend a warranty deed to a vacant tract of land. The friend promptly and properly recorded his deed. Shortly thereafter, the friend built a house on the property and has lived there since. The granddaughter did not actually own the vacant tract of land at the time. Her grandfather was the record title holder, but he had promised to leave the land to her. Later, the grandfather died and his will devised the land to the granddaughter. Pressed for money, the granddaughter sold the land to a creditor by warranty deed, which it promptly and properly recorded. Although the creditor paid full value for the land, it did not conduct a title search of the land before accepting the deed. Both the friend and the creditor now claim ownership of the land. The jurisdiction in which the land is located has the following statute: "No unrecorded conveyance or mortgage of real property shall be good against subsequent purchasers for value without notice unless the conveyance is recorded." (NOTICE) Who has title to the land?

ANSWER: (B) The friend, because the creditor was not a purchaser for value without notice of the friend's interest.

17. A small parish hired a contractor to build a school wing onto its church for a cost of $200,000 by August 31. The parties entered into a written contract that provided for five progress payments of $40,000 each at various stages of completion. On July 18, after the contractor had spent $160,000 on performance and received $120,000 in progress payments, he notified the parish that he was quitting the project because a more lucrative job came up. The parish hired some local laborers who did construction work for a living to finish the job by August 31 for $120,000, which was a reasonable price given the short notice. If the parish sues the contractor for breach of contract, and the contractor countersues to recover the $40,000 in costs that he has not been paid, what is the likely award? (SIMILAR Q'S QUESTION: Which of the following statements regarding the parties' remedies is correct?) (A) The parish can recover $120,000, the cost to complete construction of the wing. (OR another A answer choice: A. The parish can recover $80,000, the difference between the contract price and the total amount that was expended in construction of the wing.) (B) The parish can recover $40,000, the difference between the contract price and the total amount that the parish paid for construction of the wing. (C) The contractor can recover $40,000, the difference between the amount it expended on performance and the amount it was paid, to prevent the parish's unjust enrichment. (D) Neither party can recover anything, because the $40,000 extra that the parish had to pay to complete construction of the wing is offset by the $40,000 difference between the contractor's expenditures and the payments that the parish made to the contractor.

ANSWER: (B) The parish can recover $40,000, the difference between the contract price and the total amount that the parish paid for construction of the wing. The parish can recover $40,000, which is the amount above the contract price that it cost the parish to get the wing completed. In construction contracts, when the builder breaches after partially performing, the standard measure of damages to which the owner is entitled is the cost of completion plus reasonable compensation for any delay in performance (unless completion would involve undue economic waste). In addition, most courts allow the builder to offset or recover for work performed to date if necessary to avoid the unjust enrichment of the owner. Here, the parish paid the contractor $120,000 and had to pay the laborers an additional $120,000. Thus, the total cost of the wing was $240,000, which is $40,000 more than the contract price. No offset will be allowed for the reasons discussed in (C) and (D) below. Hence, the parish can recover $40,000. (A) is incorrect because the cost of completion is determined from the perspective of the owner, i.e., how much additional money did the owner have to pay to above the contract price to have construction completed? Here, the owner (the parish) paid $40,000 more to have the wing completed than it would have paid had the contract been performed to completion. The parish would be unjustly enriched if it could recover another $80,000 beyond the damages it suffered. (C) and (D) are incorrect because the parish was not unjustly enriched by the additional amount that the contractor expended in performance over the progress payments that it received. The parish still had to pay $40,000 more than the contract amount for completion of the wing because of the contractor's breach. Thus, the parish received no windfall as a result of the contractor's additional expenditures.

(Torts) 6. Some teenage boys on spring break were canoeing on a river when their canoe overturned. A nearby fisherman overheard the teens' cries for help and immediately dove into the river to rescue them. The fisherman hit his head on some hidden rocks and suffered a head injury. If the fisherman sues the teens to recover damages and prevails, what is the likely reason?

ANSWER: (B) The teens put themselves in danger by not paying attention to the rocks and currents in the river.

(Property) 16. A landlord leased an office building to a tenant for 10 years. The tenant, a second-year law student, was familiar with the state's recording act, which provided: No conveyance is valid against any subsequent purchaser for value without notice unless the conveyance is recorded. No lease for three years or more is valid against a subsequent purchaser for value without notice unless the lease has been recorded. (NOTICE) Believing it would be obvious to any prospective purchaser that the tenant was in possession of the property, she failed to record the lease. Shortly thereafter, the landlord entered into a contract to sell the leased property to a buyer. Before purchasing the property, the buyer merely drove by it, and thus did not notice the tenant's occupancy. The standard title search did not reveal the lease because it was unrecorded. The buyer tendered the purchase money to the landlord, and the landlord conveyed to the buyer the property by warranty deed. The buyer subsequently found the tenant in possession of the premises and ordered her to vacate. The tenant refused and asked the buyer where she should send the rent checks. In an action by the buyer to evict the tenant, how should the court rule? (A) The tenant wins, because the buyer's drive-by inspection will be deemed to confer actual notice on him. (B) The tenant wins, because the buyer had a duty to properly inspect the property. (C) The buyer wins, because the buyer is a subsequent purchaser for value and the tenant failed to record. (D) The buyer wins, because the tenant knew of the statute and willfully failed to record

ANSWER: (B) The tenant wins, because the buyer had a duty to properly inspect the property. The tenant will prevail because the buyer did not properly inspect the property. A title search is not complete without an examination of possession. If the possession is unexplained by the record, the purchaser is obligated to make inquiry. The purchaser is charged with knowledge of whatever an inspection of the property would have disclosed and anything that would have been disclosed by the possessor. Thus, the buyer is on constructive notice of the tenant's possession and anything that would have been disclosed by inquiring of the tenant. Because the buyer had this notice, he is not protected by the notice recording statute, and he will take subject to the tenant's lease. (A) is wrong because "actual notice" means that the buyer was aware of the tenant's possession. The facts make clear that the buyer did not know of the tenant's possession. Furthermore, the act of driving by has no legal consequence; thus, it would not be deemed to confer anything on the buyer. (C) is wrong because the buyer had constructive notice of the tenant's possession and, therefore, is not protected by the recording statute. (D) is wrong because the tenant's state of mind is irrelevant for purposes of applying the recording statute.

7. On September 15, a manufacturer of office furniture received an email purchase-order form from a retailer of office furniture. The order was for 100 executive leather swivel chairs and specified a delivery date no later than November 1, at a total cost of $10,000, as quoted on the manufacturer's website. Two days later, the manufacturer emailed its own purchase-order acceptance form to the retailer, who was a new customer and had never seen the form before. The purchase-order acceptance form stated that it was an acceptance of the specified order, was signed by the manufacturer's sales manager, and contained all of the terms of the retailer's form, but it also contained an express warranty and a clause disclaiming all implied warranties such as the implied warranty of merchantability. (Similar question has different ending..."but it also contained a clause providing liquidated damages in the event of a breach of contract.) Assuming that there were no further communications between the parties, what is the status of the relationship between the parties? (A) There is an enforceable contract between the parties, the terms of which are comprised of the language in the manufacturer's form. (B) There is an enforceable contract between the parties, the terms of which do not include the disclaimer of implied warranties in the manufacturer's form. (C) There is no enforceable contract between the parties because the manufacturer's form constituted a rejection of the retailer's offer and a counteroffer by the manufacturer. (D) There is no enforceable contract between the parties because the manufacturer's form added an additional term that materially altered the terms of the retailer's offer.

ANSWER: (B) There is an enforceable contract between the parties, the terms of which do not include the disclaimer of implied warranties in the manufacturer's form. The manufacturer and the retailer have a contract without the disclaimer. In contracts for the sale of goods, a definite expression of acceptance operates as an acceptance even if it states additional terms. Between merchants, additional terms proposed by the offeree in an acceptance automatically become part of the contract unless (i) they materially alter the original terms of the offer (e.g., they change a party's risk or the remedies available); (ii) the offer expressly limits acceptance to the terms of the offer; or (iii) the offeror had already objected to the additional terms or objects within a reasonable time. Here, a clause was added by the manufacturer (the offeree) providing for an express warranty and a disclaimer of all implied warranties, including the warranty of merchantability. The disclaimer materially altered the original terms of the offer. Therefore, the disclaimer would not become part of the contract. (A) is therefore incorrect. (C) is incorrect because it reflects the common law "mirror image" rule, which the UCC has rejected in sale of goods cases. (D) is incorrect because under the UCC, the inclusion of a material additional term does not prevent formation of a contract; instead, a contract is formed without the inclusion of that additional term.

(Property) 7. The owner in fee simple of a tract of land sold it to a farmer for $850,000. To finance the purchase, the farmer obtained a mortgage loan from a financing company for $600,000. The deed from the owner to the farmer was promptly and properly recorded, but due to an oversight the mortgage from the financing company was not immediately recorded. A few months later, the farmer approached the financing company about getting a second mortgage. The financing company turned him down, so he contacted a bank. Not having knowledge of the previous mortgage on the property, the bank agreed to loan the farmer $300,000 secured by a mortgage on the land, which it promptly and properly recorded. One day later, the financing company, having discovered that its original mortgage had not been recorded, properly recorded it. The jurisdiction's recording statute provides: "Any conveyance or mortgage of an interest in land, other than a lease for less than a year, shall not be valid against any subsequent purchaser for value, without notice thereof, whose conveyance is first recorded." (RACE-NOTICE) The farmer struggled to keep up with his mortgage payments, and finally stopped making payments altogether on both mortgages. The bank began foreclosure proceedings, but did not include the financing company as a party. At the foreclosure sale, a buyer purchased the land, having no actual knowledge of the mortgage with the financing company. Soon after, the financing company declared its loan in default and sought to foreclose on the land. May the financing company foreclose against the buyer? (C) No, because its failure to promptly record extinguished its rights against all parties except the farmer, the original mortgagor.

ANSWER: (B) Yes, The financing company may foreclose against the buyer because it was not included as a party in the foreclosure proceeding brought by the bank. Generally, the priority of a mortgage is determined by the time it was placed on the property. When a mortgage is foreclosed, the buyer at the sale will take title as it existed when the mortgage was placed on the property. Thus, foreclosure will terminate interests junior to the mortgage being foreclosed but will not affect senior interests. However, the junior mortgagee has the right to pay the senior interest off (i.e., redeem it) in order to avoid being wiped out by its foreclosure. Thus, those with interests subordinate to those of the foreclosing party are necessary parties to the foreclosure action. Failure to include a necessary party results in the preservation of that party's interest despite foreclosure and sale. Here, even though the financing company created its mortgage first, its interest was junior to the bank's interest by virtue of the recording statute. Hence, when the bank brought its foreclosure action, it should have included the financing company as a necessary party. Because the financing company was not included, its mortgage interest on the land is preserved and it may bring a foreclosure action. (A) is incorrect because the financing company did not hold the senior mortgage interest at the time of the foreclosure by the bank. Mortgagees for value such as the bank are treated as "purchasers" under recording statutes. By failing to record its mortgage before the bank executed and recorded its mortgage for value, the financing company became the junior mortgagee under the jurisdiction's race-notice recording statute. Thus, had it been made a party to the foreclosure proceeding by the bank, its interest would have been wiped out. (C) is incorrect because the financing company's delay in recording only affected its rights vis-a-vis the bank. By the time of the foreclosure sale, the financing company's mortgage interest was recorded; hence, the buyer will take subject to it because he had constructive (record) notice of it. (D) is incorrect because the fact that the buyer obtained the right to redeem the property when he purchased it does not preclude the financing company from beginning foreclosure proceedings. To redeem the property, the buyer must pay off the financing company's mortgage prior to the date set for the foreclosure sale.

(Torts) 2. A 15-year-old took the keys to the family car, which was parked on the street in front of the house, and began driving. He turned right at the first intersection, swinging a little bit wide as he turned. A motorist coming down the street from the opposite direction swerved his auto to the right because he feared a collision. He lost control of the auto and struck a tree in a landowner's front yard. As the motorist staggered out of his auto, dazed but otherwise unhurt, he tripped over some decorative boulders that the landowner had placed in his yard and fell, breaking his wrist as he landed. The motorist brought an action against the 15-year-old for personal injuries and established the above facts. At the close of the motorist's case, the attorney for the 15-year-old moves for a directed verdict. How should the court rule on the directed verdict motion?

ANSWER: (C) Deny the motion, because the jury could find that the 15-year-old has not acted as a reasonably prudent person would have acted in similar circumstances.

(Criminal Procedure) 2. The defendant robbed a bank and fled in a getaway car driven by an accomplice, not realizing that one of the bundles of money he took had the serial numbers recorded and had a tiny tracking device attached to the wrapper. The bank's security consultant obtained portable tracking equipment and was able to trace the bundle of money to the defendant's house. The police were notified and they arrived at the defendant's house a few hours after the robbery. They knocked on the door, announced their presence, and saw someone matching the description of the robber in the hallway. They entered and arrested the defendant, and then conducted a protective sweep of the house for the accomplice, who they believed had a gun. They did not find him, but while checking a closet, they discovered several of the bundles of money from the bank and a gun the defendant had used in the robbery. The police also discovered two clear plastic bags of what appeared to be marijuana sitting on top of a dresser. They seized the money, the gun, and the two bags. Later testing confirmed that the substance in the bags was marijuana. The defendant was charged with the bank robbery and with possession of the marijuana. At a preliminary hearing, he moves to suppress introduction of the money, gun, and marijuana. How should the court rule? (A) Grant the motion as to the marijuana but not as to the money or the gun because the money and gun were found as a result of the protective sweep for the defendant's accomplice. (B) Grant the motion as to the money and the gun but not as to the marijuana because the bags containing the marijuana were clearly visible on the dresser during the search. (C) Grant the motion as to all of the evidence seized. (D) Deny the motion as to all of the evidence seized.

ANSWER: (C) Grant the motion as to all of the evidence seized. The court should suppress all of the evidence because it was the fruit of an unconstitutional arrest. As a general rule, the police must have an arrest warrant to effect an arrest of an individual in his own home. There is no general "emergency" exception to the warrant requirement. While police officers in hot pursuit of a fleeing felon or trying to prevent the destruction of evidence may sometimes make a warrantless search and seizure, the burden is on the government to show that one of those exceptions applies. Here, the police did not arrive at the defendant's house in hot pursuit of the defendant, and there was no indication that the defendant might be destroying the money or other evidence; i.e., there were no circumstances precluding them from keeping the house under surveillance while they obtained a warrant. Hence, the arrest was unconstitutional. Because an arrest constitutes a seizure under the Fourth Amendment, the exclusionary rule applies, and evidence that is the fruit of the unconstitutional arrest may not be used against the defendant at trial. Here, all of the evidence was seized without a warrant, and none of the other exceptions to the warrant requirement are applicable. While the protective sweep that turned up the money and gun probably would have been within the bounds of a search incident to an arrest because the police had reason to believe an armed accomplice was present, the arrest in violation of the Fourth Amendment makes the search unlawful. Similarly, while the bags of marijuana were discovered in plain view, the police have to be legitimately on the premises for that exception to apply. Thus, (C) is correct; (A), (B), and (D) are incorrect.

(Criminal Procedure) 21. Suspecting criminal activity, a police officer acting without a warrant peeked through a small opening in the shutters of an apartment. The officer observed the apartment's tenant and the defendant making methamphetamine. The officer immediately entered the apartment and arrested the tenant and the defendant, and he confiscated the ingredients for the methamphetamine, the tools used for methamphetamine production, and any completed methamphetamine for evidence. The search is later ruled invalid at a suppression hearing instituted by the tenant. May the defendant now claim that her Fourth Amendment rights have been violated by the seizure of the ingredients, tools, and methamphetamine from the apartment? (A) Yes, because the items will be used in evidence against her. (B) Yes, because the search was ruled invalid at a suppression hearing. (C) No, because she was not the owner or occupier of the apartment. (D) No, because the officer discovered the contraband in plain view.

ANSWER: (C) No, because she was not the owner or occupier of the apartment. The defendant cannot claim a reasonable expectation of privacy for Fourth Amendment purposes because she was not an owner, occupier, or overnight guest of the place searched. To raise a Fourth Amendment claim of an unreasonable search or seizure, a person must have a reasonable expectation of privacy with respect to the place searched or the item seized. It is not enough merely that someone has an expectation of privacy in the place searched. The Supreme Court has imposed a standing requirement so that a person can complain about an evidentiary search or seizure only if it violates her own reasonable expectations of privacy. The Court has held that a person has a reasonable expectation of privacy any time (i) she owned or had a right to possession of the place searched, (ii) the place searched was in fact her own home, whether or not she owned or had a right to possession of it, or (iii) she was an overnight guest of the owner of the place searched. Thus, because there are no facts that state the defendant is the owner of the apartment, had a right to possession of it, or was an overnight guest, the defendant would not have standing to challenge the search of the tenant's apartment. (A) is incorrect because standing to raise a Fourth Amendment claim does not exist merely because a person will be harmed by introduction of evidence seized during an illegal search of a third person's property. The person must establish that her own reasonable expectation of privacy has been violated. (B) is wrong because the results of the suppression hearing apply only to the tenant. The defendant must establish standing in her own right to challenge the search. (D) is incorrect. Under the plain view exception, officers may make a warrantless seizure when they are: (i) legitimately on the premises, (ii) see evidence, fruits or instrumentalities of a crime, or contraband in plain view, and (iii) have probable cause to believe that the item is evidence, contraband, or a fruit or instrumentality of a crime. Although it is unlikely that the officer was legitimately on the tenant's premises, the issue here is not whether the search was unconstitutional. The question is whether the defendant has standing to object to the search, and (D) does not address that issue.

(Criminal Procedure) 13. Police officers were executing a search warrant at a home suspected of containing evidence of illegal gambling. No one was at home when the police arrived. After searching the first floor, the officers went upstairs. A friend of the owner then entered the house carrying a briefcase. He set the briefcase on the floor, opened it, and then heard the officers. He became frightened, left the briefcase sitting in the middle of the floor, and hid in a closet. The police officers returned to the first floor and immediately spotted the briefcase, which they knew was not there earlier. Because the briefcase was open, the officers saw its contents—betting slips—and seized them. Because they knew that someone had entered the house since they arrived, they re-searched the first floor. They found the friend and informed him that he was under arrest, clapped handcuffs on him, and read him his Miranda warnings. One of the officers patted the friend down to check for weapons. The officer noticed a bulge in the friend's pocket. Although the officer knew that the bulge was unlikely to be a weapon, he reached into the pocket anyway, and discovered a package that appeared to be (and later proved to be) heroin. The friend was charged with possession of narcotics. At a suppression hearing, will the court agree with the public defender's contention that the friend's arrest was illegal? (A) Yes, because the police officer who searched the friend knew that he did not have a weapon in his pocket. (B) Yes, because the friend's mere presence in the house did not give the police probable cause to believe he had committed a crime, and they had no basis for searching him at all, because he did not act toward them in a threatening manner. (C) No, because the contents of the briefcase gave the police probable cause to arrest the man. (D) No, because the police had a right to search the friend for gambling slips, and the discovery of the heroin was merely incidental to a lawful search.

ANSWER: (C) No, because the contents of the briefcase gave the police probable cause to arrest the man. A police officer may conduct a warrantless search of a person incident to the person's lawful arrest. Any arrest is sufficient, as long as it was lawful (e.g., reasonable and based on probable cause). The contents of the briefcase supplied probable cause to believe that the friend was involved in the gambling operation, and thus his arrest was constitutional. The police had searched the house for gambling paraphernalia pursuant to a search warrant. Upon seeing such paraphernalia in the briefcase, which was not previously present, they had reasonable grounds to believe that the person who left the briefcase was involved in the gambling operation. When the officers found the friend, who had not been present during the initial search, they had reasonable grounds to believe that he had left the briefcase and was therefore involved in the commission of gambling offenses. Thus, the friend's arrest was constitutional/lawful. (A) focuses on the propriety of the search that uncovered the heroin, rather than on the validity of the arrest itself. Do not be sidetracked. The call of the question concerns the lawfulness of the arrest. As has been explained above, the friend's arrest was constitutional and based on grounds entirely independent of the legality of the subsequent search and seizure. Moreover, because the arrest was lawful, it does not matter whether the officer thought the bulge was a weapon. This alludes to whether a valid warrantless frisk was performed. A police officer may frisk a person for weapons without a warrant if the officer has reason to believe the suspect is armed and dangerous. But here, the friend had been placed under arrest. Incident to arrest, a person may be thoroughly searched for weapons or any type of evidence. Thus, (A) is incorrect. Although it is true that, as (B) states, the friend's mere presence in the house did not give probable cause to believe he had committed a crime, his arrest was not based on his mere presence. As has been explained, reasonable grounds to believe that the friend was part of the gambling operation arose from the presence of betting slips in the briefcase and the great likelihood that the friend was the person who brought the briefcase into the house. Thus, (B) incorrectly states the basis for the friend's arrest. Also, (B) incorrectly states that there was no basis to search the friend, because he had not behaved threateningly. In fact, the police may conduct a search incident to a constitutional arrest without actually fearing for their safety. Note also that this second part of (B), similarly to (A), incorrectly focuses on the search of the friend, rather than on the arrest itself. (D) incorrectly asserts a right to search the man independent of any probable cause to arrest him. A search warrant does not authorize the police to search persons found on the premises who are not named in the warrant. However, if the police have probable cause to arrest a person discovered on the premises, they may search him incident to the arrest. Consequently, any right that the police had to search the friend arose from their arrest of him, which was based on probable cause. (D) ignores the necessity of probable cause to arrest. Of course, (D) also attempts the same distraction as (A) and (B); i.e., it focuses on the search of the friend as a means of either justifying or attacking the arrest, rather than on the grounds for the arrest itself.

(Criminal Procedure) 11. A man on parole after being convicted of possession of cocaine was suspected of selling cocaine out of his home. His parole officer came to his house and rang the bell. As soon as the man opened the door to see who was there, the officer entered the home, despite the man's protests. After searching the home, the parole officer discovered several bags of marijuana in a drawer. The man was arrested and charged with possession of marijuana with intent to sell. A statute in the jurisdiction in which the search took place provides that, as a condition of parole, a parolee is on notice that his parole officer may conduct a search of the parolee's person or home, without probable cause, at any time of the day or night. The man moved to have evidence of the marijuana suppressed by the court, claiming that the state statute that authorized the search was unconstitutional under the Fourth Amendment prohibition of unreasonable searches and seizures. Will he prevail? (A) Yes, unless probable cause was established by the officer's tip in conjunction with other circumstances. (B) Yes, because a search warrant was not obtained and no exception to the warrant requirement applies. (C) No, because the man had a diminished expectation of privacy and the government has a heightened need to search probationer's homes. (D) No, because the search was incident to a lawful arrest.

ANSWER: (C) No, because the man had a diminished expectation of privacy and the government has a heightened need to search probationer's homes. The man will not prevail in his motion to suppress. To be reasonable under the Fourth Amendment, most searches must be pursuant to a warrant. However, several types of inspections and searches do not require a warrant or even probable cause. The Supreme Court has held that the Fourth Amendment is not violated by a statute authorizing warrantless searches of a parolee's home—even absent probable cause—if a statute provides for such searches. The Court reasoned that in such circumstances, the parolee has a diminished expectation of privacy and the government has a heightened need for searching parolees; thus the search is reasonable in a constitutional sense.

(Criminal Procedure) 28. Acting with probable cause, the police arrested a man in connection with the armed robbery of a liquor store. After being given Miranda warnings, the man confessed to the robbery but denied his involvement with several other recent armed robberies of businesses in the area. He was formally charged with the one robbery and put into a cell with a paid informant working undercover for the police. The informant had been instructed to find out what he could about the other robberies but not to ask any questions. The informant began talking about a convenience store robbery in which a bystander was shot and seriously injured by the robber, and he deliberately misstated how it happened. The man, unaware that his cellmate was an informant, interrupted to correct him, bragging that he knew what really happened because he was there, and proceeded to make incriminating statements about the robbery. The man was subsequently charged with armed robbery and attempted murder in the convenience store robbery. At a motion-to-suppress hearing on that charge, if the man's attorney moves to exclude the statements made to the informant, should the motion be granted? (A) Yes, because the informant deliberately elicited incriminating statements in violation of the man's Sixth Amendment right to counsel. (B) Yes, because the informant's conduct constituted custodial interrogation in violation of the man's Fifth Amendment privilege against self-incrimination. (C) No, because the man had not yet been charged with the robbery of the convenience store when he made the statements to the informant. (D) No, because the informant's conduct did not constitute interrogation.

ANSWER: (C) No, because the man had not yet been charged with the robbery of the convenience store when he made the statements to the informant. The man's motion should be denied because neither his Fifth nor Sixth Amendment rights were violated by the informant's conduct. The Sixth Amendment right to counsel applies to all critical stages of a criminal prosecution after formal proceedings have begun, but does not apply in precharge custodial interrogations. Because this right is "offense specific," the fact that the right to counsel has attached for one charge does not bar questioning without counsel for an unrelated charge. Because the man has not been charged with the convenience store robbery, his Sixth Amendment right to counsel has not been violated. The Fifth Amendment privilege against self-incrimination requires Miranda warnings and a valid waiver before any statement made by the accused during custodial interrogation can be admitted. However, this requirement does not apply where interrogation is by an informant who the defendant does not know is working for the police, because the coercive atmosphere of police-dominated interrogation is not present. [Illinois v. Perkins (1990)] Because the man was not aware of the informant's status, the informant's conduct did not constitute a police interrogation. (A) is wrong despite the fact that the informant's conduct may have been deliberately designed to elicit incriminating remarks. As discussed above, the man's right to counsel did not attach for purposes of the convenience store robbery. (B) is incorrect because, as discussed above, the Miranda warnings need not be given before questioning by a cellmate working covertly for the police. (D) is incorrect because interrogation refers not only to express questioning, but also to any words or actions on the part of the police that the police should know are reasonably likely to elicit an incriminating response from the suspect. Here, the informant, working for the police, made statements about the convenience store robbery that were intended to, and reasonably likely to, prompt a response from his cellmate. Hence, it is not the absence of "interrogation" that avoids the Miranda problem, but the fact that the man did not know that his cellmate was working for the police.

(Criminal Procedure) 22. A driver was operating her car on a city street when she was stopped by a police officer for speeding. As the police officer reached the driver's car, he saw her put something into her purse. The officer told the driver, "Ma'am, you were speeding; that's why I stopped you. I'd like your driver's license, and, by the way, what did you just put into your purse?" The driver responded, "It's just a marijuana cigarette, but don't worry, I've only had two and my driving judgment hasn't been impaired." The officer took her purse, removed the "joint," and arrested the driver for possession of marijuana as well as speeding. At the driver's trial for marijuana possession, the prosecution seeks to introduce the marijuana cigarette into evidence. The driver's attorney moves to suppress the evidence. Should the defense motion be granted? (A) Yes, because the marijuana cigarette is fruit of the poisonous tree. (B) Yes, because the police officer did not have a valid search warrant. (C) No, because the police officer had probable cause to search the driver's purse. (D) No, because the marijuana cigarette was discovered incident to a lawful arrest.

ANSWER: (C) No, because the police officer had probable cause to search the driver's purse. The defense motion should be denied because the police officer had probable cause to search the driver's purse. This case falls within the automobile exception to the warrant requirement. Under that exception, if the police have probable cause to believe that a vehicle contains contraband or fruits, instrumentalities, or evidence of a crime, they may search the vehicle, including the driver's belongings, without a warrant. Here, the driver's response established probable cause to search her purse. (A) is incorrect. Persons temporarily detained for routine traffic stops are not in custody for Miranda purposes. Therefore, the driver was not entitled to Miranda warnings, and her statement about the marijuana was not tainted. Her statement thus properly provided the probable cause for the search of her purse. Furthermore, Miranda violations do not require the exclusion of nontestimonial evidence resulting from voluntary statements, so even if the driver's statement was obtained in violation of Miranda, the marijuana cigarette would likely be admissible. (B) is wrong because this case falls within the automobile exception to the warrant requirement. (D) is incorrect because the search incident to a lawful arrest exception to the warrant requirement applies when a police officer searches a person after the person has been arrested. Here, the police officer searched the driver's purse before arresting her for possession of marijuana.

(Criminal Procedure) 29. A woman was arrested, given Miranda warnings, and questioned about an armed robbery. After she asked to speak with an attorney, the police stopped questioning her about the robbery. Several hours later, the police gave the woman a fresh set of Miranda warnings and began to question her about a different robbery. She did not repeat her request for an attorney and instead made several incriminating statements about the robbery. At the woman's trial for the robbery for which she made incriminating statements, the prosecution seeks to have her statements introduced into evidence. If the woman's attorney objects on appropriate grounds, how should the court rule? (A) Overrule the objection, because the police did not badger the woman into confessing. (B) Overrule the objection, because the woman did not renew her request for an attorney after receiving fresh Miranda warnings. (C) Sustain the objection, because the police did not honor the woman's request. (D) Sustain the objection, because a confession obtained in violation of a defendant's Miranda rights but otherwise voluntary may be used against the defendant.

ANSWER: (C) Sustain the objection, because the police did not honor the woman's request. The court should sustain the objection because the police did not honor the woman's request for an attorney. At any time prior to or during a custodial interrogation, the accused may invoke a Miranda (Fifth Amendment) right to counsel. If the accused invokes this right, all questioning must cease until the accused is provided with an attorney or initiates further questioning himself. Thus, the police questioning of the woman about the robbery was improper, and she can have her statements excluded. (A) is incorrect. After receiving Miranda warnings, if an accused invokes the right to remain silent, the police cannot badger the accused. However, courts have ruled that if the police scrupulously honor the request, they can rewarn the accused and later resume questioning, at least about a different crime. Here, however, the accused did not simply invoke the right to remain silent, but rather requested an attorney. After such a request, as indicated above, all questioning must cease. (B) is incorrect because the accused does not need to reassert the right to an attorney; all questioning must stop until the accused is provided an attorney or resumes the questioning herself. (D) is incorrect. It is stating the rule for impeachment—a confession obtained in violation of a defendant's Miranda rights but otherwise voluntary may be used against the defendant for purposes of impeachment, but there is no such rule for use of the confession for other purposes.

(Property) 8. A landowner conveyed his land to a famous guitarist. The guitarist put the deed in his guitar case and took off for a three-week band tour. While the guitarist was out of town, the landowner offered to sell the same land to his neighbor for $5,000. Although the neighbor knew that the guitarist had already bought the land, the neighbor paid the landowner $5,000 and promptly recorded the deed she received. Thereafter, the neighbor conveyed the land to her ex-husband for $15,000. The ex-husband knew nothing about the guitarist's deed and promptly recorded the deed he received. Two weeks later, the guitarist returned and recorded his deed to the land. A month after that, the ex-husband conveyed the land to a buyer for $17,000. The buyer knew that the guitarist held a deed to the land, but paid the ex-husband $17,000 anyway. The buyer immediately recorded and filed an appropriate action against the guitarist and the ex-husband to determine ownership of the land. The land is situated in a state with the following statute: "No conveyance or mortgage of an interest in land is valid against any subsequent purchaser for value without notice thereof whose conveyance is first recorded." (Race-Notice) How will the court most likely rule? (A) The guitarist has superior rights to both the ex-husband and the buyer. (B) The ex-husband has superior rights to both the guitarist and the buyer (C) The buyer has superior rights to both the guitarist and the ex-husband. (D) The buyer has superior rights to the ex-husband, but the guitarist has superior rights to the buyer.

ANSWER: (C) The buyer has superior rights to both the guitarist and the ex-husband. Even though the neighbor wasn't a BFP, once she recorded and the ex-husband didn't have knowledge of the guitarist, so he was a BFP. The buyer then took under the shelter rule so he has title superior to the guitarist.

(Property) 14. A woman purchased a tract of land from a man by warranty deed. Unbeknownst to the woman, the man was not the actual owner of the tract. The woman built a home on the tract and moved into it. Two years later, the actual owner learned of the man's transaction with the woman and prevented the woman from entering the tract from that point forward. This led to a costly court battle. When the woman notified the man and told him that she thought it was his duty to straighten this out, he ignored her. The woman would succeed in a suit for damages against the man for breach of which of the following covenants of title? (A) The covenant of quiet enjoyment only. (B) The covenants of seisin, right to convey, quiet enjoyment, warranty, further assurances, and the covenant against encumbrances. (C) The covenants of seisin, right to convey, quiet enjoyment, warranty, and further assurances. (D) The covenants of seisin and right to convey only.

ANSWER: (C) The covenants of seisin, right to convey, quiet enjoyment, warranty, and further assurances. The woman would succeed in a suit for damages against the man for breach of the covenants of seisin, right to convey, quiet enjoyment, warranty, and further assurances, but not on the covenant against encumbrances. A general warranty deed gives the grantee six covenants of title: the right to seisin, the right to convey, a covenant against encumbrances, the covenant of quiet enjoyment, the covenant of further assurances, and a general warranty. Under the covenants of quiet enjoyment, warranty, and further assurances, the man promised that (i) the woman would not be disturbed in her possession of the tract; (ii) he would defend the woman's title against lawful claims; and (iii) he would perform whatever acts are necessary to perfect the woman's title. Because the man neither owned the tract of land nor was acting as the actual owner's agent, he breached the covenants of seisin and right to convey at the time of the conveyance to the woman. When the actual owner prevented the woman from re-entering the property, this interfered with the woman's quiet enjoyment, and the man's refusal to "straighten this out" was a breach of the covenant of further assurances. Thus, (C) is the correct answer. Hence, (A) is incorrect because quiet enjoyment was not the only covenant breached. There is nothing in the facts to suggest the property is encumbered; thus, the man did not breach the covenant against encumbrances, and (B) is therefore incorrect. (D) is incorrect because seisin and right to convey were not the only covenants that the man breached.

(Property) 13. A landowner possessed a 40-acre tract of land. He had inherited 30 acres and had possessed the other 10 acres for longer than the statutory period necessary to acquire title by adverse possession from a rancher. The landowner entered into a land sale contract promising to convey the 40 acres to a developer. The contract provided that the landowner would convey marketable title. The developer paid the landowner the purchase price and accepted a deed from him. The developer promptly recorded the deed. The rancher, having learned of the sale, brought an action against the developer to quiet title. The developer realized for the first time that there were no covenants for title in his deed. The developer brings an action against the landowner. What is the most likely outcome of the suit? (A) The developer will win, because the landowner breached the terms of the contract. (B) The developer will win, because the landowner misrepresented the size of the tract. (C) The landowner will win, because the terms of the deed control his liability. (D) The landowner will win, because the developer was negligent in not checking the covenants of title at the time of closing.

ANSWER: (C) The landowner will win, because the terms of the deed control his liability. The landowner will win because the terms of the deed, not of the contract, control his liability. There is an implied covenant in every land sale contract that at closing the seller will provide the buyer with a title that is "marketable." Marketable title is title reasonably free from doubt, i.e., title that a reasonably prudent buyer would be willing to accept. It need not be a "perfect" title, but the title must be free from questions that might present an unreasonable risk of litigation. Generally, this means an unencumbered fee simple with good record title. Generally, a title acquired by adverse possession is not considered marketable because the purchaser might be later forced to defend in court the facts that gave rise to the adverse possession against the record owner. Here, the marketability requirement did not have to be implied, it was an express term of the contract. Under the doctrine of merger, the contract merges into the deed, and the terms of the contract are meaningless. Even though the contract specified a "good and marketable title," it is the deed that controls, and the deed contained no covenants of title. A deed does not incorporate the title terms of a contract. Thus, (A) is wrong. (B) is wrong; it is not supported by the facts. (D) is wrong because the developer's negligence is irrelevant.

(Property) 18. A woman arranged with a bank to take out a loan for $30,000, secured with a mortgage on her home. On June 3, the woman executed the note and the mortgage, and the bank gave her a certified check for $30,000. On June 4, the woman sold her home to a wealthy buyer for $150,000 in cash. The buyer knew nothing about the mortgage. On June 5, the buyer recorded her deed to the property. Two hours after the buyer recorded, the woman fled the country. On the evening of June 8, which was a Saturday, the buyer presented her niece with a deed to the property as a gift. At 10 a.m. on June 10, the bank recorded its mortgage. At 2 p.m. on June 10, the niece recorded her deed. After the woman missed her first mortgage payment on July 1, the bank employees went to the title office. They discovered the deeds to the buyer and her niece. The bank demanded that the niece satisfy the $30,000 mortgage. The niece filed an appropriate suit to determine the various interests in the property. The recording statute in the jurisdiction reads, in relevant part: "A conveyance of an estate in land shall not be valid against any subsequent purchaser for value, without notice thereof, unless the conveyance is recorded." (NOTICE) How should the court rule? (A) The niece owns the property subject to the bank's mortgage, because the niece is a donee. (B) The niece owns the property subject to the bank's mortgage, because the bank recorded before the niece. (C) The niece owns the property free of the bank's mortgage, because the buyer was a bona fide purchaser for value without notice. (D) The niece owns the property free of the bank's mortgage, because the bank does not qualify as a bona fide purchaser for value.

ANSWER: (C) The niece owns the property free of the bank's mortgage, because the buyer was a bona fide purchaser for value without notice. The niece owns the property free of the mortgage because the buyer's status as a bona fide purchaser without notice brings the "shelter rule" into play. In general, a person who takes from a bona fide purchaser will prevail against any interest that the transferor-bona fide purchaser would have prevailed against, even if the person taking the property has actual or record notice of the prior interest. Hence, the buyer's status as a bona fide purchaser without notice "shelters" her niece from the bank's interest, and (C) is correct. (A) is incorrect because the niece's "sheltered" status as a transferee of a bona fide purchaser makes the fact that the recording act does not directly protect donees irrelevant. (B) is incorrect because a pure notice recording statute requires actual or constructive notice at the time of the conveyance. When the buyer conveyed to her niece, the bank had not yet recorded. (D) is incorrect because mortgagees for value are treated as "purchasers" under the recording act.

(Property) 21. A buyer purchased a parcel of land from a seller for $500,000. The buyer financed the purchase by obtaining a loan from the seller for $300,000 in exchange for a mortgage on the land. The seller promptly and properly recorded his mortgage. Shortly thereafter, the buyer gave a mortgage on the land to a creditor to satisfy a preexisting debt of $100,000 owed to the creditor. The creditor also promptly and properly recorded its mortgage. Within a year, the buyer stopped making payments on both mortgages, and the seller brought an action to foreclose on his mortgage. The creditor was not included as a party to the foreclosure action. The seller purchased the property at a public foreclosure sale in satisfaction of the loan. The creditor subsequently discovered the sale and informed the seller that it was not valid. Who has title to the land? (A) The seller, because he gave a purchase money mortgage and the creditor's mortgage was for a preexisting debt. (B) The seller, because the public foreclosure sale extinguished the creditor's interest. (C) The seller, but he must redeem the creditor's mortgage to avoid foreclosure. (D) The buyer, because the seller's foreclosure action was invalid without the inclusion of the creditor as a necessary party.

ANSWER: (C) The seller, but he must redeem the creditor's mortgage to avoid foreclosure. The seller has title to the land, but he must redeem the creditor's mortgage to avoid foreclosure. As a general rule, the priority of a mortgage is determined by the time it was placed on the property. When a mortgage is foreclosed, the purchaser at the sale will take title as it existed when the mortgage was placed on the property. Thus, foreclosure will terminate interests junior to the mortgage being foreclosed but will not affect senior interests. However, if a lien senior to that of a mortgagee is in default, the junior mortgagee has the right to pay it off (i.e., redeem it) to avoid being wiped out by its foreclosure. Thus, those persons with interests subordinate to those of the foreclosing party are necessary parties to the foreclosure action. Failure to include a necessary party results in the preservation of that party's interest despite foreclosure and sale. Hence, the seller's failure to include the creditor as a party to the foreclosure action preserved the creditor's mortgage on the property. To avoid the creditor's foreclosing (because the buyer was in default of the creditor's mortgage as well), the seller will need to pay off the creditor's mortgage. (A) is wrong because it is irrelevant. While a purchase money mortgage ("PMM"), given when the mortgagor buys the property, is considered to have priority over non-PMM mortgages executed at about the same time, even if the other mortgages are recorded first, that rule is not applicable here because the facts indicate that the seller's PMM was executed and recorded before the creditor's mortgage came into existence. (B) is wrong because the creditor was not included as a party to the foreclosure action. Thus, as discussed above, its interest is not extinguished by the seller's foreclosure action. (D) is wrong because the failure to include the creditor in the foreclosure action does not invalidate the action, it just preserves the creditor's junior mortgage on the property

(Criminal Procedure) 41. Acting on an anonymous telephone call, the police went to a woman's apartment, pounded on the door, and demanded to search it for possible stolen property. The woman refused. The police then kicked open the door and placed the woman under arrest. The woman then offered to give the officers some inside information in exchange for her release. Before she could say anything else, the woman was given Miranda warnings by the police. Thereafter, she told the police that she knew of a large supply of stolen property stored at a nearby warehouse and said that she and a friend had been selling the stolen property out of the warehouse for years. The police raided the warehouse and recovered the stolen property. The woman was charged with conspiracy to sell stolen property and for possession of stolen property. At her trial, the woman moved to suppress the statements. Which of the following is the woman's best argument in support of the motion to suppress?

ANSWER: (C) The woman's statements were fruits of an unlawful arrest, and although the Miranda warnings may have been sufficient to protect her right against self-incrimination, they were not sufficient to purge the taint of the illegal arrest.

3. A young man was planning a ski trip to Colorado to celebrate his graduation from business school. Because he had a problem with drugs in his late teens, his father was afraid that he would relapse when he got to Colorado, where marijuana is legal. Consequently, his father told him, "If you refrain from smoking marijuana during your trip to Colorado, I will give you $5,000 as a down payment on that car you are planning to buy." The young man said nothing but was pleased by the offer because he needed the money and had no plans to smoke marijuana in any case. The young man went to Colorado and did not smoke marijuana there. Three weeks into the trip, and a week before he was due to go home, his father died suddenly of a heart attack, prompting him to cut his trip short and return home. If the young man seeks payment of the $5,000 from his father's estate, will he likely prevail? (A) No, because he remained silent following his father's offer and, thus, did not make a valid acceptance of the offer. (B) No, because his father's offer to pay was terminated upon his death. (C) Yes, because he has performed under a valid contract, and thus his father's estate must now perform. (D) Yes, because he changed his position for the worse in reliance on his father's promise, and thus the executor is estopped from denying that the contract existed.

ANSWER: (C) Yes, because he has performed under a valid contract, and thus his father's estate must now perform. The young man will prevail because he has performed under a valid contract. The parties entered into a valid unilateral contract: The father offered to give his son $5,000 if he did not smoke marijuana during his trip to Colorado; the son accepted by giving up something that he had a legal right to do under Colorado law. Thus, there was consideration on both sides. Because the young man performed his duties under the contract by refraining from smoking marijuana while in Colorado, the father's estate is bound to perform its duties and pay the young man. (A) is incorrect because a unilateral contract such as this one is accepted by performance rather than by a promise. (B) is incorrect because an offer will not be terminated by the death of the offeror if the offeror's power to revoke is limited by law, such as in the case of a valid unilateral contract. Here, the young man had begun performance (by refraining from smoking marijuana), making the offer irrevocable during the time he was given to complete performance. (D) is incorrect because it contemplates promissory estoppel, a remedy that is relied on to make a promise enforceable when it would otherwise be unenforceable because of insufficient consideration. Because there was sufficient consideration between the young man and his father, as discussed above, a promissory estoppel argument would not be appropriate here.

28. A girlfriend and boyfriend decided to get married. The girlfriend was one month short of her 18th birthday, and the boyfriend was 19. They went to a jeweler who made wedding rings to order and described what they were interested in. The girlfriend and boyfriend then signed a purchase order for two rings, a woman's band for $500 and a man's for $650. After the girlfriend's 18th birthday, the rings were ready. The girlfriend went to the jeweler and told him that they would not be needing the rings because she and the boyfriend had broken off their engagement. When he protested that they were custom-made and would probably not sell to anyone else, the girlfriend said, "All right, I've got $400 in my savings account. I'll take my ring, but you'll have talk to my boyfriend about the other one." The jeweler had the girlfriend sign another purchase order for the woman's band at $400, payment to be made by the end of the month. When the jeweler did not hear from the girlfriend after another month, he brought an action for breach of contract against her. Evidence produced at trial established that the market value of the rings was $500 and $650 for the woman's and man's rings, respectively, and that the age of majority in the jurisdiction was 18.Is the jeweler entitled to recover against the girlfriend? (A) Yes, in the amount of $1,150. (B) Yes, in the amount of $500. (C) Yes, in the amount of $400. (D) No

ANSWER: (C) Yes, in the amount of $400. The jeweler is entitled to recover $400 because, on attaining the age of majority, the girlfriend affirmed the contract as to her ring at that price. Infants generally lack legal capacity to incur binding contractual obligations. Contracts entered into by infants are voidable at this election unless the contract is for a "necessity." On reaching majority, an infant may affirm; that is, choose to be bound by her contract. Here, at the time of entering into the contract for the two rings, the girlfriend was not yet 18 years old (the applicable age of majority). Because a wedding ring is not a necessity, the girlfriend's contract was voidable at her option. However, when the girlfriend went back to see the jeweler, she had reached the age of 18. At that time, she chose to be bound by the contract, although her affirmation was limited to her ring (instead of both rings), and to a lower price for her ring than that which was originally agreed upon. When a voidable promise is reaffirmed, the promise will be enforced according to the terms of the reaffirmation rather than the original obligation. Thus, the girlfriend is now bound by her contract, but only for the purchase of her ring at a price of $400. (D) is incorrect because it ignores the fact that, having affirmed the contract (albeit on different terms), the girlfriend, who is no longer an infant, is bound to the terms of the reaffirmed contract. (A) is incorrect because $1,150 reflects the purchase price of both rings as contained in the original contract. On reaching majority, the girlfriend chose not to be bound by the original contract. Similarly, (B) is incorrect because $500 reflects the purchase price of the girlfriend's ring as contained in the original agreement (as well as the market value of the ring). On reaching majority, the girlfriend could have refused to pay anything. She is now bound only to the contract as affirmed by her, which is for $400.

(Property) 3. A homeowner agreed to sell his home to an accountant. He gave the accountant a general warranty deed and the accountant gave him $86,000, his asking price. The deed was recorded. A few years later, the accountant sold the property to a doctor, who paid her $125,000 for the property. To save on attorney's fees, the accountant went to her local office supply store to purchase a general warranty deed form. The store was out of those forms, but the clerk suggested that she use the form labeled "Quitclaim Deed," asserting that it would transfer the title just as well as the other form. The accountant purchased the form and filled in the blanks with the appropriate information she copied from her old deed. The doctor did not hire a lawyer to represent him in the purchase of the house.The doctor accepted the deed from the accountant and gave her $125,000. Soon after the doctor moved into the house, it was discovered that the homeowner's title was not good. The true owner now demands that the doctor vacate. Title is judicially determined to be with the true owner, and the doctor is forced out. Does the doctor have any action against the homeowner or the accountant based on any covenant for title? (A) Yes, the doctor can sue both and can recover $125,000. (B) Yes, the doctor can sue the homeowner, but not the accountant, and can recover his full $125,000. (C) Yes, the doctor can sue the homeowner, but not the accountant, and can recover only $86,000. (D) No, the doctor can sue neither the homeowner nor the accountant.

ANSWER: (C) Yes, the doctor can sue the homeowner, but not the accountant, and can recover only $86,000. The homeowner gave a warranty of title to the accountant when he gave her a general warranty deed. This warranty runs with the land and can be enforced by any subsequent purchaser. Damages are limited, however, to the purchase price received by the warrantor. Because the homeowner received only $86,000 from the accountant, that is all that the doctor can recover. The doctor gets nothing from the accountant because she gave only a quitclaim deed, which gives no warranty of title. (A) is incorrect because the doctor cannot prevail over the accountant at all, and he can recover only $86,000 against the homeowner. (B) is incorrect. While the doctor can win over the homeowner and not the accountant, he is limited to the money the homeowner received from the accountant—$86,000. (D) is incorrect. Because the homeowner gave a general warranty deed, that warranty for title runs with the land and can be enforced by any subsequent purchaser. There are two (sometimes three) future covenants contained in a general warranty deed—the covenants of quiet enjoyment, warranty, and (sometimes) further assurances. Quiet enjoyment is a covenant that the grantee will not be disturbed in her possession or enjoyment of the property by a third party's lawful claim of title. Warranty is a covenant that the grantor agrees to defend on behalf of the grantee any lawful or reasonable claims of title by a third party. Further assurances is a covenant to perform whatever acts are reasonably necessary to perfect the title conveyed if it turns out to be imperfect. These three covenants are future covenants that are breached, if at all, only on interference with the possession of the grantee or her successors; thus, they "run" with the grantee's estate. Here, the future covenants contained in the accountant's deed from the homeowner run with the land, allowing the doctor to seek recourse against the homeowner, the original grantor, for the disturbance in possession.

(Criminal Procedure) 7. A police officer was given a tip about a blonde male living in a nearby trailer park who was selling narcotics. The officer immediately drove to the trailer park and obtained from the manager the names of six blonde males who had trailers or mobile homes in the trailer park. At the first lot, the officer knocked on the defendant's door, announced that he was a police officer, and asked to talk to the defendant. The defendant's girlfriend, who did not live there but had been visiting, told the officer that the defendant would not be back for some time. The officer, believing that the girlfriend lived there, told her that he suspected that the defendant was dealing drugs and asked her if he could look around a little. The girlfriend said, "Sure, why not?" and let the officer in. After seeing nothing in the main living area, he went into the small back bedroom and opened several small storage compartments. In the corner of one of the compartments, he found an opaque bag. On opening it, he observed that it contained what appeared to be marijuana and confiscated the bag. Shortly thereafter, the defendant was arrested and charged with possession of narcotics with intent to distribute, a felony. On a motion by the defendant's attorney to suppress the introduction of the marijuana into evidence, how is the court likely to rule? (A) For the defendant, because his girlfriend did not live in the trailer. (B) For the defendant, because the search exceeded the scope of the consent. (C) Against the defendant, because mobile homes fall within the automobile exception to the warrant requirement. (D) Against the defendant, because the officer reasonably believed that the defendant's girlfriend lived in the trailer.

ANSWER: (D) Against the defendant, because the officer reasonably believed that the defendant's girlfriend lived in the trailer. The court should deny the defendant's motion because the officer reasonably believed that the defendant's girlfriend lived in the trailer, making the search valid. Under the exclusionary rule, evidence obtained from an unconstitutional search must be excluded from trial. To be valid, searches must be reasonable. The Supreme Court has held that most searches are unreasonable unless the police obtain a warrant before searching. However, there are six categories of searches that the Court has held to be reasonable without a warrant. One such category is searches conducted pursuant to consent. To fall within this exception to the warrant requirement, consent must be given by one who appears to have an apparent right to use or occupy the premises and the search cannot go beyond the scope of the consent given. The consent is valid as long as the police reasonably believed that the person who gave the consent had the authority to do so, and the scope of the consent is limited only to areas to which a reasonable person under the circumstances would believe it extends. Here, the girlfriend's consent was valid because the officer believed that she lived there. His belief appears to be reasonable because she answered the door, knew of the defendant's whereabouts, and readily consented to the search. Therefore, the search was valid under the consent exception and the evidence should not be excluded. (A) is incorrect because consent is not invalid merely because the person who gave it did not actually have authority to do so; the police need only reasonably believe that the person had authority to consent, and as explained above, it was reasonable for the officer to believe that the defendant's girlfriend had authority here. (B) is incorrect because the scope of consent extends to any area where a reasonable person under the circumstances would assume it extends. Because the officer told the girlfriend that he suspected the defendant of dealing drugs, it was reasonable to assume that he was looking for drugs and so would probably look in even small containers. (C) is incorrect because it appears that the defendant's trailer would not fall within the automobile exception. Certain searches of automobiles are excluded from the requirement of a warrant because the Supreme Court has held that people have a lesser expectation of privacy in an automobile than in other areas and automobiles are likely to disappear before a warrant can be acquired. The automobile exception extends not only to cars, but also to other vehicles that are readily mobile and as to which there is a lesser expectation of privacy. However, nothing in the facts here indicates that the defendant's mobile home may readily be moved, and because it appears to be the defendant's regular home rather than a vehicle, it is doubtful that the Court would find the requisite lesser expectation of privacy. Therefore, the trailer would not fall within the automobile exception to the warrant requirement.

(Property) 12. An elderly aunt devised her land to a charity "because my nephew has been stealing from me." When the nephew, who had been caring for the aunt, discovered the will, he threatened to withdraw his care unless she conveyed the land to him. The nephew obtained a blank deed and filled in the description of the land and the parties' names. Under "consideration" he wrote, "past and future care." The aunt signed the deed and the nephew recorded it. Subsequently, the nephew sold the land for market value to a buyer who was unaware of the nephew's threat to the aunt. The buyer recorded her deed. Last month, the aunt died. If the charity brings suit to impose a constructive trust on the land, will it prevail?

ANSWER: (D) No, because the buyer is a bona fide purchaser.

(Torts) 9. A city ordinance required that all dogs be leashed when taken outside of an enclosed area. One dog owner often allowed his dog to run loose in front of his house. One clear and sunny day when the dog was running loose, a pizza delivery driver was driving carefully on that street at a speed somewhat below the posted limit. The dog dashed out into the street from between two parked cars. The delivery driver alertly applied her brakes but could not avoid striking the dog. A man driving directly behind the delivery driver promptly applied his brakes as soon as he saw the delivery driver brake. However, the man's vehicle struck the rear of the delivery driver's car. Both of the vehicles suffered damage, and both drivers suffered minor personal injuries. Will the man prevail if he sues the delivery driver for his vehicle damage and personal injuries?

ANSWER: (D) No, because the delivery driver was not negligent.

(Criminal Procedure) 30. A defendant was arrested on suspicion of running an illegal "moonshine" operation. After taking the defendant back to the police station, an officer began questioning the suspect, thinking that his partner had already given the defendant a Miranda warning. The defendant voluntarily confessed to each and every element of the crime. At trial, the defendant took the witness stand and testified on his own behalf, declaring that he was innocent and that a distillery that the officers found at his home belonged to someone else. The prosecution, on cross-examination, produced the confession that the defendant gave concerning his illegal activities. The defense counsel objected to the admission of the confession. How should the court rule on the defendant's objection? (A) Sustained, because all evidence obtained in violation of Miranda rights is inadmissible. (B) Sustained, because the prosecution did not get permission from the court in advance to use the confession for any purpose. (C) Overruled, because the prosecution may question the defendant on cross-examination concerning any issue that was brought out in his defense. (D) Overruled, but the confession should be admitted only for the limited purpose of impeachment.

ANSWER: (D) Overruled, but the confession should be admitted only for the limited purpose of impeachment. The defendant's objection should be overruled. A confession obtained in violation of Miranda, but otherwise voluntary, can be used for the limited purpose of impeaching a defendant who testifies at trial. In contrast, an involuntary confession cannot be used to impeach. Here, there are no facts to indicate that the defendant's statement was involuntary. Thus, it can be used to impeach the defendant. (A) is too broad a statement. Although a confession obtained in violation of Miranda is inadmissible in the state's case-in-chief as evidence of guilt, as discussed above, such evidence is admissible for limited purposes. (B) is wrong. Advance permission from the court is not a requirement if the confession is used to impeach. (C) is a correct statement but it does not speak directly to the issue of whether the confession is admissible and to what extent.

(Property) 9. A rancher entered into a written contract to buy a farm from a farmer for $100,000. The contract stipulated for closing on September 30. In addition, the contract contained the following provision: "The taxes shall be prorated as agreed to by the parties at a later date." Upon the signing of the contract, the rancher gave the farmer a check for $10,000 as a down payment. On September 28, the rancher notified the farmer that he would not be able to close on the farm until October 2, because the closing on his current home, the proceeds from which were to be applied to his purchase of the farm, was unavoidably delayed due to his buyer's illness. Meanwhile, the farmer had difficulty finding a home she liked as well as the farm. She decided that she would rather not sell the farm and wished to avoid the contract with the rancher. On October 2, the rancher showed up at the closing with the $90,000 to tender to the farmer. The farmer did not show up. The rancher sues for specific performance. In whose favor will the court most likely rule?

ANSWER: (D) The rancher, because time was not of the essence. The rancher will prevail because there is no evidence that time was of the essence. In general, courts presume that time is not of the essence in real estate contracts. Thus, the closing date stated in the contract is not absolutely binding in equity, and a party, even though late in tendering his own performance, can still enforce the contract if he tenders within a reasonable time. (One to two months is usually considered reasonable.) Time will be considered of the essence only if: (i) the contract so states, (ii) the circumstances indicate it was the parties' intention, or (iii) one party gives the other notice that he desires to make time of the essence. The contract in this case made no mention that time was of the essence. The facts do not indicate any circumstances, such as rapidly fluctuating prices or the need for the money to close another critical transaction, that would indicate that the rancher and the farmer intended time to be of the essence. The farmer did not give the rancher reasonable notice before September 30 that she wanted to make time of the essence. Thus, the court will not find that time is of the essence here. Because time is not of the essence, the rancher is not in material breach and is entitled to specific performance. (A) is wrong because the Statute of Frauds is not violated here. Contracts for the sale of land must be in writing to be enforceable. The essential terms for purposes of the Statute of Frauds are: the description of the property, the identification of the parties, and the price. The tax provision is not an essential term. It is an incidental matter, which need not appear in writing or even be agreed upon. (B) is wrong because, as discussed above, the rancher is not in material breach. Time was not of the essence, so the fact that the rancher did not tender his performance on September 30 did not constitute a breach of the land sale contract. (C) is wrong because the doctrine of equitable conversion will not affect the rights of the parties in this situation. The doctrine of equitable conversion holds that once an enforceable contract of sale is signed, the purchaser's interest is real property, and the seller's interest (the right to proceeds) is personal property. This is important with respect to which party bears the risk of loss if the property is damaged before the date set for closing or if one of the parties dies prior to closing. It has no effect in situations like this one where the question in issue is the enforceability of the contract itself.

(Torts) 7. A woman took her car in for scheduled maintenance. The mechanic certified that the car was in perfect working order. Later that day, the woman was driving beyond the posted speed limit when her brakes failed, causing her car to strike a pedestrian. If the pedestrian brings an action against the mechanic who certified the woman's car as operable, what will be the probable outcome? (a) Judgment for the mechanic, because the pedestrian was legally a bystander. (b) Judgment for the mechanic, because the woman's negligence was an independent, superseding cause. (c) Judgment for the pedestrian if the mechanic was negligent in inspecting the car. (d) Judgment for the pedestrian, because the mechanic was strictly liable in tort.

ANSWER: (c) Judgment for the pedestrian if the mechanic was negligent in inspecting the car. The pedestrian will prevail if the mechanic was negligent in inspecting and certifying the car. Because the brakes failed soon after the mechanic certified the car, it is highly likely that the mechanic was negligent. There is no strict liability in tort for service transactions, so (D) is wrong. Privity of contract is not required to find liability for negligence, and the pedestrian's presence on the street makes her a foreseeable plaintiff, so (A) is wrong. (B) is incorrect because this is not a superseding cause situation; the mechanic's potential negligence would not be cut off by the woman's foreseeable negligent driving.

18. On August 1, a buyer contracted to purchase a parcel of land from a seller at a price of $100,000. The written agreement required that the buyer deposit 5% of the purchase price (i.e., $5,000) with the seller on August 1 and that, should the buyer fail to tender the remainder of the purchase price on or before September 15 (the appointed closing date), the $5,000 would be forfeited to the seller. On September 15, the buyer told the seller that he was no longer interested in purchasing the parcel and demanded a refund of his $5,000. The seller refused and sold the parcel the next day to a third party for $100,000. The buyer filed suit against the seller to recoup the $5,000 he had given the seller on August 1. If the court rules in favor of the seller, which statement best supports this outcome? A) $5,000 constitutes reasonable damages for breach. B) The provision allowing the seller to keep the $5,000 was a valid penalty clause. C) The buyer's breach left him with no further rights in the $5,000. D) The risk of loss is on the buyer.

ANSWER: A) $5,000 constitutes reasonable damages for breach. The provision providing for forfeiture of the $5,000 is a liquidated damages provision. It will be enforceable if it is not a "penalty" but rather a reasonable estimate of damages for breach. (B) is incorrect because, if the provision is indeed a penalty, it will not be enforceable. (C) is incorrect because, although the buyer is in breach, the liquidated damages provision would not be enforceable if it were an invalid penalty; rather, the seller would recover only actual damages, which, on the facts, seem to be nil. (D) is incorrect because this is not a risk of loss problem. There is no "loss" in the sense of damage to the property or its value; rather, the issue is the enforceability of the liquidated damages provision in the contract.

23. A woman contracted with a dog breeder for the purchase of a show dog. Prior to the sale, the breeder falsified the dog's pedigree records to indicate that there were several champions in the dog's bloodline. After a few weeks of enjoying the dog's companionship, the woman discovered the breeder's actions when she attempted to register the dog with a local dog club. The woman wishes to keep the dog, but is understandably unhappy with the high price that she paid. What is the woman's best course of action for relief? A) The woman should notify the seller and sue for warranty damages. B) The woman should seek to rescind the contract based on fraudulent misrepresentation. C) The woman should seek reformation of the contract to reflect a fair price for the dog. D)The woman should take no action because she wishes to keep the dog.

ANSWER: A) The woman should notify the seller and sue for warranty damages. The woman should notify the seller and sue for damages because the dog the woman received was not of the quality she was promised, in violation of an express warranty made by the seller. Any affirmation of fact made by a seller to a buyer creates an express warranty if the statement is part of the basis of the bargain. Here the woman was looking for a show dog, and the breeder represented to her that this dog was part of a champion bloodline. The dog's pedigree clearly was a part of the basis of the bargain because the buyer paid a higher price for the dog than she would have if the dog was not from a champion bloodline. The breeder's actions created an express warranty. The dog the woman received, however, was not as the breeder represented, thus the breeder breached this warranty. Nevertheless, the woman has decided to keep the dog. If a buyer accepts goods that breach one of the seller's warranties, the buyer may recover damages. The basic measure of damages in such a case is the difference between the value of the goods as delivered and the value they would have had if they had been according to contract, plus incidental and consequential damages. To recover damages for any defect as to accepted goods, the buyer must, within a reasonable time after she discovers or should have discovered the defect, notify the seller of the defect. This would be the best course of action among the options given for the woman because she would keep the dog and get back some of the money she overpaid for the dog.

25. A landowner and a purchaser orally agreed that the landowner would convey 20 acres of his 160-acre farm to the purchaser. At the time of their agreement, the landowner wrote on the back of an envelope, "I hereby promise to convey the northern 20 acres of my farm to [the purchaser] for $10,000." One month later, the purchaser tendered $10,000 to the landowner, but the landowner refused to convey the 20 acres. If the purchaser sues the landowner to convey the land and the landowner prevails, what will be the most likely reason? A) The writing was not signed by the landowner. B) The writing was not signed by the purchaser. C) The writing did not describe the property with specificity. D) The writing was on the back of an envelope.

ANSWER: A) The writing was not signed by the landowner. If the landowner prevails, it will be because the writing was not signed by the landowner. Under the Statute of Frauds, to be enforceable a contract for the sale of land must be evidenced by a writing signed by the party sought to be charged. Here, the landowner is the party that the purchaser is seeking to charge, so his signature is required on the writing.

1. On August 1, a realtor mailed a written offer to a developer for the sale of a large tract of land. The offer included the following terms: This offer expires on September 1, if the offeree has not caused an acceptance to be received by the offeror on or before that date. Early on September 1, the developer sent a written acceptance by messenger but the messenger company negligently withheld delivery to the realtor until September 2. On September 4, the realtor entered into a contract for sale of the tract to another buyer but did not inform the developer of the transaction. When the developer contacted the realtor a few days later, the realtor said there was no contract between them. What is the most accurate statement regarding the relationship between the parties? A. No contract between the realtor and the developer arose on September 2. B. A contract would have arisen if a letter of acceptance were mailed on September 1. C. The realtor's silence constituted an acceptance of the developer's written message on September 2. D. A voidable contract arose on September 1.

ANSWER: A. No contract between the realtor and the developer arose on September 2. No contract arose on September 2 because the realtor's offer expired on September 1, when the realtor did not receive the developer's acceptance. If a period of acceptance is stated in an offer, the offeree must accept within that period to create a contract. Failure to timely accept terminates the power of acceptance in the offeree (i.e., a late acceptance will not be effective and will not create a contract). Under the mailbox rule, an acceptance generally is effective upon dispatch (i.e., the acceptance creates a contract at the moment it is mailed or given to the delivery company). However, the mailbox rule does not apply where the offer states that acceptance will not be effective until received. In the latter case, acceptance is effective only upon receipt. Here, the realtor's offer specifically stated that the acceptance must be received by September 1 to be effective. Thus, the realtor opted out of the mailbox rule, and no contract was created by delivery of the acceptance on September 2. Note that the developer will not be able to successfully argue that the acceptance was valid since the late delivery was the messenger company's fault. This would be a valid argument if the mailbox rule applied here, because the acceptance would have been effective on September 1, when the message was given to the messenger company. However, by opting out of the mailbox rule, the realtor put the burden of any negligence in delivery on the developer. Thus, there was no valid acceptance. (B) is incorrect because of the requirement that acceptance be received by September 1. This requirement obviates the general "mailbox rule," so that the mere mailing of a letter (or sending of a message) does not operate as an effective acceptance. (C) is incorrect because the realtor was not obligated to respond in any way to the message received on September 2. Once the specified time passed without receipt of acceptance, the offer (as well as the developer's power of acceptance) was terminated. Thus, receipt of the message on September 2 created neither a contract nor an obligation on the part of the realtor to respond to the message. (D) is incorrect because no contract, voidable or otherwise, arose on September 1. As explained above, there could be no contract because acceptance of the offer was not received as specified by the offer. Also, the facts do not indicate circumstances under which a contract is usually held to be voidable. A voidable contract is a contract that one or both parties may elect to avoid (e.g., contracts of infants). The facts of this question provide no basis for concluding that any contract that might have arisen between these parties would be voidable.

(Criminal Procedure) 16. The police, suspecting that the defendant was dealing drugs, observed several people walk up to the defendant's door, knock on his door, and then exchange cash for small packages that the police believed contained drugs. Two uniformed police officers then walked up to the door and knocked. The defendant answered the door, and one police officer asked if they could come in and take a look around. The defendant, believing that he had no other choice but to let the officers inside, agreed. Once inside, they discovered equipment used for making methamphetamine and several tablets of methamphetamine that were sitting on a table covered by a bed sheet. One officer promptly arrested and handcuffed the defendant while the other seized the equipment and tablets. Prior to his trial for the illegal manufacture and possession of methamphetamine, the defendant moved to suppress the evidence as having been illegally seized. Should the motion be granted? A. No, because the defendant allowed the police officers to enter his home and look around. B. No, because exigent circumstances existed for the warrantless seizure of the evidence. C. Yes, because the police should have secured the area and obtained a warrant to seize the evidence. D. Yes, because the defendant's consent was not voluntary.

ANSWER: A. No, because the defendant allowed the police officers to enter his home and look around. The evidence should not be suppressed because the defendant consented. To be reasonable under the Fourth Amendment, most searches must be pursuant to a warrant. The warrant requirement serves as a check against unfettered police discretion by requiring the police to apply to a neutral magistrate for permission to conduct a search. A search conducted without a warrant will be invalid (and the evidence discovered during the search generally must be excluded from evidence) unless the search and seizure falls within an exception to the warrant requirement. One exception to the warrant requirement is when the police have valid consent to search the premises. The police may conduct a valid warrantless search when they have a voluntary consent to do so. Knowledge of the right to withhold consent, while a factor to be considered, is not a prerequisite to establishing a voluntary consent. In the instant case, there are no facts that indicate that the police put any undue pressure on the defendant to consent to the search. Although it is a factor to be considered in determining whether the consent was voluntary, the defendant's subjective mistake about being able to withhold consent would probably not, by itself, be sufficient to deem the consent involuntary. As a result, (A) is the correct answer, and (D) is incorrect. (B) is incorrect. The Supreme Court has made it clear that there is no general "emergency" exception to the warrant requirement, although the police may seize "evanescent" evidence in certain circumstances. That said, there is no indication that evidence here would disappear, as it seems that the defendant's operation was ongoing, thus giving the police time to get a warrant. (C) is incorrect because the police may conduct a warrantless search with the defendant's permission, as they did in this question.

(Criminal Procedure) 15. A man and a woman were traveling in the man's car when they were stopped by the police for running a red light. Before the police came up to the car, the man told the woman, "You owe me a favor. Keep this package for me," and gave the woman a small foil package. The woman put the package in her backpack, saying, "O.K., but don't tell me what's in it." Before the police even began to question the occupants, the man blurted out, "I'm clean, man, but she has a stash," pointing at the woman. The officers searched the backpack that the woman was holding and found the foil package, which contained heroin. The woman was arrested, but the man was not. Is the evidence found on the woman admissible? A. Yes, under the automobile exception. B. Yes, because due process imputes knowledge where there is willful ignorance. C. No, because due process forbids granting of immunity to the more culpable defendant. D. No, because the woman did not know that the package contained heroin.

ANSWER: A. Yes, under the automobile exception. The evidence is admissible because the search was valid. Even though the police have validly stopped an automobile, they cannot search the vehicle without meeting the requirements of one of the exceptions to the warrant requirement, such as the automobile exception (which requires probable cause) or consent. The automobile exception comes into play when the police have probable cause to believe that the vehicle contains evidence of a crime. Under the exception, the police may search anywhere in the vehicle in which the item for which they have cause to search may be hidden, including packages in the vehicle. The statement of the man to the police officers gave them probable cause to believe that the car contained evidence of a crime (i.e., that the woman had drugs somewhere in the car). Thus, the requirement for application of the automobile exception was present, providing validity for the warrantless search conducted by the police. Because the search was valid, the evidence found on the woman is admissible. Besides being an incorrect statement of law, (B) is incorrect because it focuses on the woman's knowledge of the contents of the package. Whether the woman knew that heroin (or some other illegal substance) was in the package is irrelevant to the admissibility of the heroin. Even assuming that the woman knew of the contents, the search would not be valid unless there was a ground for the warrantless search. (D) similarly links the woman's knowledge of the package's contents to the admissibility of the evidence. As noted above, the admissibility of the evidence is dependent on the validity of the search that produced the evidence, rather than on the knowledge of the defendant as to the existence of the evidence. (C) is incorrect for three reasons: First, due process does not prohibit granting of immunity to a more culpable defendant. Second, there is no indication that immunity was even granted here (immunity from prosecution may be granted to compel a witness to answer questions. The facts merely state that the man was not arrested; this does not necessarily mean that he was granted immunity). Third, the call of the question relates to the admissibility of the evidence, and a grant of immunity does not relate to the question of the admissibility of the evidence found on the woman; such admissibility is determined by the validity of the search of the woman by the officers.

(Criminal Procedure) 14. A defendant was driving through an apartment building area plagued with an unusually high incidence of burglaries and assaults. Acting pursuant to a police department plan to combat crime by randomly stopping automobiles in the area between midnight and 6:00 a.m., a police officer stopped the defendant and asked him for identification. As the defendant handed the officer his license, the officer directed a flashlight into the automobile and saw what appeared to be the barrel of a shotgun protruding from under the front seat on the passenger side of the car. The officer ordered the defendant from the car, searched him, and discovered marijuana cigarettes and a shotgun. At the defendant's trial for unlawful possession of narcotics, his motion to suppress the use of the marijuana as evidence should be: A: sustained, because the marijuana was discovered as a result of the unlawful stopping of the defendant's automobile. B: sustained, because the use of the flashlight constituted a search of the interior of the defendant's automobile without probable cause. C: denied, because the officer's conduct was consistent with the established police plan. D: denied, because the discovery of the gun in plain view created the reasonable suspicion necessary to justify the arrest and search of the defendant.

ANSWER: A: sustained, because the marijuana was discovered as a result of the unlawful stopping of the defendant's automobile.

24. On January 30, a company that designs and builds generators to standard industrial specifications received a telephone call from a buyer who ordered two generators at a price of $25,000 each. The parties agreed that delivery of the first generator would be on March 15, and the second on April 30. Payment was to be made no more than 30 days after delivery. On March 12, the company delivered the first generator, which the buyer accepted. On April 22, the company completed the second generator but had not yet notified the buyer. On April 23, the buyer, having made no payment to the company, canceled the agreement. The company brings an action against the buyer for breach of contract. How much should the company recover in damages? A) Nothing. B) $25,000 only. C) Damages for total breach of contract for the sale of two generators, because the buyer accepted part performance. D) Damages for total breach of contract for the sale of two generators, because the goods were made for the buyer.

ANSWER: B) $25,000 only. The company should recover $25,000 only. Contracts for goods for $500 or more must be evidenced by a writing to be enforceable. There are three exceptions to this rule: specially manufactured goods unsuitable for resale in the seller's regular course of business, contracts admitted in court, and contracts partially accepted (enforceable to the extent of the acceptance). Here, the contract was for $50,000 and was oral. Thus, it will be enforceable only if one of the exceptions applies. The buyer's acceptance of the first generator constitutes part acceptance that will make the buyer liable to the extent of the acceptance: $25,000. Therefore, (B) is correct and (A) is incorrect.

22. A park board in a large suburb announced that it was accepting bids for renovation work on its recreation center. A builder advertised for sub-bids for the electrical work, and a local electrician submitted to the builder by electronic bidding service a sub-bid of $130,000. However, due to the bidding service's negligence, the sub-bid that the builder received from the electrician read $30,000 instead of $130,000. Because this was the lowest sub-bid that the builder received for the electrical work, and $60,000 less than the next lowest sub-bid, the builder awarded the subcontract to the electrician. Based in part on the electrician's sub-bid, the builder came up with a bid for the job that beat out all of the competition and won the job. What is the electrician's best argument to successfully refuse to perform the resulting contract? A) The contract would be unconscionable. B) The builder should have been alerted to the existence of a mistake in the sub-bid. C) The bidding service, not the electrician, was responsible for the faulty bid. D) The builder's failure to check out all sub-bids precludes enforcement of the contract.

ANSWER: B) The builder should have been alerted to the existence of a mistake in the sub-bid. The electrician's best argument would be that the great difference between the electrician's sub-bid, as transmitted, and the next lowest sub-bid should have alerted the builder to the obvious mistake in the electrician's sub-bid. Typically, if only one of the parties entering into a contract is mistaken about facts relating to the agreement, the unilateral mistake will not prevent formation of a contract. However, if the nonmistaken party is or had reason to be aware of the mistake made by the other party, the contract is voidable by the party who made the mistake. Thus, the electrician's best argument is that the $60,000 difference between the electrician's bid of $30,000 and the next lowest bid should have alerted the builder to the existence of a mistake, so the electrician should be able to refuse to perform the contract.

(Property) 10. To buy a house, an investor secured a $10,000 mortgage from a bank. The bank promptly and properly recorded its mortgage. Subsequently, the investor financed certain improvements to the house with a $2,000 mortgage on the land from a finance company. The finance company promptly and properly recorded its mortgage. Before the investor made a payment on either mortgage, the federal government announced that it would begin storing nuclear waste products in the area. The value of property, including the investor's house, plummeted. The investor did not pay either the bank or the finance company, and the bank brought a proper action to foreclose, notifying both the investor and the finance company. A buyer bought the house at the foreclosure sale for $6,000, which was its fair market value. There are no special statutes in the jurisdiction regarding deficiency judgments. What does the investor owe? A $5,000 to the bank and $1,000 to the finance company. B $4,000 to the bank and $2,000 to the finance company. C Nothing to the bank and $2,000 to the finance company. D $4,000 to the bank and nothing to the finance company.

ANSWER: B. $4,000 to the bank and $2,000 to the finance company. Absent any anti-deficiency statutes, the investor remains personally liable to pay for any shortfall arising from the foreclosure sale. Proceeds from the sale are used to satisfy the loan that was foreclosed first. Hence, all of the proceeds ($6,000) went to the bank. Thus, the investor must pay the balance still due the bank ($4,000) and the entire amount of the finance company's mortgage ($2,000), which is terminated by the foreclosure of the senior mortgage. (A) is wrong because foreclosure sales are not allotted proportionally between senior and junior interests. (C) is wrong because foreclosure does not extinguish the underlying debt. (D) is wrong because the finance company's mortgage does not remain on the land after foreclosure of the senior mortgage; hence, the investor is liable for that debt as well.

(Criminal Procedure) 37. The defendant was arrested, given Miranda warnings, and charged with burglary. At the police station, he telephoned his mother and asked her to come to the station to post bail. Instead, his mother immediately called the family attorney. In the meantime, the police had begun questioning the defendant. Although he never told the police to stop the questioning, his answers were at first vague or clearly unresponsive. During the course of the questioning, the family attorney phoned the station and told the police that she had been hired to represent the defendant and would be there in half an hour. The police did not inform the defendant of the attorney's call. Ten minutes later, the defendant admitted to committing the burglary, and signed a statement to that effect prepared by the police. The attorney arrived a few minutes later and advised the defendant to remain silent, but he told her that he had already signed a confession. How should the court rule on the attorney's pretrial motion to exclude the confession as evidence at trial? A - Grant the motion, because the police had a duty to inform the defendant that an attorney was coming to represent him. B - Grant the motion, because the defendant has been deprived of his Sixth Amendment right to counsel. C - Deny the motion, because the defendant's statement admitting the crime was voluntary. D - Deny the motion, because the defendant waived his Miranda rights.

ANSWER: D - Deny the motion, because the defendant waived his Miranda rights. The defendant's confession should be admitted because he waived his Fifth Amendment privilege against compelled self-incrimination after receiving Miranda warnings. Miranda v. Arizona requires that a person in custody be informed of his right to remain silent and his right to the presence of an attorney during questioning. A suspect may subsequently waive his rights by making a confession, as long as the waiver was knowing and voluntary. In this case, the defendant received proper Miranda warnings, and there is no indication that he did not understand what his rights were. Although his answers during questioning were initially unresponsive, he never asked for an attorney or indicated that he wished to remain silent, and he voluntarily confessed after a relatively short period of interrogation. Hence, he validly waived his Miranda rights. (A) is incorrect because the police have no duty to inform the defendant that an attorney is attempting to see him. The defendant's ignorance of his attorney's efforts has no bearing on whether he made a knowing waiver of his Miranda rights. (B) is incorrect because the defendant's right to counsel was not violated. Although the defendant does have a separate Sixth Amendment right to counsel under Escobedo v. Illinois because he has already been arrested and charged with the crime, this right would only be violated if the defendant, after being informed of his right to counsel, had requested an attorney or had been prevented from seeing his attorney. Here, he made no request to see an attorney - even when he called his mother - and his attorney was allowed to see him immediately upon her arrival. Thus, he has waived his Sixth Amendment right to counsel. (C) is incorrect even though it is true that the defendant made a voluntary statement. Due process requires that for confessions to be admissible, they must be "voluntary," based on the totality of the circumstances, and here all of the circumstances indicate that the defendant's confession was voluntary. However, even a voluntary confession will be inadmissible if it was obtained in violation of Miranda rights. (D) is therefore a better choice than (C).

6. A sporting goods retailer whose tent stock was running low saw a listing for the tent she wanted priced at $90 in the catalog of a large camping goods manufacturer. The retailer phoned the manufacturer and placed her order for 10 tents on May 1. The next day, the manufacturer mailed the retailer a letter informing her that the tents were now $92 and that they would be shipped to her on May 16. The retailer received the letter on May 4, but never responded. On May 15, the retailer received a catalog from another company showing tents similar to the ones that she ordered, but for a cost of $70. She immediately called the manufacturer to cancel her order. Nevertheless, the manufacturer shipped the tents to the retailer on May 16. Assuming that the parties' communications were sufficient to form a contract, on what day was the contract formed? A. May 1, the day the retailer placed her order. B. May 2, the day the manufacturer sent its letter. C. May 4, the day the retailer received the letter. D. May 16, the day the tents were shipped.

ANSWER: B. May 2, the day the manufacturer sent its letter. The contract was formed on May 2. An offer to buy goods for shipment is generally construed as inviting acceptance either by a promise to ship or by shipment. Here, the letter constitutes a promise to ship and thus is an acceptance. The rule for acceptances is that they are effective as soon as they are dispatched, which was May 2. Thus, (B) is correct. (C) is incorrect because, under the mailbox rule, a letter of acceptance creates a contract at the moment of dispatch, not on the day that the offeree receives it. (A) is wrong because the order was an offer, not an acceptance of the catalog listing. Catalogs containing price quotations are generally construed as invitations to offer rather than offers. (D) is wrong because acceptance occurred before shipment; it occurred when the manufacturer sent its promise to ship.

(Property) 2. A seller entered into a written contract to sell a tract of land to a buyer. The buyer was to pay $1,500 per month for five years, at which time the seller would deliver a warranty deed. The contract was silent as to the quality of title to be conveyed. After making 12 payments, the buyer discovered that a neighbor had an easement of way over the land, which was not discussed at the time the seller and buyer entered into the contract. The neighbor had not used the easement over the previous year because she had been out of the country. On the basis of the easement, the buyer wishes to cancel the contract. Which party is more likely to prevail? A The seller, because the neighbor's easement has been extinguished. B The seller, because the buyer has no basis on which to rescind the contract. C The buyer, because the obligation to convey marketable title is implied. D The buyer, because the seller has breached the covenant against encumbrances.

ANSWER: B. The seller, because the buyer has no basis on which to rescind the contract. The seller is more likely to prevail because the buyer has no basis on which to rescind the contract. Absent a provision to the contrary, a contract for the sale of land contains an implied promise by the seller that she will deliver to the buyer a marketable title at the time of closing. This promise imposes on the seller an obligation to deliver a title that is free from reasonable doubt; i.e., free from questions that might present an unreasonable risk of litigation. Title is marketable if a reasonably prudent buyer would accept it in the exercise of ordinary prudence. An easement that reduces the value of the property (e.g., an easement of way for the benefit of a neighbor) generally renders title unmarketable. If the buyer determines, prior to closing, that the seller's title is unmarketable, he must notify the seller and allow a reasonable time to cure the defect. If the seller is unable to acquire title before closing, so that title remains unmarketable, the buyer can rescind, sue for damages caused by the breach, or obtain specific performance with an abatement of the purchase price. However, the buyer cannot rescind prior to closing on grounds that the seller's title is unmarketable. Where an installment land contract is used, the seller's obligation is to furnish marketable title when delivery is to occur, e.g., when the buyer has made his final payment. Thus, a buyer cannot withhold payments or seek other remedies on grounds that the seller's title is unmarketable prior to the date of promised delivery. Here, there is a valid easement on the property (see below), but the seller has four years in which to cure this defect. Thus, the buyer cannot yet rescind on grounds that title is unmarketable. (A) is incorrect because the neighbor's easement has not been extinguished. An easement can be extinguished where the owner of the privilege demonstrates by physical action an intention to permanently abandon the easement. Mere nonuse is not sufficient to terminate an easement, unless the nonuse is combined with other evidence of intent to abandon it. Here, the fact that the neighbor did not use the easement for a year because she was out of the country does not establish her intent to abandon the easement. (C) is incorrect because, although the law implies in every land sale contract a covenant that title will be marketable, the seller has until the time of delivery to cure the defect. (D) is incorrect because the deed has not yet been delivered. The covenant against encumbrances is a covenant contained in a general warranty deed which assures that there are neither visible encumbrances (e.g., easements) nor invisible encumbrances (e.g., mortgages) against the title or interest conveyed. This covenant is breached, if at all, at the time of conveyance. Here, the deed has not yet been delivered, and thus this covenant has not yet been breached.

(Torts) 6. A landlord employed his friend as the on-site manager of one of his apartment buildings despite being aware that he had previously been arrested for criminal battery, disorderly conduct, and driving while intoxicated. The manager did a good job dealing with the general maintenance of the apartment building, although the landlord was aware that he continued to drink heavily. One night the manager, who was extremely intoxicated, attempted to swat an insect on the ceiling of his apartment and could not do so after several attempts. Enraged, he took a pistol from his drawer and shot at the insect. The bullet missed the insect and passed through the ceiling of his apartment into the apartment above, lodging in the leg of a tenant's social guest. Does the guest have a viable cause of action against the landlord? A. Yes, because the guest had been invited onto the property by the tenant. B. Yes, because the landlord was aware of the manager's habitual drunkenness and propensity for violence. C. No, because the landlord cannot be held liable for the manager's intentional torts. D. No, because shooting an insect was outside the scope of the manager's employment

ANSWER: B. Yes, because the landlord was aware of the manager's habitual drunkenness and propensity for violence. Because the landlord knew about the manager's continued heavy drinking and tendencies toward violence, the guest has a cause of action for negligence in the landlord's hiring of the manager. An employer owes a duty to all those who may foreseeably come into contact with his employee to exercise due care in the hiring, supervision, and retention of the employee, and the landlord's retention of the manager under these circumstances may be a breach of that duty. (A) is incorrect because the landlord's liability here is based on negligent hiring rather than the guest's status on the property. (C) is also incorrect. An employer can be held directly liable for the intentional tort of an employee if it was foreseeable and the employer was negligent in hiring or retaining the employee. (D) is a true statement that would be relevant for vicarious liability purposes. However, it does not preclude the landlord from being liable for his own negligence based on the foreseeability of his employee acting violently.

33. A print cartridge company sent a letter to a business office offering to supply a free premium printer if the business office would agree to purchase all the print cartridges the office would need from the print cartridge company. This letter arrived in the hands of the business owner on the same day the office printer failed. The business office had experienced a slow month and the business owner was trying to decide whether to pay the office rent for the month or fix the printer. Based on the offer, the owner paid the rent. A week after she put the rent check in the mail, the owner received a second letter from the print cartridge company indicating that the printer program was being canceled due to a lack of printers. The next day, before she read the second letter, the owner mailed her acceptance letter to the print cartridge company. The print cartridge company refused to supply the business owner with a printer. If the business owner brings a breach of contract action against the print cartridge company, how should the court rule? A: For the print cartridge company, because its offer to the business office was not sufficiently definite. B: For the print cartridge company, because its revocation letter was received by the business owner before she dispatched the acceptance letter. C: For the business owner, because she mailed her acceptance letter without being aware that the print cartridge company had revoked its offer. D: For the business owner, because the owner used funds to pay the office rent in reliance on the print cartridge company's offer.

ANSWER: B: For the print cartridge company, because its revocation letter was received by the business owner before she dispatched the acceptance letter. The print cartridge company will prevail because the business owner could no longer accept the offer. A revocation generally is effective when received by the offeree. A written communication is considered to have been received when (i) it comes to a person's attention, or (ii) it is delivered at a place of business through which the contract was made. The communication need not be read by the recipient to be effective. Hence, the business owner received the revocation when the revocation letter arrived. Her receipt of the revocation letter before she dispatched her acceptance letter effectively revoked the offer (even though the business owner was unaware of its contents when she mailed the acceptance).(A) is wrong because an offer to make a requirements contract (i.e., the buyer promises to buy from a certain seller all the goods it requires and the seller agrees to sell that amount to the buyer) is sufficiently definite because the quantity is capable of being made certain by reference to objective, extrinsic facts (i.e., the buyer's actual requirements). (C) is wrong because, as stated above, the revocation was effective when it was received (i.e., when it came into the offeree's possession), not when the offeree becomes aware of the revocation. (D) is wrong because the business owner's response to the offer was not reasonably foreseeable. As an exception to the general rule that a revocation is effective on receipt, an offer cannot be revoked and will be treated as an option contract for a reasonable length of time where the offeror could reasonably expect that the offeree would rely to her detriment on the offer. However, this exception usually is applied in only two circumstances: an offer for a unilateral contract and a subcontractor's bid to a general contractor. If the offeror is seeking a bilateral contract in a circumstance other than that of a subcontractor, it would be extremely rare for the offer to be irrevocable due to detrimental reliance. Generally, an offeree must accept the offer before relying on it. Here, it is not even clear that the business owner suffered a detriment (unless she could have skipped paying the rent that month). She was already obligated to pay rent. In any case, nothing suggests that the cartridge company reasonably should have foreseen that the business owner would rely on the offer the way she did (without having accepted it yet).

21. The owner of a sporting goods store noticed that her tent stock was running low. After consulting various manufacturers' catalogs, she decided to order from a large manufacturer of camping equipment whose catalog listed the 9 x 12 tent that she wanted at a cost of $70 per tent. On April 1, the store owner phoned the manufacturer and placed her order for 10 tents with the manufacturer's sales agent. The next day, the manufacturer mailed the store owner a signed letter stating that there was a contract for sale for 10 tents at $70 per tent plus a $40 shipping fee, and that the tents would be shipped on April 16. The store owner received the letter on April 4, but she never responded. On April 15, she received a catalog from another tent company showing tents similar to the ones that she ordered, but for a cost of only $60. The store owner immediately called the manufacturer with whom she had placed her order to cancel it. Nevertheless, the manufacturer shipped the tents to her on April 16. If the manufacturer sues the store owner to enforce the contract, is the manufacturer likely to prevail? A) No, because there was no meeting of the minds regarding the price term. B) No, because the store owner's promise was not in writing. C) Yes, because the manufacturer's April 2 letter was sufficient to bind the store owner. D) Yes, because the price of each tent is less than $500.

ANSWER: C) Yes, because the manufacturer's April 2 letter was sufficient to bind the store owner. The manufacturer will prevail because the April 2 letter was sufficient to bind the store owner. There are two issues involved here, the battle of the forms provisions of the UCC and the Statute of Frauds. Generally, to be enforceable, a contract for the sale of goods priced at $500 or more must be evidenced by a writing signed by the party sought to be bound. However, in contracts between merchants, if one party, within a reasonable time after an oral agreement has been made, sends to the other party a written confirmation of the understanding that is sufficient under the Statute of Frauds to bind the sender, it will also bind the recipient if: (i) the recipient has reason to know of the confirmation's contents; and (ii) she does not object to it in writing within 10 days of receipt. Here, both parties are merchants: one is a store owner and the other a manufacturer. The day after the store owner's oral order was placed, the manufacturer mailed a confirmation letter. Assuming the letter was signed or on the manufacturer's letterhead, it would be sufficient to bind the sender. Since the store owner received the letter and read it, she had reason to know of its contents and did not respond. Thus, the writing is sufficient under the Statute of Frauds to bind the store owner as well as the manufacturer. There is also an issue with the fact that the confirmatory memo contains a new term regarding shipping fees. If both parties to the contract are merchants, additional terms in an acceptance or confirmation will be included in the contract unless: (i) they materially alter the original terms of the offer (e.g., they change a party's risk or the remedies available), (ii) the offer expressly limits acceptance to the terms of the offer; or (iii) the offeror has already objected to the particular terms, or objects within a reasonable time after notice of them is received. The shipping term does not materially alter the terms of the offer, the offer did not limit the acceptance to its terms, and the store owner has not objected. Thus, the new term will become part of the contract.

34. A doll collector knew that an acquaintance from her doll collectors club coveted one particular doll that she owned. The doll collector mailed a letter to the acquaintance on May 3 offering to sell the doll to her for $750. Her letter arrived on May 4. On May 5, the doll collector changed her mind and immediately mailed a revocation to the acquaintance. This revocation arrived on May 7. As the mail carrier handed it to her, the acquaintance simultaneously handed to the mail carrier her own letter to the doll collector, unequivocally accepting her offer. What is the result of the actions here? A: The revocation was effective upon mailing, and the acceptance would be treated as a counteroffer. B: The acceptance was effective, as long as the acquaintance had no knowledge of the contents of the doll collector's letter when she handed her letter to the mail carrier. C: The outcome would turn on the court s determination as to whether the doll collector's letter had been received by the acquaintance before she had entrusted the letter of acceptance to the mail carrier. D: Handing a letter to a mail carrier is not a proper posting of the acceptance, and hence the acquaintance's purported acceptance is not timely.

ANSWER: C: The outcome would turn on the court s determination as to whether the doll collector's letter had been received by the acquaintance before she had entrusted the letter of acceptance to the mail carrier. The outcome would turn on the court's determination as to whether the doll collector's letter had been received by the acquaintance before she had entrusted the letter of acceptance to the mail carrier. At common law, an acceptance is effective upon dispatch (e.g., upon mailing a properly addressed and stamped letter) under the mailbox rule. The mailbox rule does not apply to revocations, however revocations are effective only upon receipt. Receipt does not require knowledge of the revocation, but merely possession of it. The communication need not be read by the recipient to be effective. [See Restatement (Second) of Contracts §68] The facts here present a close question as to whether there has been a dispatch of the acceptance before the receipt of the revocation. The outcome of this question will depend on the court's determination as to what came first (the posting of the acceptance or receipt of the revocation). This will decide the existence or nonexistence of the contract. (A) is incorrect because, as indicated above, revocation is effective only upon receipt, not mailing. (B) is incorrect because whether the acceptance is effective depends on whether the revocation was received before the acceptance was dispatched, and whether the revocation was received first is not dependent on whether the acquaintance had knowledge of its contents, but rather it depends on whether she had possession of it. (D) is incorrect because the mailbox rule makes an acceptance effective upon posting, and there is no reason to hold that handing a properly addressed, stamped letter to a mail carrier is not a valid posting.

20. A homeowner purchased a large recreational vehicle. Local ordinances in the homeowner's suburb prohibited residents from parking recreational vehicles on the street or in an open driveway, so the homeowner contacted a local contractor and explained his requirements for a garage. The contractor measured the vehicle and then entered into a written contract to build a garage for the homeowner at a price of $5,000, payable on completion of the job. When the garage was finished but before the contractor was paid, the homeowner drove the vehicle into the garage, only to discover that the garage was three inches too short to accommodate the vehicle. The homeowner was told that it would cost $4,000 to partially dismantle the garage and rebuild it to fit the vehicle. The homeowner refused to pay the contractor anything for the job. The contractor consulted with several independent real estate appraisers, and they all agreed that the garage had enhanced the value of the homeowner's property by $6,000. If the contractor sues the homeowner, what amount will the contractor likely recover? A $5,000, because a three-inch variation in length is, at most, a minor breach. B $2,000, measured by the difference between the amount that the garage enhanced the value of the property and the cost to rebuild the garage to specifications. C $1,000, measured by the difference between the contract price and the amount it would cost to rebuild the garage to specifications. D Nothing, because the three-inch error was a material breach, and the contractor will be unable to successfully claim substantial performance.

ANSWER: C. $1,000, measured by the difference between the contract price and the amount it would cost to rebuild the garage to specifications. The contractor will recover $1,000 in a restitution or quasi-contract action. Despite having built a garage, the contractor has breached the contract here. He was to build a garage that would fit the homeowner's recreational vehicle and the garage that he built is too short. Moreover, the breach was material, even though the garage was a mere three inches short, because the homeowner did not receive the substantial benefit of his bargain—a place for his vehicle. Therefore, the contractor cannot recover in contract and (A) is incorrect. Nevertheless, courts agree that where the breach is not willful, the contractor can recover on the failed contract in restitution or quasi-contract to prevent unjust enrichment. Here, it is clear that the breach was not willful. (D) is therefore incorrect. Regarding the amount of recovery, most courts would limit the recovery to the contract price (rather than the benefit received by the nonbreaching party), offset by the damages to the nonbreaching party (here, the $4,000 cost of rebuilding the garage as bargained for). Thus, (C) is correct and (B) is incorrect.

(Criminal Procedure) 31. A thief sold some stolen goods to a dealer. Several weeks later, the police raided the dealer's store and arrested him. In this raid, the police seized the goods the thief sold to the dealer and a record book in which the dealer had recorded this transaction. However, at the dealer's subsequent trial for receiving stolen goods, the charges against him were dismissed when the court ruled that the search warrant had been improperly issued. The police were able to trace the stolen goods to the thief because of fingerprint identification and the information contained in the dealer's record book. At his trial, the thief made a motion to suppress the stolen goods and record book. What should the judge do? A Grant the motion, because the evidence is the fruit of the poisonous tree in that the search of the dealer's store was improper. B Grant the motion, because the trial court in the dealer's case has already ruled that the evidence was improper. C Deny the motion, because the thief has no standing to object to the search. D Deny the motion, because the thief's fingerprints on the stolen goods were what led to his identification.

ANSWER: C. Deny the motion, because the thief has no standing to object to the search. The court should deny the motion because the thief had no standing to object to the search. A person challenging the admissibility of seized evidence must have standing to do so. As a general rule, standing requires a person to have a reasonable expectation of privacy in the place being searched or the item being seized. One may not challenge a search or seizure by claiming that another person's constitutional rights have been violated. Here, the thief had no ownership interest in the dealer's store. He had no reasonable expectation of privacy with respect to it; i.e., he was not present when the search was made, and he had no ownership interest in the stolen goods. Thus, he lacks the standing to object to their illegal seizure. (A) and (B) are incorrect because, while the dealer does have such standing and was successful in having the evidence suppressed at his trial, what occurred at the dealer's trial is not relevant to the thief's motion. (D) is incorrect because the only evidence containing the thief's fingerprints were the stolen goods. If it is found that these items were illegally seized, it would follow that the evidence arising out of this illegal seizure, including the thief's fingerprints, was also illegally seized.

8. On July 1, a cattle rancher offered to sell his ranch to a dairy farmer for $150,000. The dairy farmer paid the cattle rancher $1,000 to hold the offer open for a period of 30 days. On July 10, the dairy farmer wrote to the cattle rancher, telling him that he could not pay more than $100,000 for the ranch, and that if he would not agree to accept that amount, he would not go through with the deal. The dairy farmer received no reply from the cattle rancher. On July 29, the dairy farmer mailed a letter to the cattle rancher telling him that he accepted his offer to sell the ranch and enclosed a check for $150,000. The cattle rancher received this letter on August 1. Has a contract been formed between the parties for the sale of the ranch? A: No, because the dairy farmers letter of July 10 terminated the cattle ranchers offer. B: No, because the cattle rancher did not accept the dairy farmers counteroffer of $100,000. C: No, because the cattle rancher did not receive the dairy farmers acceptance within 30 days. D: Yes, because the dairy farmer dispatched his acceptance of the cattle ranchers offer prior to the expiration of 30 days.

ANSWER: C. No, because the cattle rancher did not receive the dairy farmers acceptance within 30 days. No contract was formed because the cattle rancher did not receive the dairy farmers acceptance within 30 days. Under the mailbox rule, acceptance by mail or similar means creates a contract at the moment of dispatch. However, the mailbox rule does not apply to option contracts. An acceptance under an option contract is effective only upon receipt. [Restatement (Second) of Contracts §63] Here, an option contract existed because the dairy farmer paid the cattle rancher $1,000 to hold the offer open for 30 days. The dairy farmer mailed his acceptance within 30 days but it was not received by the cattle rancher within the 30-day period, so the acceptance was not effective. The option specified the period of time during which the offer would remain open, after which the offer terminated. Thus, (C) is correct, and (D) is wrong. (A) and (B) are wrong because an option contract is irrevocable for the time period stated. Thus, not even the dairy farmer himself could revoke the offer within the 30-day period.

(Criminal Procedure) 17. The police received information linking a man to drug trafficking and went to the man's residence, where he lived with his mother. The police found the mother at home, and she told them that her son was not expected back until later. The police informed the mother that they suspected the man of selling drugs and asked if they could search his room. She replied, "I'm finished with that no-good bum; not only is he into drugs, but he has been stealing my money to pay for them, and all the time I'm making his bed and fixing his food. You can search his room. He likes to keep his private stuff under his pillow. I hope he goes to jail." The police searched the man's room and discovered a quantity of marijuana under the pillow of his bed. If before trial the man's attorney moves to suppress the marijuana on grounds that the search was invalid, should the court grant the motion? A. Yes, because the man had a legitimate expectation of privacy in the area searched, and the police did not have a warrant. B. Yes, because the man's mother's consent was given at a time when police knew her interests were in conflict with the man's. C. No, because the man's mother had the authority to consent to the search of his room. D. No, because with the mother's statement the police had probable cause to search the room.

ANSWER: C. No, because the man's mother had the authority to consent to the search of his room. The man's motion to suppress should be denied because his mother had authority to consent to the search of his room. A search of a residence can be based on the voluntary consent of the occupant. Where a parent has general access to a room occupied by a son or daughter, the parent can give a valid consent to a general search of the room even if the son or daughter is an adult. The facts in the question indicate that the man's mother had general access to his room ("and all the time I'm making his bed"). Therefore, her consent is valid and eliminates the need for probable cause and a warrant. (A) is wrong. The man had a legitimate expectation of privacy in the area searched, but the consent of his mother eliminated the need for a warrant. (B) is wrong. At one time, some courts required an "amicable relationship" between the parties before the police could rely on a third party's consent. The "amicable relationship requirement" is no longer recognized by the courts. (D) is not a good answer. It is true that with the mother's statement the police had probable cause to search the man's room. However, probable cause alone would not validate the search. The police would need probable cause plus a warrant or a valid consent. In this question the search would have to be based on consent.

(Criminal Procedure) 8. The president of a private college received a report that there was a great deal of cocaine use occurring on the second floor of the dormitory. The president persuaded the school security officers to place several concealed microphones in the second-floor student lounge. Conversations occurring in the lounge were monitored by the security officers and they recorded a conversation in which a sophomore at the college offered to sell cocaine to a freshman. A tape of the conversation was taken to the local police, who played it for a local judge. The judge issued a warrant to search the sophomore's room. While searching the room the police discovered a large amount of cocaine and the sophomore was arrested and charged with unlawful possession of narcotics. His attorney moved to prevent the introduction of the cocaine into evidence. Will the motion most likely be granted? A. Yes, because the sophomore's privacy was unreasonably invaded. B. Yes, because the electronic surveillance was "fundamentally unfair." C. No, because the police properly obtained a search warrant. D. No, because the college president was acting on behalf of the college population in general.

ANSWER: C. No, because the police properly obtained a search warrant. The motion to suppress should be denied because a valid search warrant was obtained. A search warrant must be based on probable cause. Here, there was sufficient information for a judge to conclude that there was probable cause to believe that evidence of a crime would be found in the sophomore's room. Thus, the warrant was properly obtained. (A) is wrong because the sophomore had only a limited expectation of privacy in a dormitory's lounge. Also, even if his privacy had been invaded, any invasion here was done by private persons, not the state, and thus would not prevent introduction of the evidence. (B) is wrong because it is untrue as a matter of law. (D) is wrong because it is not a sufficient basis to deny the sophomore's motion. It is irrelevant that the college president acted on behalf of the college population.

(Property) 1. An owner obtained a loan of $60,000 from a bank in exchange for a promissory note secured by a mortgage on his land, which the bank promptly and properly recorded. A few months later, the owner obtained another loan of $60,000 from a lender, in exchange for a promissory note secured by a mortgage on the land, which the lender promptly and properly recorded. Subsequently, the owner sold the land to a buyer for $150,000 and conveyed a warranty deed. The buyer expressly agreed with the owner to assume both mortgages, with the consent of the bank and the lender. A few years later, the bank loaned the buyer an additional $50,000 in exchange for an increase in the interest rate and principal amount of its mortgage on the land. At that time, the balance on the original loan from the bank was $50,000. Shortly thereafter, the buyer stopped making payments on both mortgages and disappeared. After proper notice to all appropriate parties, the bank instituted a foreclosure action on its mortgage, and purchased the property at the foreclosure sale. At that time the principal balance on the lender's mortgage loan was $50,000. After fees and expenses, the proceeds from the foreclosure sale totaled $80,000. Assuming that the jurisdiction permits deficiency judgments, which of the following statements is most accurate? A The bank keeps the entire $80,000 and can proceed personally against the owner for its deficiency, while the lender's mortgage remains on the land. B The bank keeps the entire $80,000, the lender's mortgage on the land is extinguished, and both the bank and the lender can proceed personally against the owner for their deficiencies. C The bank keeps $50,000, the lender is entitled to $30,000, and only the lender can proceed personally against the owner for its deficiency. D The bank keeps $50,000, the lender is entitled to $30,000, and neither the bank nor the lender can proceed personally against the owner for their deficiencies.

ANSWER: C. The bank keeps $50,000, the lender is entitled to $30,000, and only the lender can proceed personally against the owner for its deficiency. The bank's original mortgage has priority in the proceeds, followed by the lender's mortgage, and only the lender can proceed against the owner because the bank modified its mortgage after the owner had transferred to the buyer. Generally, the priority of a mortgage is determined by the time it was placed on the property, and the proceeds of a foreclosure sale will be used to pay off the mortgages in the order of their priority. However, if the landowner enters into a modification agreement with the senior mortgagee, raising its interest rate or otherwise making the agreement more burdensome, the junior mortgage will be given priority over the modification. Thus, if the first mortgage debt is larger because of the modification, the second mortgage gains priority over the increase in the debt. Here, the bank and the buyer modified the original mortgage by increasing the principal amount and the interest rate. This modification is not given priority over the lender's mortgage, and foreclosure proceeds will not be applied against it because the senior lender's mortgage was not fully satisfied from the proceeds. With regard to the deficiency, the owner is liable to the lender because when a grantee signs an assumption agreement, becoming primarily liable to the lender, the original mortgagor remains secondarily liable on the promissory note as a surety. Here, the buyer assumed the lender's mortgage and became primarily liable; however, the owner remained secondarily liable as surety and can be required to pay off the rest of the lender's mortgage loan. On the other hand, the owner will not be liable to pay off the balance of the bank's loan, because when a mortgagee and an assuming grantee subsequently modify the original obligation, the original mortgagor is completely discharged of liability. The owner had nothing to do with the modification agreed to by the bank and the buyer that increased the amount of the mortgage debt, and will not be even secondarily liable for that amount. (A) and (B) are incorrect because the bank is not entitled to the entire $80,000 in proceeds from the sale and because the owner is not liable to the bank for more than the original loan amount. (D) is incorrect because, as discussed above, the owner is secondarily liable to the lender for the $20,000 deficiency on its mortgage.

14. A sporting goods shop owner placed an online order to one of his regular suppliers for 200 12-inch leather softballs at $5 per ball, the supplier's list price, delivery within seven days. The supplier checked its inventory and discovered that it had only 180 12-inch leather softballs, which it shipped to the shop owner, along with 20 12-inch synthetic softballs. The synthetic softballs had the same list price of$5 per ball. The supplier also emailed to the shop owner the following message: "We did not have enough leather softballs in stock to fill your order. Therefore, we are sending synthetic softballs at the same list price to make up the balance of the shipment—hope you will be able to use them!" Upon receipt of the shipment and the email, what are the shop owner's options? A. The shop owner may accept the conforming part of the shipment and reject the nonconforming part, in which case he must pay the supplier $900 less any damages sustained because of the nonconforming part of the shipment. B. The shop owner may accept the shipment, in which case he must pay the supplier $1,000 less any damages sustained because of the nonconforming shipment. C. The shop owner may reject the shipment, but the supplier will not be liable for breach of contract. D. The shop owner may reject the shipment, in which case he may recover against the supplier for breach of contract.

ANSWER: C. The shop owner may reject the shipment, but the supplier will not be liable for breach of contract. The shop owner may reject the shipment, but the supplier will not be liable for breach of contract because the supplier alerted the shop owner that the shipment was an accommodation. Under Article 2 of the UCC, an offer to buy goods for current or prompt shipment is construed as inviting acceptance either by a promise to ship or by current or prompt shipment. A shipment of nonconforming goods ordinarily is an acceptance creating a bilateral contract as well as a breach of that contract. The result is different if the seller seasonably notifies the buyer that a shipment of nonconforming goods is offered only as an accommodation to the buyer; in that case, the shipment is a counteroffer, not an acceptance and breach, and the buyer is free to accept it or reject it. (A) is incorrect because, as discussed above, the shipment constituted a counteroffer by the supplier, which the shop owner was obligated to wholly accept or reject. Choices (B) and (D) are incorrect because, without a valid contract, the shop owner cannot recover any damages from the supplier.

(Criminal Procedure) 32. A plainclothes police officer who frequently ate lunch at a certain deli heard rumors that the deli's owner often placed illegal bets on sporting events. Based on the rumors, the officer peeked into an envelope next to the register and saw betting slips. The officer asked his waitress about the envelope, and she told him that the owner had given it to her and that a man in a brown cap was to pick it up at 2 pm. The officer stayed at the deli until 2 pm and watched the waitress hand the envelope to a man in a brown cap. The officer passed the above information on to a friend in the F.B.I. Several weeks later, based on that information, F.B.I. agents obtained a search warrant for the owner's home, described in the warrant as a condominium in a large multi-unit complex identified by the street address only. The agents went to the home in the early evening, while the owner was at the deli. After announcing their purpose to the owner's wife, they searched the home and found betting slips and other materials related to illegal gambling. The owner was indicted for conspiracy to violate a federal statute prohibiting the use of interstate phone lines to conduct gambling, and for the possession of betting slips. Which of the following would be the best reason for excluding the evidence found at the owner's home? A The owner was not home when the warrant was executed. B The search was conducted in the evening when it easily could have been conducted during daylight hours. C The warrant failed to specify which condominium unit was to be searched. D The waitress had never been used before as an informant.

ANSWER: C. The warrant failed to specify which condominium unit was to be searched. If the warrant was invalid, any evidence obtained thereunder will be excluded. A warrant must be based upon a showing of probable cause. When requesting a warrant, officers must submit to a magistrate an affidavit setting forth sufficient underlying circumstances to enable the magistrate to make a determination of probable cause, independent of the officers' conclusions, that evidence of a crime will be found at the premises. Also, a warrant must describe with reasonable precision the place to be searched and any items to be seized. A finding that a warrant was invalid because it was not supported by probable cause will not entitle the defendant to exclude evidence obtained under the warrant if the police have acted in good faith and reasonable reliance on a facially valid warrant. However, a police officer cannot in good faith rely on a defective search warrant if: (i) the affidavit underlying the warrant is so lacking in probable cause that no reasonable police officer would have relied on it; (ii) the warrant is defective on its face (e.g., it fails to state with particularity the place to be searched or the things to be seized); (iii) the affiant lied to or misled the magistrate; or (iv) the magistrate has wholly abandoned his judicial role. Because the deli owner lives in a condominium, the warrant should have specified which unit in the multi-unit dwelling was to be searched. Because the warrant did not so specify, it failed to describe with sufficient particularity the place to be searched. Such an absence of precision renders the warrant defective on its face, so that the FBI agents cannot be said to have relied in good faith on a facially valid warrant. Thus, the evidence obtained pursuant to this facially defective warrant will be excluded. (A) and (B) each incorrectly imply that there was something wrong with the execution of the warrant. A warrant must be executed by the police without unreasonable delay, with the police knocking and announcing their purpose (unless they reasonably believe that such notice will endanger them or lead to the destruction of evidence). The FBI agents conducted themselves in accordance with these standards. The fact that a warrant was executed in the evening or at a time when a particular person was not on the premises will not invalidate a search conducted pursuant to the warrant. Here, the agents announced themselves to the deli owner's wife, and they were not required to wait until the owner returned in order to conduct their search. (D) is incorrect because it states a fact that will not, in and of itself, invalidate a warrant and a search pursuant thereto. The sufficiency of a search warrant affidavit based on an informer's hearsay is evaluated under the totality of the circumstances. The informer's reliability and credibility, as well as her basis of knowledge, are all elements that may illuminate the issue of probable cause, but they are not strictly separate requirements. Had the waitress been used before as an informant, and been previously found to be reliable, this would have been one factor in determining her present reliability. However, the fact that the waitress had not previously served as an informer does not invalidate the warrant. Given the specificity and accuracy of the waitress's information (a man in a brown cap would pick it up at 2 pm), there was enough of a basis for her information to establish sufficient credibility to permit a magistrate to make a determination of probable cause.

(Torts) 3. A husband was on his way to meet his wife for lunch at the restaurant in the lobby of a bank building where she worked. He had just entered the building, which was owned and operated by the bank, when he heard screams and the sound of breaking glass from the restaurant area. He immediately saw that a large piece of artwork made of stained glass had fallen onto the seating area of the restaurant. In the seating area he saw several injured persons, including his wife, lying in the wreckage of the artwork. He fainted and hit his head on the marble floor, fracturing his skull. The artwork had collapsed because the pedestal that the bank had provided for the artwork was not properly constructed. If the husband sues the bank for his injury, is he likely to prevail? A. No, because he was not personally in the zone of danger of physical injury. B. No, because he did not actually see the artwork collapse onto the diners. C. Yes, because his wife was one of the persons he saw lying in the wreckage. D. Yes, because the bank had provided the pedestal for the artwork.

ANSWER: C. Yes, because his wife was one of the persons he saw lying in the wreckage. The husband will recover for his injuries because his wife was among those injured by the collapse of the artwork. The duty to avoid negligent infliction of emotional distress may be breached when the defendant creates a foreseeable risk of physical injury to the plaintiff. In most jurisdictions, a bystander who sees the defendant negligently injuring another can recover for his own distress if (i) the plaintiff and the person injured by the defendant's negligence are closely related, (ii) the plaintiff was present at the scene of the injury, and (iii) the plaintiff personally observed or perceived the event. Observation is typically by sight, but may also be by hearing or other senses under certain circumstances. Here, the husband heard the screams and the sound of breaking glass when the artwork collapsed as he entered the lobby. Even though he evidently did not see the artwork collapse on the diners, he heard it crash where his wife was sitting and saw the immediate aftermath. Because his wife was one of the persons injured by the collapse of the artwork, he can recover damages for the injuries caused by his distress. (A) is incorrect because, as stated above, the majority rule allows a bystander to recover based on the factors stated above even if he is outside the zone of danger of physical injury. (B) is incorrect because, as discussed above, a plaintiff who is present at the scene of the injury may perceive the event by hearing or other senses; under the circumstances here, it was not essential that he observe the actual collapse with his eyes. (D) is incorrect because it does not matter that the bank had provided the pedestal. Even if the negligent construction of the pedestal had been done by a third party, the bank remains liable to invitees on its premises because a business has a nondelegable duty to keep its premises safe for customers.

9. On July 1, a cattle breeder, who was planning to retire soon, sent a note to his neighbor offering to sell his prize bull for $15,000. On July 10, the neighbor, who was also a cattle breeder, wrote the following note to the retiring breeder: I have decided to take the bull. I will give you a cashier's check on delivery on Saturday, July 28. The retiring breeder did not respond. The retiring breeder did not want to deliver the bull on July 28 and did not think that the delivery day was agreed to. Instead, he delivered the bull on Monday, July 30. The neighbor refused the delivery and stated that he had found another bull he likes better. The retiring breeder sues the neighbor for breach of contract. Is the retiring breeder likely to prevail? A) Yes, because his breach, if any, was minor. B) Yes, because the parties had not agreed on July 28 as the delivery date. C) No, because there was no contract. D) No, because he did not deliver the bull on July 28.

ANSWER: D) No, because he did not deliver the bull on July 28. The retiring breeder will not prevail because he did not deliver the bull on July 28. This is a contract for a sale of goods and thus is governed by the UCC. Under the UCC, an acceptance with additional terms does not constitute a rejection and counteroffer, but rather is an effective acceptance unless made expressly conditional on the assent to the additional terms. Here, the neighbor accepted the offer and added the additional term of a delivery date. Thus, there was a contract. Whether additional terms become part of the agreement depends on whether both parties are merchants. If both parties to the contract are merchants, additional terms in the acceptance will be included in the contract unless they materially alter the terms of the offer, the offer expressly limited the acceptance to its terms, or they are objected to within a reasonable time. Here, both parties are breeders in the cattle business and, thus, are merchants. The change in the delivery date does not materially change the offer (i.e., it does not change a party's risk or remedies), the offer did not limit the acceptance to its terms, and the retiring breeder did not object. Therefore, the July 28 delivery date became part of the contract. By delivering the bull on July 30th, the retiring breeder breached the contract.

19. A homeowner contracted with a builder for the remodeling of the homeowner's bathroom and kitchen at a cost of $10,000. The contract was in writing and specified that the work was to be completed within two months after the date of execution of the contract. Two weeks after entering into the contract with the homeowner, the builder was offered an extremely lucrative job that would take all of his time and effort for several months. The builder told the homeowner that he was not going to perform. The homeowner diligently called many other contractors over a period of several weeks and none of them could offer a price anywhere near as low as $10,000 for the remodeling work that he wanted done. Two months after entering into the contract with the builder, the homeowner sued the builder for specific performance. What is the likely result of the suit? A) The court will order specific performance, because the homeowner was ready and able to perform his part of the contract. B) The court will order specific performance, because, despite diligent efforts, the homeowner could find no one who could perform the desired services at a competitive price. C) The court will not order specific performance, because the doctrine of laches applies. D) The court will not order specific performance, because the remedy at law is adequate.

ANSWER: D) The court will not order specific performance, because the remedy at law is adequate. (D) Specific performance is available only when monetary damages are inadequate. The court will not order specific performance merely because the builder had a much lower price than anyone else. Instead, the court will require the nonbreaching party to hire a different contractor, and the builder would be liable for the difference between the new price for remodeling and the original $10,000 price. It should be noted that nothing indicates that the builder was hired for his unique talents; even if he were, specific performance would not be granted because of difficulties of supervision and a reluctance to force one person to work for another. Thus, (A) and (B) are incorrect.

5. A software vendor who collected vintage baseball cards as a hobby had in his collection a Roger Maris baseball card, circa 1961, in pristine condition. The vendor knew that the local high school baseball coach would love to add that card to his own collection, so he told the coach that he was interested in selling it. The coach said that he would like to buy the card but did not have the kind of money on hand that he would need. He mentioned that he would, however, be getting a bonus in three weeks. The vendor replied that he could wait three weeks and proceeded to write the following on a piece of paper and hand it to the coach: "I will sell my 1961 Roger Maris baseball card to the coach for $2,000. This offer is good for the next 30 days." The vendor then signed the paper. The coach took the paper and went home. Two weeks later, before the coach could pay the $2,000 to the vendor, the vendor incurred an unexpected debt and gave the baseball card to his creditor as repayment of the debt. The vendor then called the coach to tell him that he was revoking his offer to sell the baseball card to the coach. The coach, who had cleared off his mantel and built a little wooden stand on which to display the card, filed suit against the vendor. Will the coach likely be successful? A. Yes, because the offer to sell was irrevocable for 30 days. (OPTION CONTRACT) B. Yes, because the vendor wrongfully prevented the fulfillment of the contractual condition. C. No, because the confirmatory writing is not binding where only one party is a merchant. D. No, because the vendor's call revoked the offer. (extra answers that are similar from a very similar question) E. Yes, because the agreement between the parties was for the sale of a good worth more than $500, and the revocation was not in writing. F. Yes, on the ground of promissory estoppel. G. No, because this was merely an offer for a unilateral contract that was revocable prior to acceptace.

ANSWER: D. No, because the vendor's call revoked the offer. The coach will not be successful in his suit, because the vendor's phone call revoked the offer prior to acceptance. The vendor's writing was an offer to sell the card to the coach for $2,000. Note that the coach did not make an offer because there was no expression of promise, undertaking, or commitment. He merely expressed that he would like the card but did not have cash. He did not propose any terms. Since the coach's statement was not an offer, the vendor's writing could not be an acceptance. Despite the fact that the vendor's offer states that it is good for 30 days, it is revocable at any time. Generally, offers can be revoked at will by the offeror, even if he has promised not to revoke for a certain period. This power to revoke is limited only by option contracts, the start of performance of a unilateral contract, or merchant's firm offers. None of these exceptions apply here. This is not an offer for a unilateral contract, and in any case, the coach's actions do not amount to the beginning of performance. The offer also was neither an option contract nor a merchant's firm offer. An option is a promise to keep an offer open for an agreed-upon time and requires consideration. Here, the coach gave no consideration to the vendor to hold the offer open for 30 days, so this cannot be an option contract. Consideration is not required to make the offer irrevocable if a merchant signs a writing giving assurances that the offer will be held open during the time stated (or, if no time is stated, for a reasonable time not to exceed three months). With a couple of notable exceptions (e.g., warranty of merchantability), most anyone in business is considered a merchant for purposes of Article 2. However, for the merchant rules to apply, the party must be acting in his mercantile capacity. Here, the vendor is in the software sales business and would be considered a merchant for any transaction involving his business—even if the transaction itself did not involve software. This transaction, however, involved the vendor's personal life (his hobby) rather than his business. Thus, he was not acting in his mercantile capacity and the merchant rules on firm offer and confirmatory memo would not apply to this transaction. Thus, the offer was revocable, and (A) is incorrect. (B) is incorrect for two reasons: First, there was no contract, so there was no contractual condition. Second, even if this were a contract, the vendor did not interfere with the condition of the coach paying $2,000. The fulfillment of that condition was completely within the coach's control, not the vendor's. (C) is incorrect because the writing is an offer, not a contract or confirmatory memo. If this were a contract, it would be one for the sale of goods priced at more than $500; thus, it would require a writing signed by the party to be charged. The confirmatory memo rule is a method of complying with the Statute of Frauds when there is an oral agreement by merchants, memorialized in a memo, but not signed by the party to be charged. Here, there is a writing signed by the party to be charged (the vendor), so there is no need to resort to the confirmatory memo rule. Moreover, to satisfy the Statute of Frauds and bind both parties with a merchant's confirmatory memo of an oral agreement, both parties must be merchants. As noted above, neither party is a merchant for purposes of this transaction.

13. An automobile dealer agreed to sell a car to a buyer for $30,000, with a down payment of $6,000 due at the time the sales contract was signed and the balance payable in monthly installments over a period of five years. Under the written contract, delivery would be made within 30 days. Two weeks after making the down payment, the buyer told the dealer that he lost his job and could not afford to go through with the purchase. The dealer, which could get as many of that model of car as it required from the manufacturer for a wholesale price of $21,000, put the car in question back in its inventory but refused to return the buyer's down payment. A short time later, the dealer sold it to someone else for $28,500. The buyer sues the dealer to get back his deposit, and the dealer counterclaims for damages. For purposes of this question, do not include incidental damages in your calculations. Who will prevail and for what amount? A The buyer will recover $6,000. B The buyer will recover $4,500. C The dealer will recover $9,000. D The dealer will recover $3,000.

ANSWER: D. The dealer will recover $3,000. The dealer will recover $3,000, which is the difference between its lost profits and the buyer's down payment. When the buyer repudiates or refuses to accept goods, the usual measure of the seller's damages is the difference between the contract price and the market price or the difference between the contract price and the resale price of the particular goods. However, neither of those measures of damages gives adequate compensation for the buyer's breach where the seller has an unlimited supply of the goods (i.e., a lost volume seller), because, but for the buyer's breach, the seller would have made two sales instead of one. In this type of case, lost profit is measured by the contract price less the cost to the seller. Here, the dealer could have made two sales of that model of car because it could get as many as it needed from the manufacturer. Hence, it lost a profit of $9,000 as a result of the buyer's breach. This amount is offset against the amount of the down payment that the buyer made, resulting in a net recovery of $3,000 by the dealer.

2. A homeowner offered a landscape gardener $1,000 to trim and reshape the bushes on her property if the gardener could finish the job before her garden party on June 1. The gardener told the homeowner that he would get back to her after he had checked his calendar. The next day, the gardener phoned the homeowner, who was not at home, and left a message on her voicemail that he had the time, but could not do the job for less than $1,200. The gardener did not hear from the homeowner for several days. As June 1 drew closer, the gardener phoned the homeowner again and left another message on her voicemail stating that "I'll do the job for $1,000, this weekend, unless that would be inconvenient." The homeowner replayed the second message just as she was leaving town on a business trip and did not contact the gardener. That weekend, unbeknownst to the homeowner, the gardener took his tools to the homeowner's house and trimmed and reshaped the bushes to the homeowner's specifications. When the homeowner returned from her trip several days later, the gardener presented her with a handwritten invoice for $1,000. If the homeowner refuses to pay the gardener, and the latter brings an action solely for breach of contract to recover the $1,000 contract amount, who will likely prevail? A. The gardener, because the homeowner knew of the gardener's plans to do the landscaping job over the weekend in question, putting the burden on the homeowner to call off the job if she did not want the gardener to perform. B. The gardener, because this was a unilateral contract, the terms of which the gardener accepted by performing his duties under the contract. C. The homeowner, because she revoked her offer when she ignored the gardener's subsequent phone calls agreeing to do the job at the original price. D. The homeowner, because she did not accept the gardener's offer to do the landscaping job for $1,000.

ANSWER: D. The homeowner, because she did not accept the gardener's offer to do the landscaping job for $1,000. The homeowner will likely prevail on the breach of contract claim because she did not enter into a contract with the gardener. To form a contract, there must be a valid offer and acceptance. The homeowner made an offer, but the gardener rejected the offer the next day with his first phone call. Once an offer is rejected, the offeree's power of acceptance is destroyed. Thus, the gardener's second call was not an acceptance, but rather a counteroffer. The homeowner did nothing to accept the gardener's counteroffer, and this is not the type of case where silence will be deemed to be an acceptance (e.g., where the parties have so agreed or where that has been their course of dealing). Thus, there was no acceptance and no contract to breach. (A) is incorrect because an offeree cannot be forced to speak under penalty of having silence treated as an acceptance. If an offeree silently takes offered benefits, the courts will often find acceptance, especially if prior dealings between the parties, or trade practices known to both parties, create a commercially reasonable expectation by the offeror that silence represents an acceptance; in such cases, the offeree is under a duty to notify the offeror if she does not intend to accept. As discussed above, the gardener rejected the initial offer and made a counteroffer, putting the homeowner in the position of offeree. The homeowner's silence cannot be construed as acceptance absent a showingof prior dealings between the parties or trade practices known to both. Moreover, the gardener's last phone call to the homeowner was somewhat ambiguous, and the homeowner could argue that she did not know with certainty that the gardener would still perform the job after getting no response. Further, she was not home that weekend and, therefore, did not stand idly by and knowingly accept the gardener's work. (B) is incorrect because this was not a unilateral contract. A unilateral contract exists only when an offeror makes acceptance possible only by performing a stipulated act, whereas a bilateral contract contemplates an exchange of promises. Here, the homeowner asked the gardener whether he would perform an act and expected a reply from the gardener; indeed, the gardener told the homeowner that he would telephone her with his answer the next day, which he did (in effect, rejecting the offer). Thus, this was not a unilateral contract that could properly be accepted through performance. (C) is incorrect because the gardener's power of acceptance was destroyed when he rejected the homeowner's offer, as discussed above, and once that occurred, the offer was gone and could not be revoked.

(Property) 5. A first-time home buyer financed the purchase of a house with a $100,000 mortgage she took out with a bank. The mortgage was recorded. A few years later she borrowed $5,000 from a finance company to pay for a foreign trip, using her house as security. The finance company promptly and properly recorded its mortgage on the property. One year after that, she borrowed $40,000 from an equity company to pay for an addition on the house. The equity company promptly and properly recorded the mortgage it took on the property. Shortly thereafter, she lost her job and was unable to make payments on either the finance company's or the equity company's mortgages, but she was able to make payments on the bank's mortgage. The finance company filed foreclosure of its mortgage and included the equity company in the action, and a purchaser bought the property at the foreclosure sale. What is the purchaser's obligation regarding the bank's mortgage and the equity company's mortgage? A. The purchaser takes the property subject to both mortgages. B. The purchaser takes the property subject to neither mortgage. C. The purchaser takes the property subject to the equity company's mortgage, but not subject to the bank's mortgage. D. The purchaser takes the property subject to the bank's mortgage, but not subject to the equity company's mortgage.

ANSWER: D. The purchaser takes the property subject to the bank's mortgage, but not subject to the equity company's mortgage. A foreclosure sale wipes out all junior mortgages (those that come later in time than the mortgage that was foreclosed) but does not wipe out senior mortgages (those that came earlier). Because the bank's mortgage preceded the finance company's it is senior and is not wiped out. The purchaser takes subject to this mortgage.

(Property) 6. A landowner owned a large tract of land containing numerous coal mines. To finance the renovation of some of the buildings on the land, the landowner obtained a $50,000 mortgage from a bank. Shortly thereafter, the landowner, without notifying anyone of the bank's interest, sold the surface of the land to his sister and the mineral rights to a utility company. The bank recorded its mortgage the next day; the day after that, the utility company recorded its deed; the following day, the sister recorded her deed. None of the parties dealing with the landowner had any knowledge of the others at the time of their transactions. The jurisdiction in which the land is located has the following statute: "No conveyance or mortgage of an interest in land is valid against any subsequent purchaser for value without notice thereof whose conveyance is first recorded." (RACE-NOTICE) If the sister brings an action to quiet title to the land, what would be the most likely result? A The sister would have only a reversionary interest. B The bank's mortgage would be valid and superior simply because it was first in time. C The sister would be deemed the owner in fee simple absolute and subject only to the payment of the mortgage held by the bank. D The sister would have a fee simple interest subject to the mineral rights of the utility company and the mortgage held by the bank.

ANSWER: D. The sister would have a fee simple interest subject to the mineral rights of the utility company and the mortgage held by the bank. The sister's fee simple ownership of the land would be subject to the bank's mortgage interest and the utility company's mineral interest. Under a race-notice statute, which the jurisdiction in this question has, a subsequent bona fide purchaser (i.e., one who takes for value and without notice) is protected only if she records before the prior grantee. Notice is measured at the time of the conveyance, not at the time of recording. The rationale of this type of statute is that the best evidence of which interest was created first is to determine who recorded first. As an inducement to record promptly, race-notice statutes impose on the bona fide purchaser the additional requirement that she record first. Because the bank was the first to receive a conveyance, the bank could not be held to have knowledge of any other conveyance, and when the bank recorded its conveyance first, the bank won out over the sister and the utility company under the statute. The utility company owns the mineral interest in coal on the land because it recorded before the sister. (A) is incorrect because the sister has a present ownership interest in the land, but it is subject to the bank's mortgage and the utility company's mineral interest. (B) is incorrect because the jurisdiction has a race-notice statute. Thus, the bank's interest is superior only if it is first in time and without notice of all other interests. (C) is incorrect because, as discussed above, the sister does not have a fee simple absolute; the utility company owns the mineral interest.

12. A dealer sent an e-mail to a wholesaler stating "Send 500 'Granny Rocker' chairs at your usual price." The wholesaler responded, also by e-mail, "Will ship our last 500 'Granny Rocker' chairs at $100 per chair, our usual price. 'Granny Rocker' line is being discontinued." The wholesaler's staff immediately began the paperwork for processing the order and started preparing and packing the chairs for shipment. The dealer e-mailed back to the wholesaler, "Cancel order for 'Granny Rocker' chairs; your price is too high." The dealer had found a mill outlet that was the only other source of the chairs. Although the outlet's price was also $100 per chair, it was more convenient for the dealer to buy the chairs from the outlet. The day after receiving the dealer's cancellation, the wholesaler was able to sell the 500 "Granny Rocker" chairs in its stock to a furniture chain for $100 each. The wholesaler's staff immediately began the paperwork for processing the order and repacked the chairs for shipment to the chain's stores. If the wholesaler sues the dealer for damages, how much should the wholesaler recover? A Nothing, because this was a contract between merchants and the dealer canceled within a reasonable time. B Nothing, because the wholesaler was able to cover by selling the chairs at the same price it would have received from the dealer. C $50,000, the full contract price, because the dealer breached the contract and $100 per chair was a fair price. D The wholesaler's incidental costs of preparing the paperwork and other office costs connected with preparing and repacking the chairs for shipment to the dealer.

ANSWER: D. The wholesaler's incidental costs of preparing the paperwork and other office costs connected with preparing and repacking the chairs for shipment to the dealer. The wholesaler will recover only its incidental damages, i.e., the costs of preparing to ship the chairs to the second buyer. An offer calling for shipment of goods, such as the offer here, may be accepted by prompt shipment with notice or by a promise to ship. Acceptance forms a contract. Here, the wholesaler accepted the dealer's offer by promising to ship (the warning that the wholesaler had no more chairs was unimportant surplusage since it could fill the dealer's order), and a contract was formed. The dealer breached the contract by canceling its order. When a buyer breaches by repudiating its offer, as the dealer did here, the seller has a right to recover either the difference between the contract price and the market price or the difference between the contract price and the resale price, plus incidental damages. Here, the chairs were resold. The resale price was the same as the contract price, so there are no losses there. If the seller is a lost volume seller, meaning they can obtain as many goods as they can sell, the seller will have lost out on an additional sale and is entitled to profits from that lost sale. Here, the wholesaler is not a lost volume seller because it is not able to obtain additional chairs—these were the last of the inventory and were discontinued—and so the wholesaler did not lose out on an additional sale. Therefore, the wholesaler lost no profits. It made what it would have made if the sale with the dealer had gone through. Thus, the wholesaler would be limited to its incidental damages under the first two measures of damages. (A) is incorrect because there is no rule under the UCC, which governs the contract here, that makes contracts between merchants cancelable within a reasonable time. (B) is incorrect because, as indicated above, the UCC allows the seller to recover incidental damages. (C) is incorrect because the UCC seeks only to put the nonbreaching party in as good a position as it would have been in had the other party performed, and here, awarding the wholesaler lost profits would put it in a better position than performance would have, since it would give the wholesaler a double recovery for selling the same goods. (The result would be different, however, if the wholesaler had had more chairs to sell, because in that case, the breach would have cost the wholesaler additional sales—i.e., the wholesaler could have sold to the dealer and to the other party.)

32. On September 15, a highlighter manufacturer faxed a large office supply company offering to sell the supply company 50,000 highlighters for $25,000. The supply company faxed back the following communication: We accept your offer. Please box 125 highlighters per case in post-consumer cardboard shipping boxes. Assuming the existence of a valid contract, what would its terms include? A: Only those terms set forth in the manufacturer's fax of September 15, because the manufacturer did not assent to any enlargement of the shipping terms. B: All terms set forth in the manufacturer's offer plus consistent additional terms proposed in the office supply company's acceptance. C: All terms set forth in the manufacturer's offer plus those in the office supply company s attempted acceptance that did not amount to a material alteration of the manufacturer's offer. D: All terms set forth in the manufacturer's offer plus all those in the office supply company's purported acceptance that did not amount to a material alteration of the manufacturer's offer and to which the manufacturer did not object within a reasonable time.

ANSWER: D: All terms set forth in the manufacturer's offer plus all those in the office supply company's purported acceptance that did not amount to a material alteration of the manufacturer's offer and to which the manufacturer did not object within a reasonable time. The language in choice (D) properly states the UCC position regarding the terms of the contract. Under the UCC, if both parties to a contract are merchants, additional terms in an acceptance will be included in the contract unless (i) they materially alter the original contract; (ii) the offer expressly limits acceptance to the terms of the offer; or (iii) the offeror has already objected to the particular terms, or objects within a reasonable time after notice of them is received. [UCC §2-207(2)] The manufacturer and office supply company are both merchants because they regularly deal in goods. [UCC §2-104(1)] Therefore, under Article 2 the contract will include the terms of the manufacturer's offer plus those in the office supply company's purported acceptance that did not amount to a material alteration of the offer or to which the manufacturer did not object within a reasonable time. Note that the manufacturer's offer did not expressly limit acceptance of its terms. (A) is incorrect because this was a contract between two merchants. Contract formation under the UCC for contracts between merchants is governed by the rule stated above. If one of the parties were not a merchant, (A) would be correct if one of the parties to a contract for sale of goods is not a merchant and the acceptance includes additional or different terms, such terms are considered to be mere proposals that do not become part of the contract unless the offeror accepts. [UCC §2-207(1)] However, because this is a contract between merchants, the office supply company's terms regarding shipping will be included unless they materially alter the offer, as discussed above. (B) is incorrect because it does not fully state the Code's battle of the forms provision. The choice fails to mention the manufacturer's power to object within a reasonable time. (C) is incorrect because, like (B), it does not note the manufacturer's power to object within a reasonable time.

31. (Similar to #8) On January 1, a car salesman offered to sell an antique car to a collector for $35,000 cash on delivery. The collector paid the car salesman $100 to hold the offer open for a period of 25 days. On January 4, the collector called the car salesman and left a message on his answering machine, asking him whether he would consider lowering the price to $30,000. The car salesman played back the message the same day but did not reply. On January 9, the collector wrote the car salesman a letter, telling him that he could not pay more than $30,000 for the antique car, and that if the car salesman would not accept that amount, he would not go through with the deal. The car salesman received this letter on January 10 and again did not reply. The car salesman never heard from the collector again. When did the offer that the car salesman made to the collector on January 1 terminate? A: On January 4, when the collector made a counteroffer. B: On January 9, when the collector mailed to the car salesman what amounted to a rejection. C: On January 10, when the car salesman received from the collector what amounted to a rejection. D: On January 25, when the 25-day option expired.

ANSWER: D: On January 25, when the 25-day option expired. The car salesman's offer terminated on January 25, when the 25-day option expired. An option is a distinct contract in which the offeree gives consideration for a promise by the offeror not to revoke an outstanding offer. The collector paid the car salesman $100 to hold the offer open for a period of 25 days, and the offer could not be terminated before that time, not even by the offeree (here the collector). Nor did the offer survive the option period because the option specifically identified how long the offer would be open. (A) is incorrect because the collector's words did not amount to a counteroffer because they merely inquired as to whether the collector would consider lowering his price; the words were not unequivocal. (Even if this were a counteroffer, it would not extinguish the car salesman's offer, because of the option, as explained above.) (B) is incorrect because, as discussed above, even a rejection by the offeree will not terminate the option. Also, if the communication were effective as a rejection, it would be effective when received by the offeror. (C) is incorrect because, as discussed above, even unequivocal words of rejection by the offeree will not extinguish an option, absent detrimental reliance on the part of the offeror, which was not the case here.

(Criminal Procedure) 5. A suspect was arrested for burglarizing an apartment. He was duly given Miranda warnings, and invoked his right to remain silent. When the suspect was put into the lockup, the police took from his him wallet, watch, and other personal possessions. Following standard procedure, a police officer immediately began to make an inventory of the suspect's personal effects. During the course of the inventory, the officer noticed that the suspect's watch bore an inscription with the name of a person whose apartment had been burglarized two days earlier. The officer concluded that the suspect had probably burglarized that apartment as well as the one for which he was arrested. She reported the inscription on the watch to the detective who has arrested the suspect, and the suspect was subsequently charged with the earlier burglary as well. Did the officer violate the suspect's constitutional rights by reading the inscription? (A) Yes, because items to be inventoried may be listed, but they may not be closely examined. (B) Yes, because no search warrant was obtained. (C) No, because the inventory was a routine procedure of the kind the police normally conduct when an incarceration takes place. (D) No, because it have her probable cause to believe that the suspect had committed the earlier burglary.

MY ANSWER (NOT BARBRI'S): (C) No, because the inventory was a routine procedure of the kind the police normally conduct when an incarceration takes place.


Set pelajaran terkait

Principles of Management - Final - Belmont

View Set

Nutrition Chapter 10: Trace Minerals

View Set

SAS Programming 2: Data Manipulation Techniques

View Set

Chapter 9 (unit 4) practice quiz

View Set

Legal Environment of Business Exam 3 (chapter 11)

View Set

CH. 16 - Managing Change & Stress

View Set